You are on page 1of 56

Labor Law 1 A2010 - 37 - Disini

DMA SHIPPING PHILS INC V CABILLAR UY V BUENO


452 SCRA 551 484 SCRA 628
CALLEJO, SR; February 28, 2005 PUNO; March 14, 2006
NATURE NATURE
Petition for Review Petition for review on certiorari of a decision of the CA

FACTS FACTS
- Henry Cabillar was hired by Monsoon, through DMA Shipping, as Chief Officer of the - Amalia Bueno was the Manager of Countrywide Rural Bank of La Carota, Inc. (bank
M/V Eagle Moon. hereafter) Marbel Branch. She was verbally and summarily dismissed by Atty. Andrea Uy,
- After three (3) months, Cabillar wrote the manager of Monsoon, requesting for an early interim President and Corporate Secretary of the bank, during a depositors' meeting.
repatriation and for his reliever grounded on the failure of DMA Shipping to give the - Bueno filed a case for illegal dismissal and prayed for reinstatement with full backwages
promised additional allowance. Monsoon approved an increase in Cabillar’s wage and and damages.
the latter withdrew his request for repatriation.
- While the vessel was docked in India, the gantry crane operators refused to work and ISSUE
demanded for an increase in their allowance. The master of the M/V Eagle Moon WON Uy is an officer of the bank, making her soldarily liable with the corporation for
instructed Cabillar to talk to the crew members under his immediate supervision to illegal dismissal
convince them not to proceed with the intended strike and have the matter discussed
with the management when the vessel returns to Singapore. HELD
- Instead of talking to the crew members, Cabillar himself joined the strike. Monsoon NO
expressed its displeasure on Cabillar for joining the strike. Nevertheless, Monsoon - The minutes of the depositors' meeting clearly showed that Uy was a mere depositor of
agreed to the demands of the striking crew members to avert any further losses. the bank. She was only elected as officer of the Interim Board of Directors craeted by the
- When the vessel arrived at Singapore, officers of Monsoon informed Cabillar that he association of depositors with the sole task of rehabilitating the bank (which is under
has been separated from his employment because of the incident in Calcutta. receivership).
- Cabillar filed a complaint with the POEA against DMA and Monsoon seeking payment - There is no evidence that the association of depositors that elected the interim board
for the unexpired portion of his contract. was recognized by BSP. Hence, it had no legal authority to act for the bank.
- The Executive Labor Arbiter rendered a decision in favor of Cabillar declaring his - The act of dismissing Bueno by Uy cannot be deemed as an act as an officer of the
dismissal as illegal. The NLRC and the Court of Appeals affirmed. Hence this petition. bank. Consequently, it cannot be held that there existed an employer-employee
relationship between Uy and Bueno.
ISSUES - The requirement of employer-employee relationship is jurisdictional for the provisions of
1. WON the respondent was dismissed by the petitioner Monsoon and the Labor Code on Post-employment to apply. Since such relationship was not
2. If so, WON his dismissal was for a valid cause established, the labor arbiter never acquired jurisdiction over Uy.
3. WON the respondent is entitled to backwages, damages and attorney’s fees Disposition CA decision finding Uy solidarily liable with the bank reversed
HELD
1. Petitioner was dismissed. CABRERA V NLRC (NATIONAL SERVICE
Ratio WON the respondent was dismissed or that he resigned as chief officer of the CORPORATION, VILLAMOR)
vessel is a question of fact. The labor arbiter ruled that the respondent was dismissed. 198 SCRA 573
The NLRC and the Court of Appeals affirmed. Petitioners failed to make a clear showing
CRUZ; June 27, 1991
that the findings were are arbitrary and bereft of any rational basis.
Reasoning
- The entry in the logbook of the vessel shows that the ship captain, for and in behalf of NATURE
the petitioners, dismissed the respondent for joining the strike. Appeal from the decision of the NLRC dismissing the complaint for illegal dismissal by
- The petitioners failed to adduce documentary evidence to prove their allegation that (1) the petitioners on the ground that it is without jurisdiction.
they and the respondent agreed that in consideration for the respondent’s resignation,
they would give him a “very good” rating (2) they defrayed his plane fare back to the FACTS
Philippines (3) they paid for his hotel bills in Singapore. - Dismissed by the National Service Corporation, the petitioners complained to the
2. Respondent was dismissed for just cause. Ministry of Labor and Employment on September 17, 1980. After considering the position
Ratio Under their employment contracts, the crew and officers of the vessel bound papers of the parties, the Labor Arbiter ordered the petitioners' reinstatement without loss
themselves to follow certain procedures for their grievances. of seniority rights and the payment to them of two years back wages and other benefits. 3
Reasoning The decision was appealed to and affirmed by the First Division of the NLRC on
- The crew and the respondent refused to follow the procedure and stop the strike. December 9, 1985, and in due time, the petitioners moved for the issuance of a writ of
- They may have a valid grievance against the petitioners but they are bound to follow execution. This was opposed by NASECO on the ground that it had not been furnished
the procedures set forth in their contracts of employment to address said grievances. with a copy of the decision, but the opposition was rejected and the petition was granted.
3. Petitioners are to pay indemnity. Reconsideration of the order having been denied, the NASECO appealed to the NLRC,
Ratio The petitioners themselves violated their contracts of employment with the which, through its Third Division this time, declared itself without jurisdiction and
respondent and the crew because the captain of the vessel failed to comply with the dismissed the case on August 18, 1987. 4 Citing the NHA case, the public respondent
disciplinary procedures. held that the NASECO was not covered by the Labor Code but by Civil Service rules and
Reasoning regulations, being a government-owned or controlled corporation.
- The respondent was not furnished with any written notice of any charges against him.
- There was no formal investigation of the charges. ISSUE
- Respondent was not furnished with a copy of the written notice of the penalty imposed WON the National Service Corporation is covered by the Labor Code
on him.
- For such violation, petitioners are liable for moral damages or for indemnity of P30,000, HELD
if the respondent fails to prove such moral damages. 1 In this case, the respondent failed YES
to prove such moral damages. - The decision in National Housing Corporation v. Juco was already overturned by the
Disposition AFFIRMED with MODIFICATION. Petitioners are ordered to pay P30,000 decision in National Service Corporation v. NLRC. The NLRC erred in dismissing the
by way of indemnity. The awards for other damages and attorney’s fees are deleted. petitioners' complaint for lack of jurisdiction because the rule now is that only
government-owned or controlled corporations with original charters come under the Civil
Service. The NASECO having been organized under the Corporation Law and not by
virtue of a special legislative charter, its relations with its personnel are governed by the
Labor Code and come under the jurisdiction of the National Labor Relations Commission.
1
Agabon v. NLRC, G.R. No. 158693, 442 SCRA 573, Nov. 17, 2004.
Labor Law 1 A2010 - 38 - Disini
GAMUGAMO V PNOC SHIPPING AND TRANSPORT - It may also be pointed out that upon his receipt of the amount of P512,524.15 from
respondent as retirement benefit pursuant to its retirement scheme, petitioner signed and
CORP delivered to respondent a release and undertaking wherein he waives all actions, causes
381 SCRA742 of actions, debts, dues, monies and accounts in connection with his employment with
DAVIDE JR; May 7, 2002 respondent. This quitclaim releases respondent from any other obligation in favor of
petitioner.
NATURE
Special Civil Action in the Supreme Court. Certiorari LIGHT RAILWAY TRANSIT AUTHORITY V VENUS
485 SCRA 301
FACTS
- On January 23,1963, petitioner Cayo Gamogamo was employed with the Department of PUNO; March 24, 2006
Health as Dental Aide (wherein he was also promoted to the position of Dentist 1). He
remained employed at the DOH for 14 years until he resigned on November 2, 1977. FACTS
- On November 9, 1977, petitioner was hired as a company dentist by Luzon Stevedoring - consolidated petitions of Light Rail Transit Authority (LRTA) and Metro Transit
Corporation (LUSTEVECO). Subsequently, respondent PNOC Shipping and Transport Organization, Inc. (METRO), seeking the reversal of the Decision of the Court of Appeals
Corp acquired and took over the shipping business of LUSTEVECO. Petitioner was directing them to reinstate private respondent workers to their former positions without
among those who opted to be absorbed by the respondent. He continued to work as a loss of seniority and other rights and privileges, and ordering them to jointly and severally
company dentist. pay the latter their full back wages, benefits, and moral damages. The LRTA and
- ON June 10,1993, President Fidel V. Ramos issued a memorandum approving the METRO were also ordered to jointly and severally pay attorney’s fees equivalent to ten
privatization of PNOC subsidiaries, including respondent pursuant to the provisions of percent (10%) of the total money judgment.
Section III (B) of the Guidelines and Regulations to implement E.O. No. 37. Respondent - Petitioner LRTA is a government-owned and controlled corporation created by
implemented a Manpower Reduction Program to govern employees whose respective Executive Order No. 603, Series of 1980, as amended, to construct and maintain a light
positions have been classified as redundant (respondent decreased its operations and rail transit system and provide the commuting public with an efficient, economical,
downsized its organization due to lay up and sale of its vessels. dependable and safe transportation. Petitioner METRO, formerly Meralco Transit
- Sometime in 1995, petitioner requested to be included in the next retrenchment Organization, Inc., was a qualified transportation corporation duly organized in
schedule. However, his request was turned down because: 1.) he was holding a accordance with the provisions of the Corporation Code, registered with the Securities
permanent position 2.) he was already due for mandatory retirement in April 1995 under and Exchange Commission, and existing under Philippine laws. petitioner LRTA, after a
his retirement plan. Eventually petitioner retired after serving respondent for 17 years and bidding process, entered into a ten (10)-year Agreement for the Management and
4 mos. He received a retirement pay which is equivalent to one month pay for every year Operation of the Metro Manila Light Rail Transit System from June 8, 1984 until June 8,
of service and other benefits (P512,524.15) 1994 with petitioner METRO.
- On August 30,1995, respondent’s president died and was replaced by Nemesio - The Agreement provided, among others, that –
Prudente who implemented significant cost-saving measures. He ordered that a study on “4. METRO shall be free to employ such employees and officers as it shall deem
the cost-effect of the retrenchment of employees be conducted (upon motion of 2 other necessary in order to carry out the requirements of [the] Agreement. Such employees
employees, Dr. Rogelio Buena (company doctor) and Mrs. Luz C. Reyes (telephone and officers shall be the employees of METRO and not of the AUTHORITY [LRTA].
operator) who were holding permanent/non-redundant positions. These 2 employees METRO shall prepare a compensation schedule and the corresponding salaries and
were retrenched and paid a 2-month separation pay for every year of service under fringe benefits of [its] personnel in consultation with the AUTHORITY [LRTA] [par.
Respondent’s Manpower Reduction Program. 3.05];”
- In view of the action taken by respondent in the retrenchment of the said 2 employees, - On July 25, 2000, the Union filed a Notice of Strike with the National Conciliation and
petitioner filed a complaint at the NLRC for the full payment of his retirement benefits. Mediation Board – National Capital Region against petitioner METRO on account of a
Petitioner argued that his service with the DOH should have been included in the deadlock in the collective bargaining negotiation. On the same day, the Union struck.
computation of his years of service. Hence, with an accumulated service of 32 years he They completely paralyzed the operations of the entire light rail transit system. As the
should have been paid a 2-month pay for every year of service per the retirement plan strike adversely affected the mobility of the commuting public, then Secretary of Labor
(and thus should have received at least P1,833,920) Bienvenido E. Laguesma issued on that same day an assumption of jurisdiction order [3]
- The Labor Arbiter dismissed petitioner’s complaint. On appeal however, the NLRC directing all the striking employees “to return to work immediately upon receipt of this
reversed the decision of the Labor Arbiter (considering the 14 years of his service to Order and for the Company to accept them back under the same terms and conditions of
DOH) employment prevailing prior to the strike
- Respondent filed with the CA a special civil action for certiorari. CA set aside the - Despite the issuance, posting, and publication of the assumption of jurisdiction and
judgment of the NLRC. Hence, petitioner filed this petition alleging that 1.) his years of return to work order, the Union officers and members failed to return to work. Thus,
service with the DOH must be considered as creditable service for the purpose of effective July 27, 2000, private respondents, were considered dismissed from
computing his retirement pay 2.) he was discriminated against in the application of the employment
Manpower Reduction Program. - Workers filed a complaint for illegal dismissal. On October 1, 2001, Labor Arbiter Luis
D. Flores rendered a consolidated judgment in favor of the private respondent workers
ISSUE - On May 29, 2002, on appeal, the NLRC found that the striking workers failed to heed
WON, for the purpose of computing an employee’s retirement pay, prior service rendered the return to work order and reversed and set aside the decision of the labor arbiter. The
in a government agency can be tacked in and added to the creditable service later suit against LRTA was dismissed since “LRTA is a government-owned and controlled
acquired in a government-owned and controlled corporation without original charter. corporation created by virtue of Executive Order No. 603 with an original charter” and “it
ha[d] no participation whatsoever with the termination of complainants’ employment.” In
HELD fine, the cases against the LRTA and METRO were dismissed, respectively, for lack of
NO jurisdiction and for lack of merit.
- Petition denied and the appealed decision in CA is affirmed (in favor of respondent) - On a petition for certiorari however, the Court of Appeals reversed the NLRC and
Ratio The Court cannot uphold petitioner’s contention that his 14 years of service with reinstated the Decision rendered by the Labor Arbiter. Public respondent appellate court
the DOH should be considered because his last 2 employers were government-owned declared the workers’ dismissal as illegal, pierced the veil of separate corporate
and controlled corporations and fall under the Civil Service Law. Article IX (B), Section 2 personality and held the LRTA and METRO as jointly liable for back wages.
paragraph 1 of the 1987 Constitution states: “The civil service embraces all branches,
subdivisions, instrumentalities, and agencies of the Government, including government ISSUE
owned or controlled corporations with original charters”. While respondent and WON LRTA should be held liable for the illegal dismissal of employees
LUSTEVECO are government-owned and controlled corporations, they have no original
charters; hence, they are not under the Civil Service Law HELD
Reasoning NO
- totalization of service credits is only resorted to when the retiree does not qualify for - petitioner LRTA argues that it has no employer-employee relationship with private
benefits in either of both systems. In this case, petitioner is qualified to receive benefits respondent workers as they were hired by petitioner METRO alone pursuant to its ten
granted by the GSIS, if such right has not yet been exercised. (10)-year Agreement for the Management and Operation of the Metro Manila Light Rail
Transit System with petitioner METRO. Piercing the corporate veil of METRO was
Labor Law 1 A2010 - 39 - Disini
unwarranted, as there was no competent and convincing evidence of any wrongful, the standards under which he must qualify as such. He prayed for reinstatement with
fraudulent or unlawful act on the part of METRO, and, more so, on the part of LRTA. backwages; P3,155.00 for probationary and P3,445.00 for regular salary adjustments;
- Petitioner LRTA further contends that it is a government-owned and controlled value of lodging or dormitory privileges; cost of insurance coverage for group life,
corporation with an original charter, Executive Order No. 603, Series of 1980, as medical, death, dismemberment and disability benefits; moral, and exemplary, and
amended, and thus under the exclusive jurisdiction only of the Civil Service Commission, nominal damages plus interest on the above claims with attorney's fees.
not the NLRC. - Answering the complaint, ICMC claimed that petitioner failed to quality for regular
- We agree with petitioner LRTA. Section 2 (1), Article IX – B, 1987 Constitution, employment because he showed no interest in improving his professional performance
expressly provides that “[t]he civil service embraces all branches, subdivisions, both in and out of the classroom after he had been periodically evaluated; that petitioner
instrumentalities, and agencies of the Government, including government-owned or was paid his salary up to December 31, 1985, two weeks pay in lieu of notice, and 14th
controlled corporations with original charters.” Corporations with original charters are month pay pro-rata; and that his accrued leave balance already been converted to cash.
those which have been created by special law and not through the general corporation - After the parties had formally offered their evidence, private respondents submitted
law. their memorandum on July 31, 1989 in which, among other things, they invoked ICMC's
- In Philippine National Oil Company – Energy Development Corporation v. Hon. diplomatic immunity on the basis of the Memorandum of Agreement signed on July 15,
Leogrado- “under the present state of the law, the test in determining whether a 1988 between the Philippines government and ICMC.
government-owned or controlled corporation is subject to the Civil Service Law is the - The Labor Arbiter held that ICMC's legal immunity under the Memorandum could not be
manner of its creation such that government corporations created by special charter are given retroactive effect since "[that would] deprive complainant's property right without
subject to its provisions while those incorporated under the general Corporation Law are due process and impair the obligation of contract of employment." He also expressed
not within its coverage.” doubt on the ground that it was provided for by agreement and not through an act of
- There should be no dispute then that employment in petitioner LRTA should be Congress. Accordingly, the Labor Arbiter ordered ICMC to reinstate petitioner as regular
governed only by civil service rules, and not the Labor Code and beyond the reach of the teacher without loss of seniority rights and to pay him one year backwages, other
Department of Labor and Employment, since petitioner LRTA is a government-owned benefits, and ten percent attorney's fees for a total sum of P70,944.85.
and controlled corporation with an original charter, Executive Order No. 603, Series of - Both parties appealed to the NLRC. On August 13, 1990, petitioner moved to dismiss
1980 private respondent's appeal because of the latter's failure to post a cash/surety bond. In
- In contrast, petitioner METRO is covered by the Labor Code despite its later acquisition its order of October 13, 1992, however, the NLRC ordered the case dismissed on the
by petitioner LRTA. In Lumanta v. National Labor Relations Commission, this ground that, under the Memorandum of Agreement between the Philippine government
Court ruled that labor law claims against government-owned and controlled corporations and ICMC, the latter was immune from suit.
without original charter fall within the jurisdiction of the Department of Labor and
Employment and not the Civil Service Commission ISSUE
- We therefore hold that the employees of petitioner METRO cannot be considered as WON the Memorandum of Agreement executed on July 15, 1988 granted ICMC
employees of petitioner LRTA. The employees hired by METRO are covered by the immunity from suit
Labor Code and are under the jurisdiction of the Department of Labor and Employment,
whereas the employees of petitioner LRTA, a government-owned and controlled HELD
corporation with original charter, are covered by civil service rules. Herein private YES
respondent workers cannot have the best of two worlds, e.g., be considered government Ratio The grant of immunity from local jurisdiction to ICMC . . . is clearly necessitated by
employees of petitioner LRTA, yet allowed to strike as private employees under our labor their international character and respective purposes. The objective is to avoid the
laws. danger of partiality and interference by the host country in their internal workings. The
exercise of jurisdiction by the Department of Labor in these instances would defeat the
very purpose of immunity, which is to shield the affairs of international organizations, in
EBRO III V NLRC (INTERNATIONAL CATHOLIC accordance with international practice, from political pressure or control by the host
MIGRATION COMMISSION) country to the prejudice of member State of the organization, and to ensure the
261 SCRA 399 unhampered performance of their functions. (International Catholic Migration
MENDOZA; September 4, 1996 Commission v. Calleja)
Reasoning
- The grant of immunity to ICMC is in virtue of the Convention on the Privileges and
NATURE
Immunities of Specialized Agencies of the United Nations, adopted by the UN General
Petition for review on certiorari to set aside the order dated October 13, 1992 and the
Assembly on November 21, 1947, and concurred in by the Philippine Senate on May 17,
resolution dated March 3, 1993 of the National Labor Relations Commission.
1949. This Convention has the force and effect of law, considering that under the
Constitution, the Philippines adopts the generally accepted principles of international law
FACTS
as part of the law of the land.
- Private respondent International Catholic Migration Commission (ICMC) is a non-profit
- The scope of immunity of the ICMC contained in the Convention on the Privileges and
agency engaged in international humanitarian and voluntary work. It is duly registered
Immunities of the Specialized Agencies of the United Nations is instructive. Art. III, §4 of
with the United National Economic and Social Council (ECOSOC) and enjoys
the Convention provides for immunity from "every form of legal process." Thus, even if
Consultative Status, Category II. It was one of the agencies accredited by the Philippine
private respondents had been served summons and subpoenas prior to the execution of
Government to operate the refugee processing center at Sabang, Morong, Bataan.
the Memorandum, they, as officers of ICMC, can claim immunity under the same in order
- On June 24, 1985, private respondent ICMC employed petitioner Jose G. Ebro III to
to prevent enforcement of an adverse judgment, since a writ of execution is "a legal
teach "English as a Second Language and Cultural Orientation Training Program" at the
process" within the meaning of Article III, §4.
refugee processing center. The employment contract provided in pertinent part:
- Art III §4 of the Convention on the Privileges and Immunities of the Specialized
Salary: Your monthly salary for the first 6 months probationary period is P3,155.00
Agencies of the United Nations requires that the waiver of the privilege must be express.
inclusive of cost of living allowance. Upon being made regular after successful
There was no such waiver of immunity in this case. Nor can ICMC be estopped from
completion of the six (6) months probationary period your monthly salary will be
claiming diplomatic immunity since estoppel does not operate to confer jurisdiction to a
adjusted to P3,445.00 inclusive of cost of living allowance
tribunal that has none over a cause of action.
If either party wishes to terminate employment, a notice of two (2) weeks should be
- Finally, neither can it be said that recognition of ICMC's immunity from suit deprives
given in writing to the party.
petitioner of due process. As pointed out in ICMC v. Calleja, petitioner is not exactly
- After six months, ICMC notified petitioner that effective December 21, 1985, the latter's
without remedy for whatever violation of rights it may have suffered for the following
services were terminated for his failure to meet the requirements of "1. classroom
reason:
performance . . . up to the standards set in the Guide for Instruction; 2. regular
Section 31 of the Convention on the Privileges and Immunities of the Specialized
attendance in the mandated teacher training, and in the schedule team meetings, one-
Agencies of the United Nations provides that "each specialized agency shall make
on-one conferences with the supervisor, etc.; and 3. compliance with ICMC and PRPC
provision for appropriate modes of settlement of: (a) disputes arising out of contracts or
policies and procedures."
other disputes of private character to which the specialized agency is a party." Moreover,
- On February 4, 1986, petitioner filed a complaint for illegal dismissal, unfair labor
pursuant to Article IV of the Memorandum of Agreement between ICMC and the
practice, underpayment of wages, accrued leave pay, 14th month pay, damages,
Philippine Government, whenever there is any abuse of privilege by ICMC, the
attorney's fees, and expenses of litigation. Petitioner alleged that there was no objective
Government is free to withdraw the privileges and immunities accorded.
evaluation of his performance to warrant his dismissal and that he should have been
Disposition Petition is DISMISSED for lack of merit.
considered a regular employee from the start because ICMC failed to acquaint him with
Labor Law 1 A2010 - 40 - Disini
NATIONAL MINES AND ALLIED WORKERS UNION V the reviewing court may delve into the records and examine for itself the questioned
findings.
SAN ILDEFONSO COLLEGE Disposition The Petition is DENIED.

CHIANG KAI SHEK COLLEGE V CA (NLRC, AUSTRIA V NLRC (CENTRAL PHIL. UNION MISSION
CALAYLAY, AQUINO, GACUTAN, BELO) CORP. OF THE 7TH-DAY ADVENTIST)
437 SCRA 171 312 SCRA 410
DAVIDE, JR; August 24, 2004 KAPUNAN; August 16, 1999
FACTS
FACTS
- In 1992, Ms. Diana Belo, a teacher of Chiang Kai Shek College since 1977, applied for
- Pastor Dionisio Austria worked with the Central Philippine Union Mission Corporation of
a leave of absence for the school year 1992-1993. Upon submitting her application, she
the Seventh Day Adventists (SDA) for 28 years from 1963 to 1991. He began his work
was informed of the school policy that if she takes a leave of absence, she is not assured
with the SDA on 15 July 1963 as a literature evangelist, selling literature of the SDA over
of a teaching load upon her return. She was likewise informed that only teachers in active
the island of Negros. From then on, he worked his way up the ladder and got promoted
service may enjoy the privilege and benefits provided by the school, such as free tuition
several times. In January, 1968, he became the Assistant Publishing Director in the West
for the teachers’ children.
Visayan Mission. In July, 1972, he was elevated to the position of Pastor covering the
- Ms. Belo, nonetheless, took her leave of absence. In May 1993, she attempted to return
island of Panay, and the provinces of Romblon and Guimaras. He held the same position
to CKSC and signified her readiness to teach for the coming school year. However, she
up to 1988. Finally, in 1989, he was promoted as District Pastor of the Negros Mission of
was not allowed to return. Hence, she filed a complaint for illegal dismissal, among
the SDA and was assigned at Sagay, Balintawak and Toboso, Negros Occidental, with 12
others, against CKSC.
churches under his jurisdiction. In January, 1991, he was transferred to Bacolod City. He
- The Labor Arbiter dismissed the complaint but the NLRC disagreed. The Court of
held the position of district pastor until his services were terminated on 31 October 1991.
Appeals upheld the NLRC’s ruling. Hence, this petition.
- On various occasions from August up to October, 1991, Eufronio Ibesate, the treasurer
of the Negros Mission asked him to admit accountability and responsibility for the church
ISSUE
tithes and offerings collected by his wife, Thelma Austria, in his district which amounted
WON private respondent was constructively dismissed
to P15,078.10, and to remit the same to the Negros Mission. Petitioner reasoned out that
he should not be made accountable for the unremitted collections since it was Pastor
HELD
Gideon Buhat and Ibesate who authorized his wife to collect the tithes and offerings
YES
since he was very sick to do the collecting at that time.
- Under the Manual of Regulations for Private Schools, for a private school teacher to
- On 16 October 1991, petitioner went to the office of Pastor Buhat, the president of the
acquire a permanent status of employment and, therefore, be entitled to a
Negros Mission. During said call, petitioner tried to persuade Pastor Buhat to convene
security of tenure, the following requisites must concur: (a) the teacher is a full-
the Executive Committee for the purpose of settling the dispute between him and Pastor
time teacher; (b) the teacher must have rendered three consecutive years of
David Rodrigo. The dispute between David Rodrigo and petitioner arose from an incident
service; and (c) such service must have been satisfactory. Since Ms. Belo has
in which petitioner assisted his friend, Danny Diamada, to collect from Pastor Rodrigo the
measured up to these standards, she therefore enjoys security of tenure.
unpaid balance for the repair of the latter's motor vehicle which he failed to pay to
- Constructive dismissal is defined as a cessation from work because continued
Diamada. Due to the assistance of petitioner in collecting Pastor Rodrigo's debt, the
employment is rendered impossible, unreasonable, or unlikely; when there is a demotion
latter harbored ill-feelings against petitioner. When news reached petitioner that Pastor
in rank or a diminution in pay or both; or when a clear discrimination, insensibility, or
Rodrigo was about to file a complaint against him with the Negros Mission, he
disdain by an employer becomes unbearable to the employee.
immediately proceeded to the office of Pastor Buhat and asked the latter to convene the
- Ms. Belo was constructively dismissed when the petitioners, in implementing their
Executive Committee. Pastor Buhat denied the request of petitioner since some
policies, effectively barred her from teaching for the school year 1993-1994. The three
committee members were out of town and there was no quorum. Thereafter, the two
policies are (1) the non-assurance of a teaching load to a teacher who took a leave of
exchanged heated arguments. Petitioner then left the office of Pastor Buhat. While on his
absence; (2) the hiring of non-permanent teachers in April to whom teaching loads were
way out, petitioner overheard Pastor Buhat saying "Pastor daw inisog na ina iya (Pastor
already assigned when Ms. Belo signified in May 1993 her intention to teach; and (3) the
you are talking tough)." Irked by such remark, petitioner returned to the office of Pastor
non-applicability to children of teachers on leave of the free tuition fee benefits extended
Buhat, and tried to overturn the latter's table, though unsuccessfully, since it was heavy.
to children of teachers in service.
Thereafter, petitioner banged the attache case of Pastor Buhat on the table, scattered the
- Ms. Belo was definitely singled out in the implementation of a future policy (i.e., the
books in his office, and threw the phone. Fortunately, Pastors Yonillo Leopoldo and
policy that employees not in service are not entitled to any benefit extended by the
Claudio Montaño were around and they pacified both.
school). The petitioners did not take heed of the principle enshrined in our labor laws that
- On 17 October 1991, petitioner received a letter inviting him and his wife to attend the
policies should be adequately known to the employees and uniformly implemented to the
Executive Committee meeting. From October 21 to 22, the fact-finding committee
body of employees as a whole and not in isolation.
conducted an investigation. Petitioner immediately wrote Pastor Rueben Moralde,
- The continued employment of Ms. Belo was also rendered unlikely by the insistence of
president of the SDA and chairman of the fact-finding committee, requesting that certain
the petitioners in implementing the alleged policy that a teacher who goes on leave for
members of the fact-finding committee be excluded in the investigation and resolution of
one year is not assured of a teaching load. While this alleged policy was mentioned in
the case. Out of the 6 members requested to inhibit themselves from the investigation
Mr. Chien’s letter of 9 June 1992, it was not included in the school’s written statement of
and decision-making, only 2 were actually excluded: Pastor Buhat and Pastor Rodrigo.
policies dated 13 March 1992. Hence, it was then a non-existent policy. When a non-
Subsequently, petitioner received a letter of dismissal citing misappropriation of
existent policy is implemented and, in this case, only to Ms. Belo, it constitutes a clear
denominational funds, willful breach of trust, serious misconduct, gross and habitual
case of discrimination.
neglect of duties, and commission of an offense against the person of employer's duly
- Petitioners’ invocation of the third policy – that of giving teaching assignments to
authorized representative, as grounds for the termination of his services.
probationary teachers in April – to justify their refusal to provide Ms. Belo a teaching load
is a lame excuse that rings of untruth and dishonesty. Patently clear is the illegal manner
ISSUES
by which the petitioners eased out Ms. Belo from the teaching corps.
1. WON the Labor Arbiter/NLRC has jurisdiction to try and decide the complaint filed by
- Likewise, we do not find merit in petitioners’ assertion that the Court of Appeals should
petitioner against the SDA
not have passed upon the illegality of the school policy of non-assurance of a teaching
2. WON the termination of the services of petitioner is an ecclesiastical affair, and, as
load, since the alleged illegality was never raised as an issue before the respondent
such, involves the separation of church and state
court or in the forums below. As pointed out by the private respondent, that policy was
3. WON such termination is valid
part of the defense invoked by the petitioners in the Arbiter level, in the NLRC, and in the
respondent court to the charge of illegal dismissal; and, hence, it must necessarily be
HELD
passed upon and scrutinized. Besides, that policy is intimately intertwined with the main
1. YES and 2. NO [Resolved jointly since they are related]
issue of whether Ms. Belo was illegally dismissed.
Ratio An ecclesiastical affair is one that concerns doctrine, creed or form or worship of
- This case is an exception to the general rule that the factual findings and conclusions of
the church, or the adoption and enforcement within a religious association of needful
the Labor Arbiter are accorded weight and respect on appeal, and even finality. For one
laws and regulations for the government of the membership, and the power of excluding
thing, the findings of the NLRC and the Labor Arbiter are contrary to each other; hence,
from such associations those deemed unworthy of membership.
Labor Law 1 A2010 - 41 - Disini
Reasoning personally collected and received from the church treasurers the tithes, collections, and
- Based on this definition, an ecclesiastical affair involves the relationship between the donations for the church. The testimony of Naomi Geniebla, the Negros Mission Church
church and its members and relate to matters of faith, religious doctrines, worship and Auditor and a witness for private respondents, show that Pastor Austria was able to remit
governance of the congregation. To be concrete, examples of this so-called all his collections to the treasurer of the Negros Mission. Private respondents try to pin on
ecclesiastical affairs to which the State cannot meddle are proceedings for petitioner the alleged non-remittance of the tithes collected by his wife. In the absence of
excommunication, ordinations of religious ministers, administration of sacraments and conspiracy and collusion, which private respondents failed to demonstrate, between
other activities which attached religious significance. The case at bar does not even petitioner and his wife, he cannot be made accountable for the alleged infraction
remotely concern any of the above cited examples. While the matter at hand relates to committed by his wife. After all, they still have separate and distinct personalities. Thus,
the church and its religious minister it does not ipso facto give the case a religious the allegation of breach of trust has no leg to stand on.
significance. Simply stated, what is involved here is the relationship of the church as an - Misconduct has been defined as improper or wrong conduct. It is the transgression of
employer and the minister as an employee. It is purely secular and has no relation some established and definite rule of action, a forbidden act, a dereliction of duty, willful
whatsoever with the practice of faith, worship or doctrines of the church. In this case, in character, and implies wrongful intent and not mere error in judgment. For misconduct
petitioner was not excommunicated or expelled from the membership of the SDA but was to be considered serious it must be of such grave and aggravated character and not
terminated from employment. merely trivial or unimportant. Based on this standard, we believe that the act of petitioner
- Aside from these, SDA admitted in a certification issued by its officer, Ibesate, that in banging the attache case on the table, throwing the telephone and scattering the
petitioner has been its employee for 28 years. SDA even registered petitioner with the books in the office of Pastor Buhat, although improper, cannot be considered as grave
SSS as its employee. The worker's records of petitioner have been submitted by private enough to be considered as serious misconduct. After all, as correctly observed by the
respondents as part of their exhibits. From all of these it is clear that when the SDA Labor Arbiter, though petitioner committed damage to property, he did not physically
terminated the services of petitioner, it was merely exercising its management assault Pastor Buhat or any other pastor present during the incident of 16 October 1991.
prerogative to fire an employee which it believes to be unfit for the job. As such, the In fact, the alleged offense committed upon the person of the employer's representatives
State, through the Labor Arbiter and the NLRC, has the right to take cognizance of the was never really established or proven by private respondents. Hence, there is no basis
case. for the allegation that petitioner's act constituted serious misconduct or that the same
- Finally, private respondents are estopped from raising the issue of lack of jurisdiction for was an offense against the person of the employer's duly authorized representative.
the first time on appeal. The active participation of a party coupled with his failure to - The final ground alleged by private respondents, gross and habitual neglect of duties,
object to the jurisdiction of the court or quasi-judicial body is tantamount to an invocation does not requires an exhaustive discussion. All private respondents had were allegations
of that jurisdiction and a willingness to abide by the resolution of the case and will bar but not proof. Aside from merely citing the said ground, private respondents failed to
said party from later on impugning the court or body's jurisdiction. prove culpability. In fact, the evidence on record shows otherwise. Petitioner's rise from
3. NO. the ranks proves that he was actually a hard-worker. Private respondents' evidence,
Reasoning which consisted of petitioner's Worker's Reports, revealed how petitioner travelled to
- The issue being the legality of petitioner's dismissal, the same must be measured different churches to attend to the faithful under his care. Indeed, he labored hard for the
against the requisites for a valid dismissal, namely: (a) the employee must be afforded SDA, but, in return, he was rewarded with a dismissal from the service for a non-existent
due process, i.e., he must be given an opportunity to be heard and to defend himself, cause.
and; (b) the dismissal must be for a valid cause as provided in Article 282 of the Labor Disposition Finding of the Labor Arbiter that petitioner was terminated from service
Code. Without the concurrence of these twin requirements, the termination would, in the without just or lawful cause is SUSTAINED. Petitioner is entitled to reinstatement without
eyes of the law, be illegal. loss of seniority right and the payment of full back-wages without any deduction
As to Due Process corresponding to the period from his illegal dismissal up to the actual reinstatement.
- Article 277(b) of the Labor Code further require the employer to furnish the employee Challenged Resolution of NLRC is NULLIFIED and SET ASIDE.
with 2 written notices, to wit: (a) a written notice served on the employee specifying the
ground or grounds for termination, and giving to said employee reasonable opportunity
PEÑARANDA V BAGANGA PLYWOOD CORP
within which to explain his side, and, (b) a written notice of termination served on the
employee indicating that upon due consideration of all the circumstances, grounds have [PAGE 1]
been established to justify his termination.
- The first notice, which may be considered as the proper charge, serves to apprise the CBTC EMPLOYEES UNION V CLAVE
employee of the particular acts or omissions for which his dismissal is sought. The
141 SCRA 9
second notice on the other hand seeks to inform the employee of the employer's decision
to dismiss him. This decision, however, must come only after the employee is given a DE LA FUENTE; January 7, 1986
reasonable period from receipt of the first notice within which to answer the charge and
ample opportunity to be heard and defend himself with the assistance of a NATURE
representative, if he so desires. Non-compliance therewith is fatal because these Petition for certiorari seeking to annul and set aside the decision of the respondent
requirements are conditions sine quo non before dismissal may be validly effected. Presidential Executive Assistant
- SDA failed to substantially comply with the above requirements. With regard to the first
notice, the letter dated 17 October 1991, which notified petitioner and his wife to attend FACTS
the meeting on 21 October 1991, cannot be construed as the written charge required by - Commercial Bank and Trust Company Employees' Union lodged a complaint with the
law. A perusal of the said letter reveals that it never categorically stated the particular Department of Labor, against Comtrust Bank for non-payment of the holiday pay benefits
acts or omissions on which his impending termination was grounded. In fact, the letter provided for under Art 95 of the Labor Code in relation to Rule X, Book III of the Rules
never even mentioned that he would be subject to investigation. The letter merely and Regulations Implementing the Labor Code.
mentioned that he and his wife were invited to a meeting wherein what would be - Failing to arrive at an amicable settlement at conciliation level, the parties opted to
discussed were the alleged unremitted church tithes and the events that transpired on 16 submit their dispute for voluntary arbitration. The issue presented was: "Whether the
October 1991. For this reason, it cannot be said that petitioner was given enough permanent employees of the Bank within the collective bargaining unit paid on a monthly
opportunity to properly prepare for his defense. While admittedly, SDA complied with the basis are entitled to holiday pay effective November 1, 1974, pursuant to Article 95 (now
second requirement, the notice of termination, this does not cure the initial defect of lack Article 94) of the Labor Code, as amended and Rule X (now Rule IV), Book III of the
of the proper written charge required by law. Rules and Regulations Implementing the Labor Code. "
As to Just Cause - In addition, the disputants signed a Submission Agreement stipulating as final,
- Private respondents allege that they have lost their confidence in petitioner for his unappealable and executory the decision of the Arbitrator, including subsequent
failure, despite demands, to remit the tithes and offerings which were collected in his issuances for clarificatory and/or relief purposes, notwithstanding Article 262 of the Labor
district. Settled is the rule that under Article 282 (c) of the Labor Code, the breach of trust Code.
must be willful. A breach is willful if it is done intentionally, knowingly and purposely, - In the course of the hearing, the Arbitrator apprised the parties of an interpretative
without justifiable excuse, as distinguished from an act done carelessly, thoughtlessly, bulletin on "holiday pay" about to be issued by the Department of Labor.
heedlessly or inadvertently. It must rest on substantial grounds and not on the employer's - The Union filed a Manifestation stating that in the event that said Interpretative Bulletin
arbitrariness, whims, caprices or suspicion, otherwise, the employee would eternally regarding holiday pay would be adverse to the present claim union respectfully reserves
remain at the mercy of the employer. It should be genuine and not simulated. This the right to take such action as may be appropriate to protect its interests, a question of
ground has never been intended to afford an occasion for abuse, because of its law being involved. An Interpretative Bulletin which was inexistent at the time the said
subjective nature. The records show that there were only 6 instances when petitioner
Labor Law 1 A2010 - 42 - Disini
commitment was made and which may be contrary to the law itself should not bar the Disposition The questioned decision set aside and the award of the arbitrator
right of the union to claim for its holiday pay benefits reinstated.
- Voluntary Arbitrator stated that, there is more reason to believe that, if the Bank has
never made any deduction from its monthly-paid employees for unworked Saturdays,
SONZA V ABS-CBN BROADCASTING CORPORATION
Sundays, legal and special holidays, it is because there is really nothing to deduct
properly since the monthly, salary never really included pay for such unworked days-and 431 SCRA 587
which give credence to the conclusion that the divisor '250' is the proper one to use in CARPIO; June 10, 2004
computing the equivalent daily rate of the monthly-paid employees; that “both the decree
itself and the Rules mentioned enumerated the excepted workers. It is a basic rule of NATURE
statutory construction that putting an exception limits or modifies the enumeration or Petition for review on certiorari
meaning made in the law. It is thus easy to see that a mere reading of the Decree and of
the Rules would show that the monthly-paid employees of the Bank are not expressly FACTS
included in the enumeration of the exception.” - ABS-CBN signed Agreement with Mel and Jay Management and Devt Corp (MJMDC),
- Voluntary Arbitrator directed the bank to pay its monthly paid employees their “legal which agreed to provide Sonza’s services exclusively to the network as talent for radio
holiday pay.” and TV.
- The next day, the Department of Labor released Policy Instructions No. 9 which clarifies - Sonza resigned and complained that network didn’t pay his salaries, separation pay,
controversies on the entitlement of monthly paid employees. The new determining rule is service incentive leave pay, etc. ABS filed Motion to Dismiss because there was no
this: If the monthly paid employee is receiving not less than P 240, the maximum monthly employer-employee relationship. ABS continued to remit Sonza’s monthly talent fees.
minimum wage, and his monthly pay is uniform from January to December, he is - Labor Arbiter dismissed complaint because of lack of jurisdiction. NLRC affirmed
presumed to be already paid the ten (10) paid legal holidays. However, if deductions are Arbiter’s decision. Sonza filed certiorari action with CA, which dismissed the case.
made from his monthly salary on account of holidays in months where they occur, then Hence this petition.
he is still entitled to the ten (10) paid legal holidays.
- Bank appealed to NLRC but appeal was dismissed because it was filed way beyond the ISSUE
ten-day period for perfecting an appeal and because it contravened the agreement that WON there was an employer-employee relationship between ABS-CBN and Sonza
the award shall be final and unappealable.
- Acting Secretary of Labor reversed NLRC decision and ruled that the appeal was filed HELD
on time and that a review of the case was inevitable as the money claim exceeded NO
P100,000.00. - This is the first Court resolution on nature of relationship between a station and a talent.
- Presidential Executive Assistant affirmed DOJ ruling, relying heavily on the - There are 4 elements of employer-employee relationship:
Manifestation and Policy Instructions No. 9. 1. Selection of employee
Petitioner’s Claim - if Sonza didn’t possess his skills, talents and celebrity status, ABS-CBN would not
The legal presumption established in Section 2, Rule IV, Book 111 2, of the Rules and have entered into agreement with him but would have hired him through personnel
Regulations implementing particularly Article 94 (formerly Article 208) of the Labor Code, department
is merely a disputable presumption 2. Payment of wages
Respondent’s Comments - whatever Sonza received arose from the contract and not from the employer-
- The Bank maintains that, since its inception or start of operations in 1954, all monthly- employee relation
paid employees in the Bank are paid their monthly salaries without any deduction for - the talent fee is so huge that it indicates more a contractual than an employment
unworked Saturdays, Sundays, legals and special holidays. On the other hand, it also relationship
maintains that, as a matter of fact, 'always conscious of its employee who has to work, 3. Power to dismiss
on respondent's rest days of Saturdays and Sundays or on a legal holiday, an employee - ABS-CBN couln’t retrench Sonza because it is obligated to pay talent fees for
who works overtime on any of said days is paid one addition regular pay for the day plus duration of contract
50% of said regular pay 4. Control on employee on means and methods
- The Bank further maintains that the holiday pay is intended only for daily-paid workers. - also called control test; most impt to determine relationship
- The question submitted for arbitration is now moot and academic. - Sonza contends ABS exercised control over means and methods of his work.
Court said ABS merely reserved the right to modify the program format and airtime
ISSUE schedule. Its sole concern was the quality of the show and the ratings. How
WON the permanent employees of the bank are entitled to holiday pay Sonza appeared, sounded, etc. is outside control of ABS.
- Sonza contends that ABS exercised control in providing equipment and crew.
HELD Court said these are not tools needed by Sonza. What he needed were his talent,
YES skills, costume.
- In excluding the union members of herein petitioner from the benefits of the holiday pay - Sonza contends that ABS subjected him to rules and standards. Court said that
law, public respondent predicated his ruling on Section 2, Rule IV, Book III of the Rules to the rules are the TV and Radio Code of the Kapisanan ng Broadcaster sa Pilipinas,
implement Article 94 of the labor Code promulgated by the then Secretary of labor and merely adopted by ABS as its code of ethics. It applies to broadcasters, not just to
Policy Instructions No. 9. ABS employees. Besides, these rules are merely guidelines.
- In Insular Bank of Asia and America Employees' Union (IBAAEU) vs. Inciong, 7 this - Sonza said his exclusivity is a form of control by ABS. Court said exclusivity is a
Court's Second Division, speaking through former Justice Makasiar, expressed the view widespread practice in entertainment industry, as protection of investment in
and declared that the aforementioned section and interpretative bulletin are null and void, “building up” a talent. Besides, the huge talent fees of an exclusive talent
having been promulgated by the then Secretary of Labor in excess of his rule-making compensates for exclusivity.
authority . It was pointed out, inter alia, that in the guise of clarifying the provisions on - Arbiter ruled that as talent of MJMDC, Sonza is not an employee of ABS. Sonza insists
holiday pay, said rule and policy instructions in effect amended the law by enlarging the that MJMDC is a labor-only contractor and ABS is his employer. In labor-only contract,
scope of the exclusions. there are 3 parties – the contractor, employee and the principal (deemed the real
- The questioned Section 2, Rule IV, Book III of the Integrated Rules and the Secretary's employer). Under this, the contractor is the agent of the principal. If Sonza’s argument
Policy Instruction No. 9 add another excluded group, namely, 'employees who are was true, then MJMDC turns to be the agent of both Sonza and ABS. Besides, in the
uniformly paid by the month'. While the additional exclusion is only in the form of a Agreement, there were only two parties mentioned – Sonza and ABS, with MJMDC as
presumption that all monthly paid employees have already been paid holiday pay, it Sonza’s agent.
constitutes a taking away or a deprivation which must be in the law if it is to be valid. An - Sonza argues Policy Instruction No. 40 by Minister of Labor said the types of
administrative interpretation which diminishes the benefits of labor more than what the employees in broadcast are the station and program employees. Court said this
statute delimits or withholds is obviously ultra vires. instruction is a mere executive issuance not binding on the Court.
- Court also said that Arbiter can decide a case without a formal trial.
2
SECTION 2. Status of employees paid by the month -Employees who are uniformly paid by the month, irrespective
- Sonza argues that treating talents as contractors violates right to security of tenure.
of the number of' working days therein with a salary of not less than the statutory or established minimum wage, shall Court said this right exists only if there is an employer-employee relation. Besides, law
be presumed to be paid for all days in the month whether worked or not. also protects rights of talents to contract. Besides, if hosts were employees, managers
For this purpose, the monthly minimum wage shall not be less than the statutory minimum wage multiplied by 365 days can dictate what hosts will say, and this is not conducive to press freedom.
divided by twelve.
Labor Law 1 A2010 - 43 - Disini
- Difference in tax treatment also showed that there’s no employer-employee relation. DUNCAN ASSOCIATION OF DETAILMAN-PTGWO V
- Sonza’s claim is based on their agreement. Therefore, action should not be based on
GLAXO WELLCOME PHILIPPINES INC
Labor Code but on breach of contract.
438 SCRA 343
RIZAL EMPIRE INSURANCE GROUP V NLRC (RUIZ, TINGA; September 17, 2004
CORIA)
NATURE
150 SCRA 565 Petition for review on certiorari of the decision and resolution of the Court of Appeals
PARAS; May 29, 1987
FACTS
NATURE - Petitioner Tecson was hired by respondent Glaxo Wellcome Philippines, Inc. (Glaxo) as
Petition for certiorari medical representative after Tecson had undergone training and orientation. Thereafter,
Tecson signed a contract of employment which stipulates, among others, that he agrees
FACTS to study and abide by existing company rules; to disclose to management any existing or
- August, 1977- Coria was hired by Rizal Empire Insurance Group(REIG) as a casual future relationship by consanguinity or affinity with co-employees or employees of
employee with a salary of P10.00 a day. competing drug companies and should management find that such relationship poses a
- January 1, 1978- Coria was made a regular employee, having been appointed as clerk- possible conflict of interest, to resign from the company.
typist, with a monthly salary of P300.00. - The Employee Code of Conduct of Glaxo similarly provides that an employee is
- Being a permanent employee, he was furnished a copy of petitioner company's expected to inform management of any existing or future relationship by consanguinity or
"General Information, Office Behavior and Other Rules and Regulations." affinity with co-employees or employees of competing drug companies. If management
- In the same year, without change in his position-designation, he was transferred to the perceives a conflict of interest or a potential conflict between such relationship and the
Claims Department and his salary was increased to P450.00 a month. employee’s employment with the company, the management and the employee will
- 1980- he was transferred to the Underwriting Department and his salary was increased explore the possibility of a “transfer to another department in a non-counterchecking
to P580.00 a month plus cost of living allowance, until he was transferred to the Fire position” or preparation for employment outside the company after six months.
Department as filing clerk. - Tecson was initially assigned to market Glaxo’s products in the Camarines Sur-
- July, 1983- he was made an inspector of the Fire Division with a monthly salary of Camarines Norte sales area. Subsequently, Tecson entered into a romantic relationship
P685.00 plus allowances and other benefits. with Bettsy, an employee of Astra Pharmaceuticals (Astra), a competitor of Glaxo. Bettsy
- Oct. 15, 1983- Coria was dismissed from work, on the grounds of tardiness and was Astra’s Branch Coordinator in Albay. She supervised the district managers and
unexcused absences. medical representatives of her company and prepared marketing strategies for Astra in
- Coria filed a complaint with MOLE that area. Even before they got married, Tecson received several reminders from his
- March 14, 1985- LA Ruiz reinstated him to his position with back wages. District Manager regarding the conflict of interest which his relationship with Bettsy might
- REIG appealed to the NLRC but was dismissed on the ground that the same had been engender. Still, love prevailed, and Tecson married Bettsy.
filed out of time. Hence this petition. - Tecson’s superiors informed him that his marriage to Bettsy gave rise to a conflict of
interest. Tecson’s superiors reminded him that he and Bettsy should decide which one of
ISSUE them would resign from their jobs, although they told him that they wanted to retain him
WON it is still within the jurisdiction of the SC as much as possible because he was performing his job well.
- Tecson requested for time to comply with the company policy against entering into a
HELD relationship with an employee of a competitor company. He explained that Astra,
NO Bettsy’s employer, was planning to merge with Zeneca, another drug company; and
- Under the provisions of the Revised NLRC Rules, the decision appealed from in this Bettsy was planning to avail of the redundancy package to be offered by Astra. With
case has become final and executory and can no longer be subject to appeal. Bettsy’s separation from her company, the potential conflict of interest would be
Ratio Administrative regulations and policies enacted by administrative bodies to eliminated. At the same time, they would be able to avail of the attractive redundancy
interpret the law which they are entrusted to enforce, have the force of law, and are package from Astra.
entitled to great respect (Espanol v. Philippine Veterans Administration, 137 SCRA 314 - Tecson again requested for more time resolve the problem. Tecson applied for a
[1985]).. transfer in Glaxo’s milk division, thinking that since Astra did not have a milk division, the
Reasoning potential conflict of interest would be eliminated. His application was denied in view of
- The record shows that REIG received a copy of the decision of the LA on April 1, 1985. Glaxo’s “least-movement-possible” policy. Glaxo transferred Tecson to the Butuan City-
- It filed a Motion for Extension of Time to File Memorandum of Appeal on April 11, 1985 Surigao City-Agusan del Sur sales area. Tecson asked Glaxo to reconsider its decision,
and filed the Memorandum of Appeal on April 22, 1985. but his request was denied.
- Rule VIII of the Revised Rules of the NLRC on appeal, provides that decisions or orders - Tecson sought Glaxo’s reconsideration regarding his transfer and brought the matter to
of a LA shall be final and executory unless appealed to the Commission by any or both of Glaxo’s Grievance Committee. Glaxo, however, remained firm in its decision and gave
the parties within 10 calendar days from receipt of notice and that no motion or request Tecson time to comply with the transfer order. Tecson defied the transfer order and
for extension of the period within which to perfect an appeal shall be entertained. continued acting as medical representative in the Camarines Sur-Camarines Norte sales
NLRC correctly dismissed REIG’s appeal pursuant to said rules. area.
- The NLRC didn’t commit GAD amounting to lack of jurisdiction in arbitrarily dismissing - During the pendency of the grievance proceedings, Tecson was paid his salary, but was
petitioners' appeal on a technicality. not issued samples of products which were competing with similar products
- SC need not interpret the Revised Rules of the NLRC as they are clear and explicit and manufactured by Astra. He was also not included in product conferences regarding such
leave no room for interpretation. products.
- Even on the merits, the ruling of the LA appears to be correct; the consistent - Because the parties failed to resolve the issue at the grievance machinery level, they
promotions in rank and salary of the private respondent indicate he must have been a submitted the matter for voluntary arbitration. Glaxo offered Tecson a separation pay of
highly efficient worker, who should be retained despite occasional lapses in punctuality one-half month pay for every year of service, or a total of P50,000.00 but he declined the
and attendance. Perfection cannot after all be demanded. offer. The National Conciliation and Mediation Board (NCMB) rendered its Decision
Disposition Petition DISMISSED. declaring as valid Glaxo’s policy on relationships between its employees and persons
employed with competitor companies, and affirming Glaxo’s right to transfer Tecson to
another sales territory.
- Aggrieved, Tecson filed a Petition for Review with the Court of Appeals assailing the
NCMB Decision. The Court of Appeals promulgated its Decision denying the Petition for
Review on the ground that the NCMB did not err in rendering its Decision. The appellate
court held that Glaxo’s policy prohibiting its employees from having personal
relationships with employees of competitor companies is a valid exercise of its
management prerogatives. Tecson filed a Motion for Reconsideration of the appellate
court’s Decision, but the motion was denied by the appellate court.
Labor Law 1 A2010 - 44 - Disini
ISSUES SALINAS V NLRC (ATLANTIC GULF AND PACIFIC CO)
1. WON the Court of Appeals erred in ruling that Glaxo’s policy against its employees
319 SCRA 54
marrying employees from competitor companies is valid, and in not holding that said
policy violates the equal protection clause of the Constitution PURISIMA; November 24, 1999
2. WON petitioner was constructively dismissed
FACTS
HELD - Petitioners were employed with Atlantic Gulf and Pacific Co. (AG & P):
1. NO Salinas: 1983-1988 as carpenter/finishing carpenter
- Glaxo has a right to guard its trade secrets, manufacturing formulas, marketing Alejandro: 1982-1989 as bulk cement operator, bulk cement plant/carrier operator &
strategies and other confidential programs and information from competitors, especially crane driver
so that it and Astra are rival companies in the highly competitive pharmaceutical industry. Cortez: 1979-1988 as carpenter/forklift operator
- The prohibition against personal or marital relationships with employees of competitor Samulde: 1982-1989 as lubeman/stationary operator
companies upon Glaxo’s employees is reasonable under the circumstances because - Complaints (separate but consolidated by the LA): illegal dismissal
relationships of that nature might compromise the interests of the company. In laying - Petitioners’ Claim: They had been covered by a number of contracts renewed
down the assailed company policy, Glaxo only aims to protect its interests against the continuously, with periods ranging from five (5) to nine (9) years, and they performed the
possibility that a competitor company will gain access to its secrets and procedures. same kind of work through out their employment, and such was usually necessary and
- That Glaxo possesses the right to protect its economic interests cannot be denied. No desirable in the trade or business of the respondent corporation; and their work did not
less than the Constitution recognizes the right of enterprises to adopt and enforce such a end on a project-to-project basis, although the contrary was made to appear by the
policy to protect its right to reasonable returns on investments and to expansion and employer through the signing of separate employment contracts.
growth. Indeed, while our laws endeavor to give life to the constitutional policy on social - LA: Dismissed petitions on the ground that the petitioners are project employees are
justice and the protection of labor, it does not mean that every labor dispute will be project employees whose work contracts with AG & P indicate that they were employed
decided in favor of the workers. The law also recognizes that management has rights in such category; that they have been assigned to different work projects, not just to one
which are also entitled to respect and enforcement in the interest of fair play. and that their work relation with AG & P, relative to termination, is governed by Policy
- The challenged company policy does not violate the equal protection clause of the Instruction No. 20 (rule governing project employees).
Constitution as petitioners erroneously suggest. It is a settled principle that the - Appeal to NLRC: Affirmed LA’s findings
commands of the equal protection clause are addressed only to the state or those acting
under color of its authority. Corollarily, it has been held in a long array of U.S. Supreme ISSUES
Court decisions that the equal protection clause erects no shield against merely private 1. WON the petitioners are project employees
conduct, however, discriminatory or wrongful. The only exception occurs when the state Procedural
in any of its manifestations or actions has been found to have become entwined or 2. WON this petition for certiorari was proper
involved in the wrongful private conduct. Obviously, however, the exception is not present
in this case. Significantly, the company actually enforced the policy after repeated HELD
requests to the employee to comply with the policy. Indeed, the application of the policy 1. NO
was made in an impartial and even-handed manner, with due regard for the lot of the - The petitioners are regular employees.
employee. - The mandate in Article 281 of the Labor Code, which pertinently prescribes that the
- In any event, from the wordings of the contractual provision and the policy in its 'provisions of written agreement to the contrary notwithstanding and regardless of the
employee handbook, it is clear that Glaxo does not impose an absolute prohibition oral agreements of the parties, an employment shall be deemed to be regular where the
against relationships between its employees and those of competitor companies. Its employee has been engaged to perform activities which are usually necessary or
employees are free to cultivate relationships with and marry persons of their own desirable in the usual business or trade of the employer' and that ‘any employee who has
choosing. What the company merely seeks to avoid is a conflict of interest between the rendered at least one year of service, whether such service is continuous or broken shall
employee and the company that may arise out of such relationships. be considered a regular employee with respect to the activity in which he is employed
- The Court of Appeals also correctly noted that the assailed company policy which forms and his employment shall continue while such actually exists,' should apply in the case of
part of respondent’s Employee Code of Conduct and of its contracts with its employees, petitioner.
such as that signed by Tecson, was made known to him prior to his employment. - Failure to report the termination to Public Employment Office is a clear indication that
Tecson, therefore, was aware of that restriction when he signed his employment contract petitioners were not and are not project employees. (PI No. 20 requires reports of
and when he entered into a relationship with Bettsy. Since Tecson knowingly and terminations)
voluntarily entered into a contract of employment with Glaxo, the stipulations therein - It is basic and irrefragable rule that in carrying out and interpreting the provisions of the
have the force of law between them and, thus, should be complied with in good faith.” He Labor Code and its implementing regulations, the workingman's welfare should be the
is therefore estopped from questioning said policy. primordial and paramount consideration. The interpretation herein made gives meaning
2. NO and substance to the liberal and compassionate spirit of the law enunciated in Article 4 of
- Constructive dismissal is defined as a quitting, an involuntary resignation resorted to Labor Code that "all doubts in the implementation and interpretation of the provisions of
when continued employment becomes impossible, unreasonable, or unlikely; when there the Labor Code including its implementing rules and regulations shall be resolved in
is a demotion in rank or diminution in pay; or when a clear discrimination, insensibility or favor of labor".
disdain by an employer becomes unbearable to the employee. None of these conditions - It is beyond cavil that petitioners had been providing the respondent corporation with
are present in the instant case. The record does not show that Tecson was demoted or continuous and uninterrupted services, except for a day or so gap in their successive
unduly discriminated upon by reason of such transfer. Glaxo properly exercised its employment contracts. Their contracts had been renewed several times, with the total
management prerogative in reassigning Tecson to the Butuan City sales area. When the length of their services ranging from five (5) to nine (9) years. Throughout the duration of
problem could not be resolved after several years of waiting, Glaxo was constrained to their contracts, they had been performing the same kinds of work (e.g., as lubeman, bulk
reassign Tecson to a sales area different from that handled by his wife for Astra. Notably, cement operator and carpenter), which were usually necessary and desirable in the
Glaxo did not terminate Tecson from employment but only reassigned him to another construction business of AG & P, its usual trade or business.
area where his home province, Agusan del Sur, was included. In effecting Tecson’s - Undoubtedly, periods in the present case have been imposed to preclude the
transfer, Glaxo even considered the welfare of Tecson’s family. Clearly, the foregoing acquisition of tenurial security by petitioners, and must be struck down for being contrary
dispels any suspicion of unfairness and bad faith on the part of Glaxo. to public policy, morals, good customs or public order.
2. YES
- Anent the issue that the petition should have been brought under Rule 65 and not under
Rule 45 of the Revised Rules of Court, this rule is not inflexible. In the interest of justice,
often the Court has judiciously treated as special civil actions for certiorari petitions
erroneously captioned as petitions for review on certiorari.
- With regard to the issue on non-exhaustion of administrative remedies, the Court hold
that the failure of petitioners to interpose a motion for reconsideration of the NLRC
decision before coming to this Court was not a fatal omission. The exhaustion of
administrative remedies doctrine is not a hard and fast rule and does not apply where the
issue is purely a legal one. A motion for reconsideration as a prerequisite for the bringing
Labor Law 1 A2010 - 45 - Disini
of an action under Rule 65 may be dispensed with where the issue is purely of law, as in ASIAN TRANSMISSION CORP V CA (BISIG NG ASIAN
this case. At all events and in the interest of substantial justice, especially in cases
TRANSMISSION LABOR UNION)
involving the rights of workers, procedural lapses, if any, may be disregarded to enable
the Court to examine and resolve the conflicting rights and responsibilities of the parties. 425 SCRA 478
This liberality is warranted in the case at bar, especially since it has been shown that the CARPIO-MORALES; March 15, 2004
intervention of the Court is necessary for the protection of the herein petitioner(s).
Disposition The questioned Resolution of the NLRC is SET ASIDE and another one is NATURE
hereby ENTERED ordering the respondent corporation to reinstate petitioners without Petition for certiorari seeking the nullification of the March 28, 2000 Decision of the Court
loss of seniority and with full backwages. of Appeals

ABELLA V NLRC (QUITCO, DIONELE) FACTS


- The Department of Labor and Employment (DOLE), through Undersecretary
152 SCRA 140 Cresenciano B. Trajano, issued an Explanatory Bulletin dated March 11, 1993, wherein it
PARAS; July 20, 1987 clarified, that employees are entitled to 200% of their basic wage on April 9, 1993, which,
apart from being Good Friday, and, therefore, a legal holiday, is also Araw ng Kagitingan,
FACTS which is also a legal holiday, even if unworked.
- Petitioner Rosalina Perez Abella leased a farm land in Ponteverde, Negros Occidental, - Said bulletin was reproduced on January 23, 1998, when April 9, 1998 was both
known as Hacienda Danao-Ramona, for a period of ten (10) years, renewable, at her Maundy Thursday and Araw ng Kagitingan
option, for another ten (10) years. She did renew for another ten years. During the - Despite the explanatory bulletin, petitioner Asian Transmission Corporation opted to pay
existence of the lease, she employed private respondents. Private respondent Ricardo its daily paid employees only 100% of their basic pay on April 9, 1998.
Dionele, Sr. has been a regular farm worker for 33 years while . On the other hand, - Respondent Bisig ng Asian Transmission Labor Union (BATLU) protested.
private respondent Romeo Quitco started worked for 14 years. Upon the expiration of her - In accordance with Step 6 of the grievance procedure of the Collective Bargaining
leasehold rights, petitioner dismissed private respondents and turned over the hacienda Agreement (CBA) existing between petitioner and BATLU, the controversy was submitted
to the owners thereof, who continued the management, cultivation and operation of the for voluntary arbitration.
farm - On July 31, 1998, the Office of the Voluntary Arbitrator rendered a decision directing
petitioner to pay its covered employees "200% and not just 100% of their regular daily
ISSUE wages for the unworked April 9, 1998
WON private respondents are entitled to separation pay - Subject of interpretation in the case at bar is Article 94 3 of the Labor which was
amended by Executive Order No. 2034
HELD - In deciding in favor of the Bisig ng Asian Transmission Labor Union (BATLU), the
YES Voluntary Arbitrator held that Article 94 of the Labor Code provides for holiday pay for
- The closing wasn’t due to serious losses or financial reverses. The Court cited Article every regular holiday, the computation of which is determined by a legal formula which is
284 (this should be 283) which says: not changed by the fact that there are two holidays falling on one day; and that that the
"Art. 284. Closure of establishment and reduction of personnel . — The law, as amended, enumerates ten regular holidays for every year, and should not be
employer may also terminate the employment of any employee due to the installation interpreted as authorizing a reduction to nine the number of paid regular holidays "just
of labor-saving devices, redundancy, retrenchment to prevent losses or the closing or because April 9 (Araw ng Kagitingan) in certain years, like 1993 and 1998, is also Holy
cessation of operation of the establishment or undertaking unless the closing is for the Friday or Maundy Thursday."
purpose of circumventing the provisions of this title, by serving a written notice on the - The Court of Appeals upheld the findings of the Voluntary Arbitrator, holding that the
workers and the Ministry of Labor and Employment at least one (1) month before the Collective Bargaining Agreement (CBA) between petitioner and BATLU, the law
intended date thereof. In case of termination due to the installation of labor-saving governing the relations between them, clearly recognizes their intent to consider Araw ng
devices or redundancy, the worker affected thereby shall be entitled to a separation Kagitingan and Maundy Thursday, on whatever date they may fall in any calendar year,
pay equivalent to at least his one (1) month pay or to at least one (1) month pay for as paid legal holidays during the effectivity of the CBA and that "there is no condition,
every year of service, whichever is higher. In case of retrenchment to prevent losses qualification or exception for any variance from the clear intent that all holidays shall be
and in cases of closure or cessation of operations of establishment or undertaking not compensated.
due to serious business losses or financial reverses, the separation pay shall be - The Court of Appeals further held that "in the absence of an explicit provision in law
equivalent to one (1) month pay or at least one-half (1/2) month pay for every year of which provides for [a] reduction of holiday pay if two holidays happen to fall on the same
service whichever is higher. A fraction of at least six (6) months shall be considered day, any doubt in the interpretation and implementation of the Labor Code
one (1) whole year." provisions on holiday pay must be resolved in favor of labor."
- Petitioner then contends that the aforequoted provision violates the constitutional - Hence, this petition.
guarantee against impairment of obligations and contracts, because when she leased
Hacienda Danao-Ramona, neither she nor the lessor contemplated the creation of the ISSUE
obligation to pay separation pay to workers at the end of the lease. The Court said that WON daily-paid employees are entitled to be paid for two regular holidays which fall on
this contention by petitioner is untenable. The law is clear and to permit such an the same day
argument would mean that the years of service given by the workers will mean nothing
since there is no agreement here that the new management will be the one to shoulder HELD
the separation pay. The old management, pertaining to Abella in this case, should give YES
the payment. - Holiday pay is a legislated benefit enacted as part of the Constitutional imperative that
- In any event, it is well-settled that in the implementation and interpretation of the the State shall afford protection to labor. Its purpose is not merely "to prevent diminution
provisions of the Labor Code and its implementing regulations, the workingman's welfare of the monthly income of the workers on account of work interruptions. In other words,
should be the primordial and paramount consideration. (Volshel Labor Union v. Bureau of although the worker is forced to take a rest, he earns what he should earn, that is, his
Labor Relations, 137 SCRA 43 [1985]). It is the kind of interpretation which gives holiday pay."8 It is also intended to enable the worker to participate in the national
meaning and substance to the liberal and compassionate spirit of the law as provided for celebrations held during the days identified as with great historical and cultural
in Article 4 of the New Labor Code which states that `all doubts in the implementation significance.
and interpretation of the provisions of this Code including its implementing rules and
regulations shall be resolved in favor of labor." The policy is to extend the applicability of 3
ART. 94. Right to holiday pay. (a) Every worker shall be paid his regular daily wage during regular holidays, except
the decree to a greater number of employees who can avail of the benefits under the law, in retail and service establishments regularly employing less than ten (10) workers; (b) The employer may require an
which is in consonance with the avowed policy of the State to give maximum aid and employee to work on any holiday but such employee shall be paid a compensation equivalent to twice his regular rate;
protection to labor. and (c) As used in this Article, "holiday" includes: New Year’s Day, Maundy Thursday, Good Friday, the ninth of April,
the first of May, the twelfth of June, the fourth of July, the thirtieth of November, the twenty-fifth and thirtieth of
Disposition Petition is DISMISSED. December and the day designated by law for holding a general election
4
regular holidays are now:1. New Year’s Day January 1; 2. Maundy Thursday Movable Date; 3. Good Friday Movable
Date; 4. Araw ng Kagitingan April 9 (Bataan and Corregidor Day); 5. Labor Day May 1; 6. Independence Day June 12;
7. National Heroes Day Last Sunday of August; 8. Bonifacio Day November 30; 9. Christmas Day December 25; 10.
Rizal Day December 30
Labor Law 1 A2010 - 46 - Disini
- Independence Day (June 12), Araw ng Kagitingan (April 9), National Heroes Day (last - The major ailments of the deceased could be traced to bacterial and viral infections.
Sunday of August), Bonifacio Day (November 30) and Rizal Day (December 30) were For instance, in the case of leprosy, it is known that the source of infection is discharge
declared national holidays to afford Filipinos with a recurring opportunity to from lesions of persons with active cases.
commemorate the heroism of the Filipino people, promote national identity, and deepen - Petitoner’s husband worked in a skin clinic and was exposed to different carriers of
the spirit of patriotism. Labor Day (May 1) is a day traditionally reserved to celebrate the diseases. As janitor, he was the employee most exposed to the dangerous concentration
contributions of the working class to the development of the nation, while the religious of infected material, and not being a med practitioner, least likely to know how to avoid
holidays designated in Executive Order No. 203 allow the worker to celebrate his faith them.
with his family. - GSIS’s conservative stand is not consistent with the liberal interpretation of the Labor
- As reflected above, Art. 94 of the Labor Code, as amended, affords a worker the Code and the social justice guarantee embodied in the Constitution in favor of workers.
enjoyment of ten paid regular holidays. The provision is mandatory, regardless of Disposition Decision appealed from is set aside and GSIS is ordered to pay petitioner
whether an employee is paid on a monthly or daily basis. Unlike a bonus, which is a P12T as death benefits and P1,200 as attorney’s fees.
management prerogative, holiday pay is a statutory benefit demandable under the law.
Since a worker is entitled to the enjoyment of ten paid regular holidays, the fact
ACUÑAV CA
that two holidays fall on the same date should not operate to reduce to nine the
ten holiday pay benefits a worker is entitled to receive. [PAGE 12]
- It is elementary, under the rules of statutory construction, that when the language of
the law is clear and unequivocal, the law must be taken to mean exactly what it BONIFACIO V GSIS
says. In the case at bar, there is nothing in the law which provides or indicates
146 SCRA 276
that the entitlement to ten days of holiday pay shall be reduced to nine when
two holidays fall on the same day. FERNAN; December 15, 1986
- In any event, Art. 4 of the Labor Code provides that all doubts in the
implementation and interpretation of its provisions, including its implementing NATURE
rules and regulations, shall be resolved in favor of labor. For the working man’s Petition for review on certiorari
welfare should be the primordial and paramount consideration.
- Moreover, Sec. 11, Rule IV, Book III of the Omnibus Rules to Implement the Labor Code FACTS
provides that "Nothing in the law or the rules shall justify an employer in withdrawing or - Lourdes Bonifacio was a classroom teacher assigned to the district of Bagamanoc,
reducing any benefits, supplements or payments for unworked regular holidays as Division of Catanduanes, from August 1965 until she contracted carcinoma of the breast
provided in existing individual or collective agreement or employer practice or policy. with metastases to the gastrointestinal tract and lungs which caused her death on Oct. 5,
- From the pertinent provisions of the CBA entered into by the parties, petitioner had 1978.
obligated itself to pay for the legal holidays as required by law. - Thereafter a claim for death benefits under P.D. No. 626, as amended, was filed by
Disposition Petition is dismissed. petitioner with the GSIS. The same was however denied on the ground that the
decedent's principal ailment, carcinoma of the breast with metastases to gastrointestinal
tract and lungs, is not an occupational disease for her particular work as a teacher, nor is
CLEMENTE V GSIS the risk of contracting said disease increased by her working conditions.
152 SCRA 500 - The Employees Compensation Commission (ECC), on appeal affirmed the decision of
GUTIERREZ, JR; July 31, 1987 the GSIS.

NATURE ISSUES
Petition to review decision of the Employees’ Compensation Commission (ECC) which 1. WON the GSIS and the ECC erred in denying petitioner’s claim
affirmed decision of GSIS and denied Clemente’s claim for death benefits 2. WON the rule that “in case of doubt in the implementation and interpretation of the
provisions of the Labor Code, including its implementing rules and regulations, the same
FACTS shall be resolved in favor of the laborer” applies in this case
- Carolina’s husband, Pedro Clemente was for 10 years a janitor in the DOH Dagupan
City assigned at the Ilocos Norte Skin Clinic. HELD
- He was hospitalized for 12 days due to his ailment of nephritis, and was also found to 1. NO
be suffering from Hansen’s Disease (portal cirrhosis and leprosy). He died on Nov 14, - A compensable sickness means "any illness definitely accepted as an occupational
1976. disease listed by the ECC, or any illness caused by employment subject to proof by the
- Petitioner then filed with GSIS a claim for employees’ compensation under the Labor employee that the risk of contracting the same is increased by working conditions. For
Code. This was denied by GSIS on the ground that such ailments are not occupational this purpose, the Commission is empowered to determine and approve occupational
diseases taking into consideration nature of his work. Under Art. 167(L) of the Labor diseases and work-related illnesses that may be considered compensable based on
Code and Sec. 1(b) Rule III of the Amended Rules on Employees’ Compensation, for the peculiar hazards of employment." [Art. 167(1) Labor Code as amended by P.D. No. 1368,
sickness and the resulting disability or death to be compensable, sickness must be the effective May 1, 1978].
result of an occupational disease listed under Annex A of the rules; otherwise proof must - Thus, for the sickness or the resulting disability or death to be compensable, the
be shown that the risk of contracting the disease is increased by the working conditions. sickness must be the result of an accepted occupational disease listed by the ECC, or
- Petitioner claimed that the ailments were contracted in the course of employment and any other sickness caused by employment subject to proof by claimant that the risk of
were aggravated by his work since he was in direct contact with persons suffering from contracting the same is increased by working conditions. [Sec. 1, Rule 11, Amended
different skin diseases and was exposed to obnoxious dusts and other dirt. Rules on Employees Compensation]. Carcinoma of the breast with metastases to the
- ECC also dismissed the claim since there was no substantial evidence of causal gastrointestinal tract and lungs is not listed by the Commission as an occupational
connection and there was evidence that deceased had already contracted the Hansen’s disease.
before employment. - The cancer which affected the deceased not being occupational in her particular
employment, it became incumbent upon petitioner to prove that the decedent's working
ISSUE conditions increased the risk of her contracting the fatal illness. This onus petitioner failed
WON petitioner is entitled to the compensation to satisfactorily discharge.
- Petitioner's contention that the decision of the ECC totally ignored the SC's
HELD pronouncements on compensation cases is unmeritorious. The petitioner evidently
YES overlooked that his claim is now within the ambit of the Labor Code and the rulings under
Ratio Strict rules of evidence are NOT applicable in claims for compensation. The the old law, Act No. 3428, as amended, no longer control.
degree of proof required is merely substantial evidence, which means ‘such relevant - The old law as embodied particularly in Section 43 of RA No. 772 amending Act No.
evidence as a reasonable mind might accept as adequate to support a conclusion.’ What 3812, provided for "the presumption of compensability and the rule on aggravation of
the law requires is a reasonable work-connection and not a direct causal relation. illness, which favor the employee," and "paved the way for the latitudinarian or expansive
- Doubts should be resolved in favor of the claimant-employee. application of the Workmen's Compensation Law in favor of the employee or worker."
The presumption in essence states that in any proceeding for the enforcement of the
Reasoning claim for compensation under the Workmen's Compensation Act "it shall be presumed in
Labor Law 1 A2010 - 47 - Disini
the absence of substantial evidence to the contrary that the claim comes within the > while the causes of colonic malignancy are as yet undetermined, there is a
provisions of the said Act, that sufficient notice thereof was given, that the injury was not "probability" that the fatal ailment of Bravo was work connected as shown by the fact that
occasioned by the willful intention of the injured employee to bring about the injury or he was exposed to various chemicals which are generally considered predisposing
death of himself or of another, that the injury did not result solely from the intoxicatiojn of factors of cancer (relying on the decision in Panotes vs. Employees' Compensation
the injured employee while on duty, and that the contents of verified medical and surgical Commission where it was held that the very fact that the cause of a disease is unknown
reports introduced in evidence by claimants for compensation are correct." creates the probability that the working conditions could have increased the risk of
- Thus, under the Workmen's Compensation Law, it is not necessary for the claimant to contracting the disease, if not caused by it) ; that the law merely requires reasonable
carry the burden of proof to establish his case to the point of demonstration It is not work-connection because of the liberal interpretation accorded to social legislation; that
necessary to prove that employment was the sole cause of the death or injury suffered under the theory of increased risk, her husband’s cancer of the colon is a compensable
by the employee. It is sufficient to show that the employment had contributed to the disease because his exposure to chemicals and the "stressful demand" of his work
aggravation or acceleration of such death or ailment. Once the disease had been shown increased the risk of contracting said ailment; and that Commission issued the
to have arisen in the course of employment, it is presumed by law, in the absence of Resolutions Nos. 2610 and 2677 5 which provides guidelines for deciding on pending
substantial evidence to the contrary, that it arose out of it. compensation cases regarding cancer.
- With this legal presumption in the old law, the burden of proof shifts to the employer - Solicitor General’s reply
and the employee no longer suffers the burden of showing causation. Under the present > resolutions are just proofs that the Commission is continuously in involved in its task
Labor Code, the "latitudinarian or expansive application of the Workmen's Compensation "to initiate, rationalize, and coordinate policies of the employees' compensation program."
Law in favor of the employee or worker" no longer prevails as the burden of showing They do not imply that the law merely requires reasonable work-connection because that
proof of causation has shifted back to the employee particularly in cases of sickness or requirement which was mandated in the repealed Workmen's Compensation Act is
injuries which are not accepted or listed as occupational by the ECC. The Labor Code different from the present requirement of clear medical basis "where before a mere
abolished the presumption of compensability and the rule on aggravation of illness aggravation or presumption of compensability was sufficient."
caused by the nature of the employment.
2. NO ISSUES
- While the court does not dispute petitioner's contention that under the law, in case of WON cancer of the colon and peri-appendicitis which caused the death of a former litho-
doubt in the implementation and interpretation of the provisions of the Labor Code, photo engraving supervisor are compensable diseases under the Labor Code
including its implementing rules and regulations, the doubt shall be resolved in favor of a. WON cancer of the colon and peri-appendicitis are listed under compensable diseases
the laborer, the court finds that the same has no application in this case since the under the Labor Code and Rule III, Section IV of the Amended Rules on Employees’
pertinent provisions of the Labor Code leave no room for doubt either in their Compensation
interpretation or application. b. WON petitioner could claim benefits through the increased risk doctrine
Disposition Petition is dismissed and the decisions of the GSIS and the ECC denying
the claim are affirmed HELD
NO
Ratio Article 167, paragraph (1) of the Labor Code and Rule III, Section IN of the
BRAVO V EMPLOYEES' COMPENSATION Amended Rules on Employees' Compensation provide that for a sickness and the
COMMISSION resulting disability or death to be compensable, the said sickness must be an
143 SCRA 101 occupational disease listed under Annex "A" of said Rules, otherwise, the claimant or
FERNAN; July 22, 1986 employee concerned must prove that the risk of contracting the disease is increased by
the working conditions (increased risk doctrine)
a. NO
NATURE
- Both cancer of the colon and peri-appendicitis are not listed as occupational diseases
PETITION for certiorari to review the decision of the Employees' Compensation
for Bravo's kind of employment.
Commission
b. NO
- Petitioner failed to submit convincing proofs to entitle her to compensation benefits.
FACTS
Ratio A claimant who depends on the theory of increased risk must present substantial
- Evelio Bravo was a supervising cartographer engineer at the Bureau of Coast and
proof to show that his ailment was contracted during his employment. He or she must
Geodetic Survey. As litho-photo engraving supervisor (another term for a supervising
also submit proof that the risk of contracting the ailment was increased by the particular
cartographer engineer?), he was involved in drafting and plate printing, developing and
working conditions.
processing either dry or wet negatives, and supervising the formulation of lightsensitive
Reasoning
lithographic chemicals from reagent of nitric, phosphoric, oleic acids, potassium
- On reliance on Panotes case: In the Panotes case and the Cristobal case, both
ferricynamide, ammonium hydroxide and ammonium dichromate in the kithographic
claimants presented conditions of their employment. In the present case, the petitioner
laboratory.
only enumerated the chemicals to which Bravo was allegedly exposed as a
- sometime in 1979, he complained of irregular bowel movement, constipation and
litho-photo engraving supervisor and rely on the "probability" that those
abdominal pain. In 1980 he was admitted to St. Luke’s Hospital and was diagnosed with
chemicals caused his cancer of the colon.
"adenocarcinoma sigmoid (colon) Duke's C and chronic periappendicitis". He
On interpretation in compensation cases
went through a series of operations and incurred hospitalization expenses amounting to
> Strict rules of evidence are not applied in compensation cases . However, the present
P8,650.05.
scheme and theory of employees' compensation under the Labor Code requires a clear
- He did not return to work and retired at the age of 45 under the provisions of RA 1616.
medical basis for a claim for benefits to succeed. There are no more presumptions as to
He received P37,002.31 from GSIS. He filed a claim for disability benefits in the GSIS.
what caused a particular illness because the determination of compensability is medically
- GSIS: Denied. His diagnosed disease were not occupational diseases in his particular
and scientifically oriented.
employment and his working conditions did not increase the risk of contracting them.
-He sought reconsideration, claiming that his work exposed him to chemicals. His MFR
was denied on the ground that his exposure to photographic solutions as litho-
photo engraving supervisor had no causal relationship to the development of
his adenocarcinoma considering that said ailment is traceable to "familial
multiple polyposis, chronic ulcerative colitis, chronic lympho-granuloma 5
Resolution No. 2610 approves the recommendation of the Commission's Technical Committee on Medical Matters
venereum, chronic granuloma inguinale and perhaps adenoma”. that appealed compensation cases "whose subject contingencies concern cancer diseases shall be held compensable,
- He appealed to the Employees’ Compensation Commission, but died pending the in line with pertinent Supreme Court Decisions, provided that such diseases shall be duly confirmed by formal reports
appeal. His widow, Angeles, pursued his appeal. on biopsies, or opinions of cancer specialists". That resolution shall be applied prospectively.
- Commission: affirmed GSIS deci. Bravo's ailments were "too remote to be
related causally to his work and working conditions" at the Bureau of Coast and Resolution No. 2677 amends Resolution No. 2610 by adding to the pertinent paragraph thereof the phrase "provided
Geodetic Survey. His contention that his cancer could be traced to exposure to that certain predisposing factors that are medically recognized or proven are present." It also approves the modified
guidelines on cancer of the breast, liver stomach (gastric), lungs and nasopharynx. As regards "other types of cancer
photographic solutions was merely supposition and devoid of medical support. diseases", the guideline states: "An employee's prolonged exposure to chemicals may predispose him or her to
contract and develop other types of cancer diseases". For cancer cases decided by the Supreme Court, the guidelines
states: "A claim must be resolved in favor of a claimant or appellant if facts of his or her case on record indicate
reasonable work-connection of the disease, the disease belongs to borderline or 'twilight' cases, and if the cause of
- Petitioner’s contention the cancer disease is unknown".
Labor Law 1 A2010 - 48 - Disini
On application of the resolutions by the Commission MANNING INTERNATIONAL CORPORATION V NLRC
> they were issued after the death of Evelio, and are applied prospectively. Even if they
(BENEDICTO)
were applied, the petitioner did not submit formal requirements required by said
resolutions. 195 SCRA 155
On liberal interpretation due to social legislation NARVASA; March 13, 1991
> “We are aware of the mandate that social legislation should be applied in consonance
with the principles of social justice and protection to labor. However, we cannot adopt a NATURE
sweeping interpretation of the law in favor of labor lest we engage in judicial legislation.” Petition for certiorari to review
Disposition decision of the Employees' Compensation Commission is hereby affirmed.
FACTS
PAL V NLRC (IRINEO) - Francisco Benedicto—a.k.a. Lazaro Benedicto, according to his passport—was hired by
a foreign firm, Abdulasis & Mohamed A. Aljomaih Co., thru its Philippine representative.
201 SRCA 687 Manning International Corporation, as a truck driver for its establishment in Riyadh,
NARVASA; September 24, 1991 Saudi Arabia. Benedicto was engaged for a stipulated term of two (2) years. He left for
Saudi Arabia on December 1, 1980 to fulfil his employment contract.
FACTS - Some months before the expiration of his contract with Abdulasis, etc., Benedicto was
- On the basis of the findings and recommendations of a Fact Finding Panel upon involved in a vehicular accident, was injured, and in consequence, lost both his legs.
investigation, Irineo and 3 other PAL employees, Damian, Rabasco and Macatol, were From the date of the accident, February 2, 1982, he was confined at a hospital in Saudi
prosecuted and convicted for estafa through falsification of commercial documents Arabia until sometime when his employment was terminated. He was repatriated to the
- All 4 filed motions for reconsideration and/or new trial. Only one of them, Macatol, was Philippines in August 1982.
absolved “for lack of sufficient evidence”. 12 years later, Macatol filed a complaint for - Benedicto filed a complaint with POEA for the recovery of his salary for the unexpired
illegal dismissal which was dismissed by the Labor Arbiter on the ground of prescription. portion of his contract, insurance benefits and projected cost of medical expenses
The NLRC affirmed, contending that the prescriptive period accrued from the time of his amounting to P25,000.00
dismissal and not the termination of the criminal case - POEA dismissed Benedicto’s claim upon finding that “complainant was legally
- A later appeal with the IAC resulted in the acquittal of Irineo and Rabasco on grounds of terminated from employment” because of his disability. However, Manning and Abdulasis
reasonable doubt. were ordered to provide compensation benefits for service-connected illness, injuries or
- 17 years after his dismissal, Irineo filed a complaint against PAL for reinstatement and death.
backwages, claiming the termination was illegal. The Labor Arbiter’s decision decreed his - MFR to NLRC – affirmed the decision of POEA
reinstatement without loss of seniority rights, payment of backwages and moral damages - Judgment became final and executory. Benedicto moved for computation of the
of P300k. The Arbiter overruled the defense of prescription and held that since there was amounts due him, and in substantiation, submitted receipts evidencing his actual medical
a PAL circular which placed any employee charged with any crime inimical to the expenses. His former employers opposed the motion on the ground that the medical
company’s interest under preventive suspension, and a standing order by the CIR expenses referred to another person, Lazaro Benedicto but the Administrator overruled
forbidding the dismissal of any PAL employee without court authority, Irineo’s dismissal the objection and pointed out that the names Lazaro and Francisco Benedicto both
merely amounted to suspension. The Arbiter rendered a judgment terminating Irineo’s referred to one person, and directed the issuance of an alias writ of execution to enforce
suspension with backwages and moral damages. payment of P12,000 as total and permanent disability benefits and P19,450.00 as
- PAL appealed to the NLRC but failed to obtain a reversal of the Arbiter’s decision. hospitalization and medical expenses for 120 days or a total of P31,450.00.
Hence this petition for certiorari. - MFR was filed to NLRC to protest the limitation of the award to him of medical
expenses to a period of 120 days.
ISSUE - NLRC – set aside the the POEA Order and on considerations of equity and social
WON the NLRC gravely abused discretion amounting to lack or excess of jurisdiction and justice as well as the theory of “medical treatment should not be stopped until
arbitrarily exercised power without due regard for the rule of law Benedicto’s injury or disability is healed” and entered a new judgment increasing the
amount to be paid by employers.
HELD - Petition for certiorari
YES
- That there should be care and solicitude in the protection and vindication of the rights of ISSUES
workingmen cannot be gainsaid; but that care and solicitude cannot justify disregard of 1. WON the “new judgment” of the NLRC is void ab initio, insofar as it attempts to vary
relevant facts or eschewal of rationality in the construction of the text of applicable rules the disposition of the final and executory decision of the POE Administrator
in order to arrive at a disposition in favor of an employee who is perceived as otherwise 2. WON the challenged decision of NLRC is without legal basis and unjust
deserving of sympathy and commiseration.
- The letter to Irineo from then PAL president Benigno Toda clearly indicated, “For being HELD
involved in the irregular refund of tickets in the international service to the damage and 1. YES
prejudice of the company, you are dismissed from the service effective immediately.” For Ratio When a final judgment becomes executory, it thereby becomes immutable and
the Arbiter and the NLRC to construe this as mere suspension would be illogical. unalterable, The judgment may no longer be modified in any respect, even if the
-Their reliance on PAL circular to justify their decision, construing this as a complete modification is meant to correct what is perceived to be an erroneous conclusion of fact
foreclosure of any alternative action on PAL’s part was unfounded. To further support or law, and regardless of whether the modifications is attempted to be made by the Court
their decision they refer said CIR standing order which had been imposed in relation to a rendering it or by the highest Court of the land. The only recognized exceptions are the
pending labor dispute with the CIR. However, having ended when the parties entered into correction of clerical errors or the making of so-called nunc pro tunc entries which cause
a CBA 2 years before Irineo’s dismissal, the standing order was no longer relevant to the no prejudice to any party, and, of course, where the judgment is void.
event. 2. YES
- Irineo’s assertion only after 17 years meant he slept on his rights—his claim is thus Ratio Considerations of equity and social justice” cannot prevail over against the
time-barred. expressed provision of the labor laws allowing dismissal of employees for cause and
- Premises considered, it appears that the NLRC’s conclusions are flawed by errors without any provision for separation pay.
serious as to constitute grave abuse of discretion Disposition Contested Decision ANNULLED AND SET ASIDE and REINSTATING and
Disposition Court GRANTS the petition and issues the writ of certiorari prayed for. AFFIRMING the Order of the POE Administrator.
Labor Law 1 A2010 - 49 - Disini
RELIANCE & INSURANCE CO INC V NLRC (RELIANCE - The strike that was illegal in more ways than one, the reinstated union officers were
clearly in bad faith, and to reinstate them without loss of seniority rights, is to reward
SURETY & INSURANCE EMPLOYEES UNION) them for an act public policy does not sanction.
193 SCRA 365 - The Ferrer and Almira cases did not involve illegal strikes. In Ferrer was a defective
SARMIENTO; January 25 1991 strike, one conducted in violation of the thirty-day "cooling-off" period, but one carried out
in good faith "to offset what petitioners were warranted in believing in good faith to be
NATURE unfair labor practices [committed by] Management. What Almira on the other hand
Petition for certiorari whether or not strikers have been found to have staged an illegal declared was that a violent strike alone does not make the action illegal, which would
strike may be reinstated to work. justify the dismissal of strikers.
- The Court reiterates that good faith is still a valid defense against claims of illegality of a
FACTS strike. We do find, however, not a semblance of good faith here, but rather, plain
- The manager of Reliance Surety Insurance Co (RSIC) effected a change in the seating arrogance, pride, and cynicism of certain workers.
arrangement of its personnel to avoid productivity loss due to personal and non-work- - WRT respondent, Isagani Rubio, what militates against his readmission to the firm is
related conversations, calls and visits. the fact that he had accepted the sum of P2,448.80 "in full satisfaction of the . . .
- Isagani Rubio, Rosalinda Macapagal, Glene Molina, and Severa Cansino protested the Decision" (of the Labor Arbiter).
transfer of their tables because said change was without prior notice and was just to - The sympathy of the Court is on the side of the laboring classes, not only because the
harass them as union members. When the manager insisted, a heated discussion Constitution imposes sympathy but because of the one-sided relation between labor and
ensued, during which Rubio and companions insulted the manager and supervisors. capital. The Court must take care, however, that in the contest between labor and capital,
- The 4 employees were asked to explain within 48 hours why no disciplinary action the results achieved are fair and in conformity with the rules. We will not accomplish that
should be taken against them for misconduct, insubordination, and gross disrespect. objective here by approving the act of the National Labor Relations Commission which
Tension rose in the office as Rubio continued to refuse to stay at his designated place, we hold to constitute a grave abuse of discretion.
and Molina and Macapagal still levelled insults to those who testified against them. Disposition petition is GRANTED.
Hence, Rubio and companions were placed under preventive suspension on 3 February
1987 and ultimately dismissed after investigation on 3 March 1987. PHILIPPINE AIRLINES, INC. (PAL) V NLRC
- 6 March 1987, the Reliance Surety & Insurance Employees Union (union) filed in behalf
[PAGE 25]
of the dismissed employees with the NLRC, against the RSIC a complaint for illegal
dismissal including the charge of unfair labor practice.
- Union’s claim: The company was guilty of unfair labor practice because it effected DUNCAN ASSOCIATION V GLAXO WELLCOME PHILS
transfer and changes in the seating arrangement to pressure or intimidate union [PAGE 43]
members; because it interfered in the union members' exercise of their right to self-
organization by forcing them to undertake overtime work even on a non-working
Saturday and in times when there were scheduled union meetings to prevent them from UNITED PEPSI-COLA SUPERVISORY UNION (UPSU) V
attending the same; and because, thru its manager and assistant managers, it caused LAGUESMA
the resignation and withdrawal of union members from the union. 288 SCRA 15
- Pending trial, the union filed with the DOLE a notice of strike predicated on unfair labor
MENDOZA; March 25, 1998
practices by the company. RSIC was given notice of strike and a telegram from DOLE for
initial conciliation conference both to be held on the same date. But even before the initial
conference could take place, the union in the morning of 17 March 1987 struck and FACTS
picketed the company premises, which obstructed the free ingress to and egress from its - UPSU is a union of supervisory employees. The union filed a petition for certification
premises, thereby preventing its officials and employees from doing their usual duties. election on behalf of the route managers at Pepsi-Cola Products Philippines, Inc.
- RSIC them filed with the NLRC a petition to declare the strike illegal due to the defiance However, its petition was denied by the med-arbiter and, on appeal, by the Secretary of
of the 30 or 15 day cooling-off-period, disregard of the legal requirement to furnish the Labor and Employment, on the ground that the route managers are managerial
department with the results of the strike vote at least 7 days before the strike and failure employees and, therefore, ineligible for union membership under the first sentence of Art.
to furnish a written notice of the meeting to declare a strike to the BLR or the Regional 245 of the Labor Code, which says, “Managerial employees are not eligible to join, assist
Office or form any labor organization. Supervisory employees shall not be eligible for
- Labor Arbiter found the strike to be illegal. NLRC affirmed with modification upon appeal membership in a labor organization of the rank-and-file employees but may join, assist or
holding that although the strike was illegal, dismissal was not the proper penalty. It said form separate labor organizations of their own.”
that the strikers should be reinstated without backwages due to the union’s belief that the
company was committing unfair labor practice. (Ferrer v. Court of Industrial Relations 6 ISSUE
and Almira v. BF Goodrich Philippines, Inc) 1. WON the route managers at Pepsi-Cola Products Philippines, Inc. are managerial
- Petitioner’s claim : NLRC was guilty of grave abuse of discretion. employees
2. WON the first sentence of Art. 245 of the Labor Code, prohibiting managerial
ISSUE employees from forming, assisting or joining any labor organization, is constitutional in
1. WON the strike was illegal light of Art. III, Sec.8 of the Constitution. “The right of the people, including those
2. WON the petition should be granted employed in the public and private sectors, to form unions, associations, or societies for
purposes not contrary to law shall not be abridged.”
HELD
1. YES HELD
- The strike in question was illegal, for failure of the striking personnel to observe legal 1. YES
strike requirements, to wit: (1) as to the fifteen-day notice; (2) as to the two-thirds - Their job descriptions clearly reveal so. They also fall under this category under the
required vote to strike done by secret ballot; (3) as to submission of the strike vote to the purview of art. 212. The term “manager” generally refers to “anyone who is responsible
Department of Labor at least seven days prior to the strike. for subordinates and other organization resources.” As a class, managers constitute
- NLRC also found that certain strikers harassed non-striking employees, called company three levels of a pyramid.
officers names, and committed acts of violence (as a result of which, criminal charges - What distinguishes them from the rank-and file employees is that they act in the interest
were brought with the fiscal's office.) of the employer in supervising such rank-and-file employees
- The strike itself was prompted by no actual, existing unfair labor practice committed by - Managerial employees” may therefore be said to fall into two distinct categories: the
the petitioner. In effecting a change in the seating arrangement, the petitioner merely “managers” per se, who compose the former group described above, and the
exercised a reasonable prerogative employees could not validly question, much less “supervisors” who form the latter group. Whether they belong to the first or second
assail as an act of unfair labor practice. Rearranging furniture cannot justify a four-month- category, managers, vis-à-vis employers, are, likewise, employees
long strike. As to the private respondent's charges of harassment, the Commission found 2. NO
none, and as a general rule, the Court is bound by its findings of fact. - As already stated, whether they belong to the first category (managers per se) or the
2. YES second category (supervisors), managers are employees. Nonetheless, in the United
States, as Justice Puno's separate opinion notes, supervisors have no right to form
Labor Law 1 A2010 - 50 - Disini
unions. They are excluded from the definition of the term "employee" in §2(3) of the VILLAVILLA V CA (SSS, MERCADO, COSUCO)
Labor-Management Relations Act of 1947.
212 SCRA 488
- Commission intended the absolute right to organize of government workers,
supervisory employees, and security guards to be constitutionally guaranteed. By BELLOSILLO; August 11, 1992
implication, no similar absolute constitutional right to organize for labor purposes should
be deemed to have been granted to top-level and middle managers. As to them the right FACTS
of self-organization may be regulated and even abridged conformably to Art. III, §8. - Arturo Villavilla, son of petitioners, was employed as "tripulante" (crew member) of the
- Types of Managerial Employees: fishing boat "F/B Saint Theresa" from 1974 until September 11, 1977, when the boat
> FIRST-LINE MANAGERS – The lowest level in an organization at which individuals are sank off Isla Binatikan, Taytay, Palawan. Arturo was not among the known survivors of
responsible for the work of others is called first-line or first-level management. First-line that sinking and had been missing since then.
managers direct operating employees only; they do not supervise other managers. - On November 20, 1979, petitioners Andres Villavilla and Ester Gadiente Villavilla,
Example of first-line managers are the “foreman” or production supervisor in a parents of Arturo, filed a petition with the Social Security Commission against Reynaldo
manufacturing plant, the technical supervisor in a research department, and the clerical Mercado and Marcelino Cosuco, owners of the ill-fated fishing boat, for death
supervisor in a large office. First-level managers are often called supervisors. compensation benefits of Arturo whom respondents failed to register as their employee.
> MIDDLE MANAGERS – The term middle management can refer to more than one level - On May 29, 1981, the Social Security System (SSS) filed a petition in intervention
in an organization. Middle managers direct the activities of other managers and alleging that records from the SSS Production Department showed that "F/B Saint
sometimes also those of operating employees. Middle managers’ principal Theresa", owned by Marcelino Cosuco and operated by Reynaldo Mercado, was a
responsibilities are to direct the activities that implement their organizations’ policies and registered member-employer, and that in the event petitioners succeeded in proving the
to balance the demands of their superiors with the capacities of their subordinates. A employment of Arturo with private respondents, the latter should be held liable in
plant manager in an electronics firm is an example of a middle manager. damages equivalent to the benefits due the petitioners for failure to report Arturo for
> TOP MANAGERS – Composed of a comparatively small group of executives, top coverage pursuant to Sec. 24 (a) of the Social Security Act, as amended.
management is responsible for the overall management of the organization. It - Respondent Cosuco filed his answer denying all allegations in the petition and claiming
establishes operating policies and guides the organization’s interactions with its that he already sold the fishing boat to respondent Mercado on December 10, 1975, and
environment. Typical titles of top managers are “chief executive officer,” “president,” and from then on he did not participate anymore in the operation and management of the
“senior vice-president.” Actual titles vary from one organization to another and are not boat nor in the hiring of its crewmembers.
always a reliable guide to membership in the highest management classification. - Meanwhile respondent Mercado was declared in default for failure to file his answer.
Disposition petition is DISMISSED - After petitioners had presented their evidence and rested their case, respondent
Cosuco filed a motion to dismiss (demurrer to evidence) on the ground of res judicata
and lack of cause of action.
SONZA V ABS-CBN BROADCASTING CORPORATION - Respondent Social Security Commission issued an Order dismissing the petition for
[PAGE 42] lack of cause of action.
- The Court of Appeals affirmed the questioned Order of respondent Commission there
ASIATIC DEVELOPMENT CORP V BROGADA being no reversible error.
495 SCRA 166 ISSUE
CORONA; July 14, 2006 1. WON there was an employer-employee relationship between petitioners' deceased
son, Arturo Villavilla, and herein private respondents
NATURE 2. WON private respondents are liable for death compensation benefits of Arturo Villavilla
Petition for review on certiorari 3. WON there was a violation of the Social Security Act, as amended, by private
respondents for not registering Arturo Villavilla with the System as their employee as
FACTS mandated by law
- Respondents Wellington and Flordeliza Brogada are the parents of Fermin B. Brogada
who was allegedly employed by petitioner Asiatic Development Corporation from July HELD
1994 up to his death in November 14, 1996. 1. NO
- Respondents filed with the SSC a petition for social security coverage and payment of - The records disclose that the relationship between Mercado and the crew members of
contributions in order to avail of the benefits accruing from the death of Fermin. They the ship headed by its skipper, Capt. Pedro Matibag, is one positively showing the
alleged that Fermin worked as survey aide under Engr. Bienvenido Orense, petitioner’s existence of a joint venture. This is clearly revealed in the testimonies of Capt. Pedro
geodetic engineer. Fermin was working on a project with Engr. Orense for one of Matibag and Gil Chua, a crew member, both witnesses for petitioners.
petitioner’s clients when he was shot and killed. - The arrangement between the boat owner and the crew members, one of whom was
- Petitioner denied its liability on the ground that there was no employer-employee petitioners' son, partook of the nature of a joint venture: the crew members did not
relationship between it and Fermin. It claimed that Fermin was the employee of Engr. receive fixed compensation as they only shared in their catch; they ventured to the sea
Orense. irrespective of the instructions of the boat owners, i.e., upon their own best judgment as
- SSS held that Fermin was an employee and was subject to the compulsory coverage. to when, how long, and where to go fishing; the boat owners did not hire them but simply
On appeal, the SSC resolution was affirmed by the CA. joined the fishing expedition upon invitation of the ship master, even without the
knowledge of the boat owner. In short, there was neither right of control nor actual
ISSUE exercise of such right on the part of the boat owner over his crew members.
WON an employer-employee relationship exists - It is clear that there was no employer-employee relationship between petitioner's son
Arturo and private respondent Mercado, much less private respondent Cosuco. As such,
HELD Arturo could not be made subject of compulsory coverage under the Social Security Act;
- The question of WON an employer-employee relationship exists is a question of fact. In hence, private respondents cannot be said to have violated said law when they did not
petitions for review on certiorari under Rule 45, only questions of law may be raised by register him with the Social Security System. A fortiori, respondent as well as intervenor
the parties and passed upon by this Court. Factual findings of quasi-judicial bodies, are not answerable to petitioners for any death benefits under the law.
when adopted and confirmed by the CA and if supported by substantial evidence, are - Culled from the foregoing, the inexorable conclusion is that respondent Court of
accorded respect and even finality by this Court. While this Court has recognized Appeals did not err in sustaining the judgment of respondent Social Security
several exceptions to this rule, none of these exceptions finds application here. Commission.
- Both the SSC and CA found that Fermin was petitioner’s employee. Thus, petitioner is - It may not be amiss to mention that while petitioners merely raise factual questions
liable for unpaid social security contributions. which are not proper under Rule 45 of the Rules of Court, We nevertheless went to great
- Petitioner’s claims are a mere reiteration of arguments unsuccessfully raised before the lengths in dissecting the facts of this case if only to convince Us that petitioners, who are
SSC and the CA. No compelling reason whatsoever is shown by petitioner for this Court pauper litigants and seeking claims under a social legislation, have not been denied its
to reverse the SSC’s findings and conclusions, as affirmed by the CA. benefits. For, We are not unaware that in this jurisdiction all doubts in the implementation
Disposition Petition is DENIED. and interpretation of provisions of social legislations should be resolved in favor of the
working class. But, alas, justice is not fully served by sustaining the contention of the
Labor Law 1 A2010 - 51 - Disini
poor simply because he is poor. Justice is done by properly applying the law regardless - NLRC: silent on JCT being the employer of Miguel et al after Glorious Sun ceased
of the station in life of the contending parties. operations, save for its conclusion that they ‘were absorbed by, or their work continued
under JCT’ and did not state the reason for liability in solidum of Cuevas. Computation of
the monetary award totaling P37,557,317.08 (exclusive of attorney’s fees) covers a
MIGUEL V JCT GROUP INC period starting on initial employment (with Glorious Sun) some dating back to 1978
453 SCRA 529 - Saballa v NLRC > This Court has previously held that judges and arbiters should draw
PANGANIBAN; March 16, 2005 up their decisions and resolutions with due care, and make certain that they truly and
accurately reflect their conclusions and their final dispositions. The same thing goes for
NATURE the findings of fact made by the NLRC, as it is a settled rule that such findings are
Petition for Review on Certiorari under Rule 45 of the Rules of Court CA decision on the entitled to great respect and even finality when supported by substantial evidence;
ground of grave abuse of discretion because it annulled and set aside decisions of the otherwise, they shall be struck down for being whimsical and capricious and arrived at
labor arbiter and NLRC with grave abuse of discretion. It is a requirement of due process and fair play that the
parties to a litigation be informed of how it was decided, with an explanation of the factual
FACTS and legal reasons that led to the conclusions of the court. A decision that does not
- 1984 > Glorious Sun Garment Manufacturing Company was a garment exporter until it clearly and distinctly state the facts and the law on which it is based leaves the parties in
folded up and, thereafter, De Soliel Apparel Manufacturing Corporation and American the dark as to how it was reached and is especially prejudicial to the losing party, who is
Inter-Fashion Corporation took over Glorious Sun’s manufacturing facilities and absorbed unable to pinpoint the possible errors of the court for review by a higher tribunal.
its employees (petitioners Miguel et al) Obiter
- 1986 > PCGG sequestered De Soleil and AIFC - employer-employee relationship test:
- 1989 > JCT Group, Inc. and De Soleil executed a Management and Operating 1) power to select employees
Agreement for servicing De Soleil’s export quota to ensure its rehabilitation and preserve 2) who pays for their wages
its viability and profitability 3) who has the power to dismiss them, and
- 1990 > De Soleil ceased business operations thus terminating employment 4) who exercises control in the methods and the results by which the work is
- 1993 > complaints for illegal dismissal and payment of backwages before NLRC accomplished
against De Soleil, AIFC, PCGG, Glorious Sun, JCT, Nemesio Co and Vicente Cuevas III. *** The last factor, the “control test,” is the most important.
But JCT and Cuevas filed a motion to dismiss due to lack of jurisdiction because of the Disposition Petition is DENIED and the assailed Decision AFFIRMED
absence of employer-employee relationship between them and petitioners.
- 1995 > Without resolving the motion to dismiss, Labor Arbiter Sampang rendered (1) WACK-WACK GOLF & COUNTRY CLUB V NLRC
Declaring De Soleil, AIFC, PCGG, Glorious Sun, JCT, Nemesio Co and Cuevas jointly (CAGASAN, DOMINGUEZ, BSMI)
and severally guilty of illegal dismissal and to pay complainants backwages, separation
456 SCRA 280
pay, service incentive leave pay, 13th month pay, unpaid salaries as computed by the
Research and Information Unit (2) Declaring De Soleil et al liable for the payment of CALLEJO, SR; April 15, 2005
attorney’s fees (10% of the total awards or P3,691,743.06)
- Because of the huge amount [monetary award, inclusive of attorney’s fees, aggregated NATURE
P41,313,094.98 computed by Research and Information Unit], JCT and Cuevas and Petition for review decision of CA
Glorious Sun filed separate motions with NLRC for reduction of the appeal bond . NLRC
reduced it to P5,000,000.00 for each respondent. Again, they filed a motion for FACTS
reconsideration of said order by way of further reduction of the bond to P500,000.00. CA - A large portion of the Wack Wack (WW) clubhouse (including the kitchen) was
and SCA denied their motions destroyed by fire, and because of this, the management had to suspend operations of
- 1995 > Meanwhile, Glorious Sun and JCT et al et al appealed the labor arbiter’s the Food and Beverage department, requiring the suspension of 54 employees. The
decision to NLRC and petitioners filed a motion to dismiss the appeals bec of failure to Employees’ Union found the suspension as arbitrary and constitutive of union-busting,
post a bond as required in A223 LC and went on strike. The parties soon after entered into an amicable settlement, whereby
- 1996 > NLRC absolved Glorious Sun and dismissed JCT et al’s appeal and sent to CA a “special separation benefit/ retirement package” was formulated. The same provides
for appropriate disposition for, among other things, a 1½ month separation pay for every year of service, and be
- CA: reversed NLRC decision and remanded the case to the labor arbiter bec it found no considered on priority basis for employment by concessionaires and/or contractors, and
factual basis for the ruling that JCT had become the employer of petitioners after the even by the club, upon full resumption of operations.
cessation of operations of Glorious Sun and failed to explain Cuevas liability in solidum - The package was availed of by 3 employees (Cagasan, Dominguez, and Baluyot), who
with AIF, De Soleil and JCT; hence this petition by Miguel et al received large sums of money as separation pay.
Soon after, WW entered into a Management Contract with Business Staffing and
ISSUE Management Inc (BSMI), whereby the latter will provide management services for WW.
WON CA committed grave abuse of discretion amounting in ruling to remand the case to Cagasan and Dominguez filed their application for employment with BMSI. They, by
the labor arbiter because of lack of factual findings to prove employer-employee reason of the priority given by the separation package, were rehired on probationary
relationship between JCT et al and Miguel et al which would be the basis of the liability of status by BMSI.
JCT et al - WW also engaged other contractors in the operations of the club (like janitorial services,
Finance and accounting services). Because of the various management service
HELD contracts, BMSI made an organizational analysis and manpower evaluation to streamline
NO. Instead, NLRC and the Labor arbiter abused their discretion when they ruled in its operations. It found the positions of Cagasan and Domiguez redundant, and
favor of the petitioners without determining the existence of an employer-employee subsequently terminated them. Cagasan and Dominguez then filed complaints in the
relationship between them and respondents because it was silent on why JCT and NLRC for illegal dismissal against WW. NLRC ordered reinstatement
Cuevas were held liable.
Doctrine Grave abuse of discretion implies such capricious and whimsical exercise of ISSUES
judgment as to be equivalent to lack or excess of jurisdiction. That is, power is arbitrarily 1. WON BMSI is an independent contractor (which will answer the question as WON
or despotically exercised by reason of passion, prejudice, or personal hostility; and there was an employer-employee relationship)
caprice is so patent or so gross as to amount to an evasion of a positive duty, or to a 2. WON the employees were illegally dismissed
virtual refusal to perform the duty enjoined or to act at all in contemplation of law.
Reasoning HELD
- LABOR ARBITER: made no determination whether there was employer-employee 1. YES
relationship and, if so, whether JCT et al assumed the obligations of Miguel et al’s Reasoning
previous employers. There is no dispute that given the nature of their functions and - An independent contractor is one who undertakes “job contracting,” i.e., a person who:
length of services, were regular employees. But the question is: who was/were their (a) carries on an independent business and undertakes the contract work on his own
employer/s? account under his own responsibility according to his own manner and method, free from
- MOA: does not appear that JCT became the employer of Miguel et al by virtue of this the control and direction of his employer or principal in all matters connected with the
performance of the work except as to the results thereof; and (b) has substantial capital
Labor Law 1 A2010 - 52 - Disini
or investment in the form of tools, equipments, machineries, work premises and other reinstate him to his former position without loss of seniority rights and privileges and with
materials which are necessary in the conduct of the business. Jurisprudence shows that full backwages from the date of his dismissal until actual reinstatement.
determining the existence of an independent contractor relationship, several factors may - Court of Appeals modified NLRC’s decision that of the NLRC by deleting the award of
be considered, such as, but not necessarily confined to, whether or not the contractor is traveling allowance, and ordering payment of separation pay to De Vera in lieu of
carrying on an independent business; the nature and extent of the work; the skill reinstatement.
required; the term and duration of the relationship; the right to assign the performance of
specified pieces of work; the control and supervision of the work to another; the ISSUES
employer’s power with respect to the hiring, firing, and payment of the contractor’s WON an employer-employee relationship exists between petitioner and respondent
workers; the control of the premises; the duty to supply premises, tools, appliances,
materials and labor; and the mode, manner and terms of payment. HELD
- There is indubitable evidence showing that BSMI is an independent contractor, NO
engaged in the management of projects, business operations, functions, jobs and other - De Vera was an independent contractor beinf the retained physician of petitioner
kinds of business ventures, and has sufficient capital and resources to undertake its company.
principal business. It had provided management services to various industrial and - In a long line of decisions, the Court, in determining the existence of an employer-
commercial business establishments. employee relationship, has invariably adhered to the four-fold test, to wit: the selection
- In December 1993, Labor Sec. Laguesma, in a case, recognized BSMI as an and engagement of the employee; the payment of wages; the power of dismissal; and
independent contractor. As a legitimate job contractor, there can be no doubt as to the the power to control the employee’s conduct, or the so-called “control test”, considered to
existence of an employer-employee relationship between the contractor and the workers. be the most important element.
Thus, there is no employer-employee relation between WW and the workers. - Applying the four-fold test to this case, we initially find that it was respondent himself
2. NO who sets the parameters of what his duties would be in offering his services to petitioner
Ratio As there was no employer-employee relationship between WW and the in the letter which he sent to petitioner.
complainants, there can be no illegal dismissal. - The letter was substantially the basis of the labor arbiter’s finding that there existed no
Reasoning employer-employee relationship between petitioner and respondent, in addition to the
- the complainants (private respondents herein) were validly terminated upon their option following factual settings:
to take the separation package provided by WW. Thus, the same have no cause of - The fact that the complainant was not considered an employee was recognized by the
action against WW. complainant himself in a signed letter, the tenor of which indicated that the complainant
Disposition Petition granted. CA and NLRC decisions set aside was proposing to extend his time with the respondent and seeking additional
compensation for said extension. This shows that the respondent PHILCOM did not
PHILIPPINE GLOBAL COMMUNICATIONS INC V DE have control over the schedule of the complainant as it [is] the complainant who is
proposing his own schedule and asking to be paid for the same. This is proof that the
VERA complainant understood that his relationship with the respondent PHILCOM was a
459 SCRA 260 retained physician and not as an employee. If he were an employee he could not
GARCIA; June 7, 2005 negotiate as to his hours of work.
- De Vera’s service for the respondent was covered by a retainership contract [which]
NATURE was renewed every year from 1982 to 1994. Upon reading the contract dated
petition for review on certiorari September 6, 1982, signed by the complainant himself (Annex ‘C’ of Respondent’s
Position Paper), it clearly states that is a retainership contract. The retainer fee is
FACTS indicated thereon and the duration of the contract for one year is also clearly indicated in
- Petitioner Philippine Global Communications, Inc. (PhilCom), is a corporation engaged paragraph 5 of the Retainership Contract. The complainant cannot claim that he was
in the business of communication services and allied activities, while respondent Ricardo unaware that the ‘contract’ was good only for one year, as he signed the same without
De Vera is a physician by profession whom petitioner enlisted to attend to the medical any objections. The complainant also accepted its renewal every year thereafter until
needs of its employees. 1994. As a literate person and educated person, the complainant cannot claim that he
- It appears that on 15 May 1981, De Vera, via a letter dated 15 May 1981, offered his does not know what contract he signed and that it was renewed on a year to year basis.
services to the petitioner, therein proposing his plan of works required of a practitioner in - The labor arbiter added the indicia, not disputed by respondent, that from the time he
industrial medicine. started to work with petitioner, he never was included in its payroll; was never deducted
- The parties agreed and formalized respondent’s proposal in a document denominated any contribution for remittance to the Social Security System (SSS); and was in fact
as RETAINERSHIP CONTRACT which will be for a period of one year subject to subjected by petitioner to the ten (10%) percent withholding tax for his professional fee,
renewal, it being made clear therein that respondent will cover “the retainership the in accordance with the National Internal Revenue Code, matters which are simply
Company previously had with Dr. K. Eulau” and that respondent’s “retainer fee” will be at inconsistent with an employer-employee relationship.
P4,000.00 a month. Said contract was renewed yearly. The retainership arrangement - Clearly, the elements of an employer-employee relationship are wanting in this case.
went on from 1981 to 1994 with changes in the retainer’s fee. However, for the years We may add that the records are replete with evidence showing that respondent had to
1995 and 1996, renewal of the contract was only made verbally. On December 1996 bill petitioner for his monthly professional fees It simply runs against the grain of common
Philcom, thru a letter bearing on the subject boldly written as “TERMINATION – experience to imagine that an ordinary employee has yet to bill his employer to receive
RETAINERSHIP CONTRACT”, informed De Vera of its decision to discontinue the latter’s his salary.
“retainer’s contract with the Company effective at the close of business hours of - We note, too, that the power to terminate the parties’ relationship was mutually vested
December 31, 1996” because management has decided that it would be more practical on both. Either may terminate the arrangement at will, with or without cause.Finally,
to provide medical services to its employees through accredited hospitals near the remarkably absent from the parties’ arrangement is the element of control, whereby
company premises. the employer has reserved the right to control the employee not only as to the result of
- On 22 January 1997, De Vera filed a complaint for illegal dismissal before the National the work done but also as to the means and methods by which the same is to be
Labor Relations Commission (NLRC), alleging that that he had been actually employed accomplished.
by Philcom as its company physician since 1981 and was dismissed without due - Here, petitioner had no control over the means and methods by which respondent went
process. He averred that he was designated as a “company physician on retainer basis” about performing his work at the company premises. He could even embark in the
for reasons allegedly known only to Philcom. He likewise professed that since he was private practice of his profession, not to mention the fact that respondent’s work hours
not conversant with labor laws, he did not give much attention to the designation as and the additional compensation therefor were negotiated upon by the parties. In fine,
anyway he worked on a full-time basis and was paid a basic monthly salary plus fringe the parties themselves practically agreed on every terms and conditions of respondent’s
benefits, like any other regular employees of Philcom. engagement, which thereby negates the element of control in their relationship. For sure,
- On 21 December 1998, Labor Arbiter Ramon Valentin C. Reyes came out with a respondent has never cited even a single instance when petitioner interfered with his
decision dismissing De Vera’s complaint for lack of merit, on the rationale that as a work.
“retained physician” under a valid contract mutually agreed upon by the parties, De Vera Disposition petition is GRANTED and the challenged decision of the Court of Appeals
was an “independent contractor” and that he “was not dismissed but rather his contract REVERSED and SET ASIDE. The 21 December 1998 decision of the labor arbiter is
with [PHILCOM] ended when said contract was not renewed after December 31, 1996”. REINSTATED.
NLRC reversed (the word used is “modified”) that of the Labor Arbiter, on a finding that
De Vera is Philcom’s “regular employee” and accordingly directed the company to
Labor Law 1 A2010 - 53 - Disini
SONZA V ABS-CBN - The rationale for allowing only one motion for reconsideration from the same party is to
assist the parties in obtaining an expeditious and inexpensive settlement of labor cases.
[PAGE 42] For obvious reasons, delays cannot be countenanced in the resolution of labor disputes.
The dispute may involve no less than the livelihood of an employee and that of his loved
JARDIN V NLRC (PHILJAMA INTL) ones who are dependent upon him for food, shelter, clothing, medicine, and education. It
326 SCRA 299 may as well involve the survival of a business or an industry.
- As correctly pointed out by petitioner, the second motion for reconsideration filed by
QUISUMBING; February 23, 2000 private respondent is indubitably a prohibited pleading which should have not been
entertained at all. Public respondent cannot just disregard its own rules on the pretext of
NATURE "satisfying the ends of justice", especially when its disposition of a legal controversy ran
Special civil action for certiorari seeks to annul the decision of public respondent afoul with a clear and long standing jurisprudence in this jurisdiction as elucidated in the
promulgated on October 28, 1994, in NLRC NCR CA No. 003883-92, and its resolution subsequent discussion. Clearly, disregarding a settled legal doctrine enunciated by this
dated December 13, 1994 which denied petitioners motion for reconsideration. Court is not a way of rectifying an error or mistake. In our view, public respondent gravely
abused its discretion in taking cognizance and granting private respondent’s second
FACTS motion for reconsideration as it wrecks the orderly procedure in seeking reliefs in labor
- Petitioners were drivers of private respondent, Philjama International Inc., a domestic cases.
corporation engaged in the operation of "Goodman Taxi." Petitioners used to drive private Obiter
respondent’s taxicabs every other day on a 24-hour work schedule under the boundary - There is another compelling reason why we cannot leave untouched the flip-flopping
system. Under this arrangement, the petitioners earned an average of P400.00 daily. decisions of the public respondent. As mentioned earlier, its October 28, 1994 judgment
Nevertheless, private respondent admittedly regularly deducts from petitioners’ daily is not in accord with the applicable decisions of this Court. The labor tribunal reasoned
earnings the amount of P30.00 supposedly for the washing of the taxi units. Believing out as follows:
that the deduction is illegal, petitioners decided to form a labor union to protect their - Four-fold test for employer-employee relations:
rights and interests. (1) the selection and engagement of the employee;
- Upon learning about the plan of petitioners, private respondent refused to let petitioners (2) the payment of wages;
drive their taxicabs when they reported for work on August 6, 1991, and on succeeding (3) the power of dismissal; and
days. Petitioners suspected that they were singled out because they were the leaders (4) the power of control the employees conduct.’
and active members of the proposed union. Aggrieved, petitioners filed with the labor - NLRC found that the boundary system is a leasehold system which takes it out of the
arbiter a complaint against private respondent for unfair labor practice, illegal dismissal ordinary notion of “control” over employees conduct.
and illegal deduction of washing fees. In a decision dated August 31, 1992, the labor - The SC iterated its ruling that the relationship between jeepney owners/operators on
arbiter dismissed said complaint for lack of merit. one hand and jeepney drivers on the other under the boundary system is that of
- On appeal, the NLRC (public respondent herein), in a decision dated April 28, 1994, employer-employee and not of lessor-lessee.
reversed and set aside the judgment of the labor arbiter. The labor tribunal declared that - The SC explained that in the lease of chattels, the lessor loses complete control over
petitioners are employees of private respondent, and, as such, their dismissal must be the chattel leased although the lessee cannot be reckless in the use thereof, otherwise
for just cause and after due process. he would be responsible for the damages to the lessor. In the case of jeepney
- Private respondent’s first motion for reconsideration was denied. Remaining hopeful, owners/operators and jeepney drivers, the former exercise supervision and control over
private respondent filed another motion for reconsideration. This time, public respondent, the latter. The management of the business is in the owner’s hands. The owner as holder
in its decision dated October 28, 1994, granted aforesaid second motion for of the certificate of public convenience must see to it that the driver follows the route
reconsideration. It ruled that it lacks jurisdiction over the case as petitioners and private prescribed by the franchising authority and the rules promulgated as regards its
respondent have no employer-employee relationship. It held that the relationship of the operation.
parties is leasehold which is covered by the Civil Code rather than the Labor Code. - As consistently held by this Court, termination of employment must be effected in
accordance with law. The just and authorized causes for termination of employment are
ISSUE enumerated under Articles 282, 283 and 284 of the Labor Code. The requirement of
WON the NLRC committed grave abuse of discretion in entertaining the motion for notice and hearing is set-out in Article 277 (b) of the said Code. Hence, petitioners, being
reconsideration and in holding that there is no employer-employee relationship in the employees of private respondent, can be dismissed only for just and authorized cause,
“boundary system.” and after affording them notice and hearing prior to termination. In the instant case,
private respondent had no valid cause to terminate the employment of petitioners.
HELD Neither were there two (2) written notices sent by private respondent informing each of
YES the petitioners that they had been dismissed from work. These lack of valid cause and
Ratio Only one motion for reconsideration from the same party is allowed before the failure on the part of private respondent to comply with the twin-notice requirement
NLRC in line with the policy of assisting the parties in obtaining an expeditious and underscored the illegality surrounding petitioners’ dismissal.
inexpensive settlement of labor cases. When the NLRC entertained the second motion - Under the law, an employee who is unjustly dismissed from work shall be entitled to
for reconsideration, it therefore committed grave abuse of discretion. reinstatement without loss of seniority rights and other privileges and to his full
Reasoning backwages, inclusive of allowances, and to his other benefits or their monetary
- The phrase "grave abuse of discretion amounting to lack or excess of jurisdiction" has equivalent computed from the time his compensation was withheld from him up to the
settled meaning in the jurisprudence of procedure. It means such capricious and time of his actual reinstatement It must be emphasized, though, that recent judicial
whimsical exercise of judgment by the tribunal exercising judicial or quasi-judicial power pronouncements distinguish between employees illegally dismissed prior to the effectivity
as to amount to lack of power. of Republic Act No. 6715 on March 21, 1989, and those whose illegal dismissals were
- In this case before us, private respondent exhausted administrative remedy available to effected after such date. Thus, employees illegally dismissed prior to March 21, 1989, are
it by seeking reconsideration of public respondent’s decision dated April 28, 1994, which entitled to backwages up to three (3) years without deduction or qualification, while those
public respondent denied. With this motion for reconsideration, the labor tribunal had illegally dismissed after that date are granted full backwages inclusive of allowances and
ample opportunity to rectify errors or mistakes it may have committed before resort to other benefits or their monetary equivalent from the time their actual compensation was
courts of justice can be had. Thus, when private respondent filed a second motion for withheld from them up to the time of their actual reinstatement. The legislative policy
reconsideration, public respondent should have forthwith denied it in accordance with behind Republic Act No. 6715 points to "full backwages" as meaning exactly that, i.e.,
Rule 7, Section 14 of its New Rules of Procedure which allows only one motion for without deducting from backwages the earnings derived elsewhere by the concerned
reconsideration from the same party, thus: employee during the period of his illegal dismissal. Considering that petitioners were
"SEC. 14. Motions for Reconsideration. --- Motions for reconsideration of any order, terminated from work on August 1, 1991, they are entitled to full backwages on the basis
resolution or decision of the Commission shall not be entertained except when based of their last daily earnings.
on palpable or patent errors, provided that the motion is under oath and filed within
ten (10) calendar days from receipt of the order, resolution or decision with proof of
service that a copy of the same has been furnished within the reglementary period the
adverse party and provided further, that only one such motion from the same party
shall be entertained." [Emphasis supplied]
Labor Law 1 A2010 - 54 - Disini
ISSUE
MANILA GOLF & COUNTRY CLUB INC V IAC WON persons rendering caddying services for members of golf clubs and their guests in
(LLAMAR) said clubs' courses or premises are the employees of such clubs and therefore within the
337 SCRA 207 compulsory coverage of the Social Security System (SSS)
NARVASA; September 27, 1994
HELD
NO
NATURE
Ratio The Court does not agree that said facts necessarily or logically point to such a
Petition for review
relationship, and to the exclusion of any form of arrangements, other than of
employment, that would make the respondent's services available to the members and
FACTS
guest of the petitioner. As long as it is, the list made in the appealed decision detailing
- three separate proceedings, all initiated by or on behalf of herein private respondent
the various matters of conduct, dress, language, etc. covered by the petitioner's
and his fellow caddies:
regulations, does not, in the mind of the Court, so circumscribe the actions or judgment
1) filed with the Social Security Commission (SSC) via petition of 17 persons who styled
of the caddies concerned as to leave them little or no freedom of choice whatsoever in
themselves "Caddies of Manila Golf and Country Club-PTCCEA (Philippine Technical,
the manner of carrying out their services.
Clerical, Commercial Employees Association)” for coverage and availment of benefits
Reasoning
under the Social Security Act. It alleged that although the petitioners were employees of
- In the very nature of things, caddies must submit to some supervision of their conduct
the Manila Golf and Country Club, a domestic corporation, the latter had not registered
while enjoying the privilege of pursuing their occupation within the premises and grounds
them as such with the SSS.
of whatever club they do their work in. For all that is made to appear, they work for the
2) a certification election case filed with the Labor Relations Division of the Ministry of
club to which they attach themselves on sufference but, on the other hand, also without
Labor by the PTCCEA on behalf of the same caddies- it was resolved in favor of the
having to observe any working hours, free to leave anytime they please, to stay
petitioners
away for as long they like. It is not pretended that if found remiss in the observance of
3) a compulsory arbitration case initiated before the Arbitration Branch of the Ministry of
said rules, any discipline may be meted them beyond barring them from the
Labor by the same labor organization- it was dismissed for lack of merit by Labor Arbiter
premises which, it may be supposed, the Club may do in any case even absent
on the ground that there was no employer-employee relationship between the petitioning
any breach of the rules, and without violating any right to work on their part. All
caddies and the respondent Club
these considerations clash frontally with the concept of employment. The IAC would point
- In the case before the SSC, the Club filed answer praying for the dismissal of the
to the fact that the Club suggests the rate of fees payable by the players to the caddies
petition, alleging in substance that the petitioners, caddies by occupation, were allowed
as still another indication of the latter's status as employees. It seems to the Court,
into the Club premises to render services as such to the individual members and guests
however, that the intendment of such fact is to the contrary, showing that the Club has
playing the Club's golf course and who themselves paid for such services; that as such
not the measure of control over the incidents of the caddies' work and
caddies, the petitioners were not subject to the direction and control of the Club as
compensation that an employer would possess. In the final analysis, petitioner has
regards the manner in which they performed their work; and hence, they were not the
no was of compelling the presence of the caddies as they are not required to
Club's employees.
render a definite number of hours of work on a single day . Even the group rotation
- Subsequently, all but two (Fermin Llamar and Raymundo Jomok) of the 17 petitioners
of caddies is not absolute because a player is at liberty to choose a caddy of his
of their own accord withdrew their claim for social security coverage, avowedly coming to
preference regardless of the caddy's order in the rotation.
realize that indeed there was no employment relationship between them and the Club.
Obiter (on issue of res judicata)
The Commission dismissed the petition for lack of merit:”. . . that the caddy's fees were
- That same issue of res adjudicata, ignored by the IAC beyond bare mention thereof, as
paid by the golf players themselves and not by respondent club...While respondent club
already pointed out, is now among the mainways of the private respondent's defenses to
promulgates rules and regulations on the assignment, deportment and conduct of
the petition for review.
caddies the same are designed to impose personal discipline among the caddies but not
- Because the same question of employer-employee relationship has been dragged into
to direct or conduct their actual work. In fact, a golf player is at liberty to choose a caddy
three different fora, willy-nilly and in quick succession, it has birthed controversy as to
of his preference regardless of the respondent club's group rotation system and has the
which of the resulting adjudications must now be recognized as decisive. On the one
discretion on whether or not to pay a caddy...This lends credence to respondent's
hand, there is the certification case where the decision found for the existence of
assertion that the caddies are never their employees in the absence of two elements,
employer-employee relationship between the parties; on the other, the compulsory
namely, (1) payment of wages and (2) control or supervision over them.
arbitration case which was dismissed for lack of merit on the ground that there existed no
- From this Resolution appeal was taken to the IAC by the union representing Llamar and
such relationship between the Club and the private respondent.
Jomok. The appeal ascribed two errors to the SSC: (1) refusing to suspend the
- It is well settled that for res adjudicata, or the principle of bar by prior judgment, to
proceedings to await judgment by the Labor Relations Division of National Capital
apply, the following essential requisites must concur: (1) there must be a final judgment
Regional Office in the certification election on the precise issue of the existence of
or order; (2) said judgment or order must be on the merits; (3) the court rendering the
employer-employee relationship between the respondent club and the appellants, it
same must have jurisdiction over the subject matter and the parties; and (4) there must
being contended that said issue was "a function of the proper labor office"; and (2)
be between the two cases identity of parties, identity of subject matter and identity of
adjudicating that self same issue a manner contrary to the ruling of the Director of the
cause of action.
Bureau of Labor Relations, which "has not only become final but (has been) executed or
- A certification proceedings is not a "litigation" in the sense in which the term is
(become) res adjudicata."
commonly understood, but mere investigation of a non-adversary, fact-finding character,
- IAC: declared Fermin Llamar an employee of the Manila Gold and Country Club,
in which the investigating agency plays the part of a disinterested investigator seeking
ordering that he be reported as such for social security coverage and paid any
merely to ascertain the desires of the employees as to the matter of their representation.
corresponding benefits, but it conspicuously ignored the issue of res adjudicata raised in
- In any case, this Court is not inclined to allow private respondent the benefit of any
said second assignment.
doubt as to which of the conflicting ruling just adverted to should be accorded primacy,
- The questioned employer-employee relationship between the Club and Fermin Llamar
given the fact that it was he who actively sought them simultaneously, as it were, from
passed the so-called "control test," establishment in the case — i.e., "whether the
separate fora. Accordingly, the IAC is not to be faulted for ignoring private respondent's
employer controls or has reserved the right to control the employee not only as to the
invocation of res adjudicata; on contrary, it acted correctly in doing so.
result of the work to be done but also as to the means and methods by which the same is
Disposition Reversed and set aside, it being hereby declared that the private
to be accomplished," — the Club's control over the caddies encompassing:
respondent, Fermin Llamar, is not an employee of petitioner Manila Golf and Country
(a) the promulgation of no less than 24 rules and regulations just about every aspect
Club and that petitioner is under no obligation to report him for compulsory coverage to
of the conduct that the caddy must observe, or avoid, when serving as such, any
the Social Security System.
violation of any which could subject him to disciplinary action, which may include
suspending or cutting off his access to the club premises; (b) the devising and
enforcement of a group rotation system whereby a caddy is assigned a number which
designates his turn to serve a player; (c) the club's "suggesting" the rate of fees
payable to the caddies.
Labor Law 1 A2010 - 55 - Disini
WON petitioner’s removal from a permanent position (Medical Specialist 1), as a result of
the reorganization of the DoH, was void for being violative of the constitutional provision
FELIX V BUENASEDA on security of tenure
240 SCRA 139 HELD
KAPUNAN; January 17, 1995 NO
Reasoning
NATURE - A residency or resident physician position in a medical specialty is never a permanent
Petition for review on certiorari one. Residency connotes training and temporary status. Promotion to the next post-
graduate year is based on merit and performance determined by periodic evaluations
FACTS and examinations of knowledge, skills and bedside manner. Under this system,
- Petitioner Dr. Alfredo Felix joined the National Center for Mental Health (NCMH) as a residents, especially those in university teaching hospitals enjoy their right to security of
resident physician and after only 3 years, was promoted to Senior Resident Physician, a tenure only to the extent that they periodically make the grade. While physicians (or
position he held until the Ministry of Health reorganized the NCMH in Jan. 1988, pursuant consultants) of specialist rank are not subject to the same stringent evaluation
to E.O. 119. Under the said reorganization, Felix was appointed to the position of Sr. procedures, specialty societies require continuing education as a requirement for
Resident Physician in a temporary capacity immediately after he and other employees accreditation in good standing, in addition to peer review processes based on
allegedly tendered their courtesy resignations to the Sec. of Health. Felix was later performance, mortality and morbidity audits, feedback from residents, interns and
promoted to the position of Medical Specialist 1 (Temporary Status) which was renewed medical students and research output. The nature of the contracts of resident
the following year. physicians meets traditional tests for determining employer employee
- In 1988, the Dept. of Health or DoH issued Dept. Order (D.O.) 347, which required relationships, but because the focus of residency is training, they are neither
board certification as a prerequisite for renewal of specialist positions in various med. here nor there. Moreover, stringent standards and requirements for renewal of
centers, hospitals and agencies and specifically provided that specialists working in specialist rank positions or for promotion to the next postgraduate residency year are
various branches of DoH be recognized as “Fellows” of their respective societies and/or necessary because lives are ultimately at stake.
“Diplomates” of their specialty boards or both, the purpose of which was to upgrade the - From the position of senior resident physician, which he held at the time of the
quality of specialists in DoH hospitals by requiring them to pass rigorous theoretical and government reorganization, the next logical step in the stepladder process was obviously
clinical exams given by recognized specialty boards. his promotion to the rank of Medical Specialist 1, a position which he apparently
- (Then) Sec. of Health Alfredo Bengzon issued D.O. 478 (amending Sec.4 of D.O. 347) accepted. Such status, however, clearly carried with it certain professional
which provided for an extension of appointments of Medical Specialists in cases where responsibilities including the responsibility of keeping up with the minimum requirements
termination of those who failed to meet the requirement for board certification might of specialty rank, the responsibility of keeping abreast with current knowledge in his
result in disruption of hospital services. The said order provided, among others, that: specialty and in Medicine in general, and the responsibility of completing board
xxxxx 2. Medical specialists recommended for extension of appointment shall meet certification requirements within a reasonable period of time. The evaluation made by
the following minimum criteria: petitioner's peers and superiors clearly showed that he was deficient in a lot of areas, in
a. DOH medical specialist certified addition to the fact that at the time of his non-renewal, he was not even board-certified.
b. Has been in the service of the Department at least three (3) years prior to - As respondent CSC has correctly pointed out, the appointment was for a definite and
December 1988 renewable period which, when it was not renewed, did not involve a dismissal but an
c. Has applied or taken the specialty board examination. expiration of the petitioner’s term.
- In 1991, after reviewing petitioner’s service record and performance, the Medical *On estoppel by laches:
Credentials Committee of the NCMH recommended non-renewal of his appointment as - Public policy and convenience demand that any claim to any position in the civil
Medical Specialist 1, informing him of its decision on Aug. 22, 1991. He was, however, service, permanent, temporary or otherwise, or any claim to a violation of the
allowed to continue in the service, and receive his salary even after being informed of the constitutional provision on security of tenure be made within a reasonable period of time.
termination of his appointment. The failure to assert a claim or the voluntary acceptance of another position in
- On Nov. 25, 1991, the Chiefs of Service held an emergency meeting to discuss the government, obviously without reservation, leads to a presumption that the civil servant
petitioner’s case. In the meeting, the overall consensus among the dept. heads was for has either given up his claim or has already settled into the new position. This is the
petitioner’s non-renewal where his poor performance, frequent tardiness and inflexibility essence of laches which is the failure or neglect, for an unreasonable and unexplained
were pointed as among the factors responsible for the recommendation not to renew his length of time to do that which, by exercising due diligence, could or should have been
appointment. The matter was referred to the CSC, which ruled that the temporary done earlier; it is the negligence or omission to assert a right within a reasonable time,
appointment can be terminated any time and that any renewal of such appointment is warranting a presumption that the party entitled to assert it either has abandoned it or
within the discretion of the appointing authority. Consequently, petitioner was advised by declined to assert it.
hospital authorities to vacate his cottage. Refusing to comply, petitioner filed a petition - Petitioner made no attempt to oppose earlier renewals of his temporary Specialist 1
with the Merit System Protection Board (MSPB) complaining about the alleged contracts, clearly demonstrating his acquiescence to - if not his unqualified acceptance of
harassment and questioning the non-renewal of his appointment, the MSPB, however, - the promotion (albeit of a temporary nature). Whatever objections petitioner had against
dismissed his complaint for lack of merit. the earlier change from the status of permanent senior resident physician to temporary
- This decision was appealed to the Civil Service Commission (CSC) which dismissed senior resident physician were neither pursued nor mentioned at or after his designation
the same. The MFR was also denied by the CSC hence this appeal. as Medical Specialist 1 (Temporary). He is therefore estopped from insisting upon a right
Petitioner’s claims or claim which he had plainly abandoned when he, from all indications, enthusiastically
1. CSC erred in holding that by submitting his courtesy resignation and accepting his accepted the promotion. His negligence to assert his claim within a reasonable time,
temporary appointment, petitioner had effectively divested himself of his security of coupled with his failure to repudiate his promotion to a temporary position, warrants a
tenure, considering the circumstances of such courtesy resignation and acceptance of presumption that he either abandoned (his claim) or declined to assert it.
appointment. Disposition Petition dismissed for lack of merit
2. Respondent commission erred in not declaring that the conversion of the permanent
appointment of petitioner to temporary was done in bad faith in the guise of R TRANSPORT CORP V EJANDRA
reorganization and thus invalid, being violative of the petitioner’s right of security of
428 SCRA 724
tenure.
Respondent CSC’s claims CORONA; May 20, 2004
1. The petitioner’s temporary appointments after the reorganization pursuant to E.O. 119
were valid and did not violate his constitutional right to security of tenure. NATURE
2. Petitioner is guilty of estoppel or laches, having acquiesced to such temporary Petition for review of the decision of the CFI of Iloilo
appointments from 1988-1991
3. The respondent CSC did not act with grave abuse of discretion in affirming the FACTS
petitioner’s non-renewal of his appointment to the NCMH. - Rogelio Ejandra worked for petitioner bus company as a driver.
- On Jan 31 1996, he was apprehended for obstruction of traffic. His license was
ISSUE confiscated. He reported this to his manager, Oscar Pasquin, who gave him P500 to
redeem the license. He was able to retrieve the license after a week since the
apprehending officer turned it in only then.
Labor Law 1 A2010 - 56 - Disini
- On feb 8, 1996, he reported for work. The company said they were reviewing if they - In May 1979, the Company terminated the Agency Manager's Contract. After vainly
were going to allow him drive again. Also, he was being blamed for damage to the bus. seeking a reconsideration, Basiao sued the Company in a civil action and this (he
Ejandra said the bus was damaged during the week he wasn’t able to drive. claimed) prompted the latter to terminate also his engagement under the first contract
- Petitioner, on the other hand, claims that Ejandra is a habitual absentee and has and to stop payment of his commissions starting April 1, 1980.
abandoned his job. To belie private respondent’s allegation that his license had been - Basiao filed w/ the Ministry of Labor a complaint against the Company and its
confiscated, petitioner asserted that, had it been true, he should have presented an president. The complaint sought to recover commissions allegedly unpaid, plus
apprehension report and informed petitioner of his problems with the LTO. But he did attorney's fees. The respondents claim: Ministry had no jurisdiction over Basiao's claim,
not. Petitioner further argued that private respondent was not an employee because asserting that he was not the Company's employee, but an independent contractor and
theirs was a contract of lease and not of employment, with petitioner being paid on that the Company had no obligation to him for unpaid commissions under the terms and
commission basis conditions of his contract.
- The labor arbiter ruled in favor of Ejandra. It was held that he didn’t abandon his work, - The Labor Arbiter found for Basiao. He ruled that the underwriting agreement had
since there was valid reason for his 1 week absence. He also was not afforded due established an employer-employee relationship between him and the Company, and this
process. NLRC and CA affirmed. conferred jurisdiction on the Ministry of Labor to adjudicate his claim. Said official's
decision directed payment of his unpaid commissions ". . . equivalent to the balance of
ISSUES the first year's premium remaining unpaid, at the time of his termination, of all the
1. WON there was an employee employer relationship insurance policies solicited by . . . (him) in favor of the respondent company . . ." plus
2. WON Ejandra was dismissed for a just cause 10% attorney's fees.
- This decision was, on appeal by the Company, affirmed by the NLRC.
HELD
1. YES ISSUE
- Petitioner is barred to negate the existence of an employer-employee relationship. He WON Basiao had become the Company's employee by virtue of the contract invoked by
has invoked rulings on the right of an employer to dismiss an employee for just cause. him, thereby placing his claim for unpaid commissions within the original and exclusive
The power to dismiss an employee is one of the indications that there was such jurisdiction of the Labor Arbiter under the provisions of Section 217 of the Labor Code
relationship. Also, A97 of the Labor Code says that employees can be paid in form of
commissions. HELD
2. NO NO
- To constitute abandonment, two elements must concur: (1) the failure to report for work - Basiao was not an employee of the petitioner, but a commission agent, an independent
or absence without valid or justifiable reason and (2) a clear intention to sever the contractor whose claim for unpaid commissions should have been litigated in an ordinary
employer-employee relationship. Petitioner did not fulfill the requisites. First, Ejandra’s civil action. The Labor Arbiter erred in taking cognizance of, and adjudicating, said claim,
absence was justified since his license wasn’t release until after a week. Second, being without jurisdiction to do so, as did the respondent NLRC in affirming the Arbiter's
Ejandra did not want to sever their relationship when he got his license back. Third, labor decision. This conclusion renders it unnecessary and premature to consider Basiao's
arbiter Yulo correctly observed that, if private respondent really abandoned his work, claim for commissions on its merits.
petitioner should have reported such fact to the nearest Regional Office of the -“Control test" (Viana vs. Alejo Al-Lagadan, 1956):
Department of Labor and Employment in accordance with Section 7, Rule XXIII, Book V "In determining the existence of employer-employee relationship, the following
of Department Order No. 9, series of 1997 (Rules Implementing Book V of the Labor elements are generally considered, namely: (1) the selection and engagement of the
Code). Petitioner made no such report. employee; (2) the payment of wages; (3) the power of dismissal; and (4) the power
- In addition, he wasn’t also given due process by not giving him notice and hearing. to control the employees' conduct - although the latter is the most important
Disposition Decision reversed element (35 Am. Jur. 445). . . ,"
- However, not every form of control that the hiring party reserves to himself over the
conduct of the party hired in relation to the services rendered may be accorded the effect
SONZA V ABS-CBN of establishing an employer-employee relationship between them in the legal or technical
[PAGE 42] sense of the term.
- Logically, the line should be drawn between rules that merely serve as guidelines
INSULAR LIFE V NLRC (BASIAO) towards the achievement of the mutually desired result without dictating the means or
methods to be employed in attaining it, and those that control or fix the methodology and
179 SCRA 459 bind or restrict the party hired to the use of such means. The first, which aim only to
NARVASA; November 15, 1989 promote the result, create no employer-employee relationship unlike the second, which
address both the result and the means used to achieve it.
NATURE - Rules and regulations governing the conduct of the business are provided for in the
Petition for certiorari and prohibition to review the resolution of the NLRC. Insurance Code and enforced by the Insurance Commissioner. It is, therefore, usual and
expected for an insurance company to promulgate a set of rules to guide its commission
FACTS agents in selling its policies that they may not run afoul of the law and what it requires or
- In 1968, Insular Life Assurance Co., Ltd. (Company) and Melecio T. Basiao entered into prohibits. Of such a character are the rules which prescribe the qualifications of persons
a contract by w/c Basiao was "authorized to solicit w/in the Phils applications for who may be insured, subject insurance applications to processing and approval by the
insurance policies and annuities in accordance with the existing rules and regulations" of Company, and also reserve to the Company the determination of the premiums to be
the Company; he would receive "compensation, in the form of commissions . . . ", and paid and the schedules of payment. None of these really invades the agent's contractual
the "rules in Rate Book and its Agent's Manual, as well as all its circulars and those prerogative to adopt his own selling methods or to sell insurance at his own time and
which may from time to time be promulgated by it . . ." were made part of said contract. convenience, hence cannot justifiably be said to establish an employer-employee
- The contract also contained provisions governing the relations of the parties, the duties relationship between him and the company.
of the Agent, the acts prohibited to him, and the modes of termination of the agreement, - Mafinco Trading Corporation v Ople : a person engaged to sell soft drinks for
viz.: another, using a truck supplied by the latter, but with the right to employ his own workers,
"RELATION WITH THE COMPANY. The Agent shall be free to exercise his own sell according to his own methods subject only to prearranged routes, observing no
judgment as to time, place and means of soliciting insurance. Nothing herein working hours fixed by the other party and obliged to secure his own licenses and defray
contained shall therefore be construed to create the relationship of employee and his own selling expenses, all in consideration of a peddler's discount given by the other
employer between the Agent and the Company. However, the Agent shall observe and party for at least 250 cases of soft drinks sold daily, was not an employee but an
conform to all rules and regulations which the Company may from time to time independent contractor.
prescribe. - Investment Planning Corporation of the Philippines v SSS : there was no
"TERMINATION. The Company may terminate the contract at will, without any employer-employee relationship between a commission agent and an investment
previous notice to the Agent, for or on account of . . . (explicitly specified causes) . . . company, but that the former was an independent contractor where said agent and
- in April 1972, the parties entered into another contract - an Agency Manager's others similarly placed were: (a) paid compensation in the form of commissions based on
Contract, while Basiao concurrently fulfilled his commitments under the first contract percentages of their sales, any balance of commissions earned being payable to their
with the Company. legal representatives in the event of death or registration; (b) required to put up
performance bonds; (c) subject to a set of rules and regulations governing the
Labor Law 1 A2010 - 57 - Disini
performance of their duties under the agreement with the company and termination of
their services for certain causes; (d) not required to report for work at any time, nor to ISSUE
devote their time exclusively to working for the company nor to submit a record of their WON there was an existence of an employer-employee relationship
activities, and who, finally, shouldered their own selling and transportation expenses.
- Sara v NLRC : one who had been engaged by a rice miller to buy and sell rice and HELD
palay without compensation except a certain percentage of what he was able to buy or 1. NO
sell, did work at his own pleasure without any supervision or control on the part of his Ratio Under the control test, an employer-employee relationship exists where the
principal and relied on his own resources in the performance of his work, was a plain person for whom the services are performed reserves the right to control not only the end
commission agent, an independent contractor and not an employee. achieved but also the manner and means to be used in reaching the end.
Reasoning
- The Court has consistently held a four tier test to evaluate the existence of an
ALMIREZ V INFINITE LOOP TECHNOLOGY employer-employee relationship which include: 1) manner of selection of engagement, 2)
CORPORATION payment of wages, 3) presence or absence of power of dismissal and 4) presence or
481 SCRA 364 absence of power of control.
CARPIO-MORALES; January 31, 2006 - The last test is known as the “control test” and is regarded as the most crucial and
determinative indicator of the presence of absence of an employer-employee
relationship.
FACTS
- There is no showing of a controlling power of Infinite Loop over Almirez. They only
- Almirez was hired as a Refinery Senior Process Design Engineer for a specific project
specified what she needed to achieve but now how she was go on about it.
by respondent Infinite Loop Technology Corporation through its General Manager
- The company had hired her based on her expertise but the company naturally had to
Rubino.
appraised of the work progress.
- September 30, 1999 – Details were furnished to Almirez regarding her designation in
- The deduction for SSS and tax do not bolster Almirez’s contention that there was an
the company as well as the scope of her services. The scope of the services was to
employee-employer relationship. However, only one pay slip was issued (Januaryb 16-
commence on October 18, 1999 and had a guarantee of 12 continuous months.
31, 2000) and the rest were in cash vouchers. As such, the payslip cannot be
> The senior process design engineer was to work together with the Process Design
considered as proof of an employer-employee relationship.
Consultant in performing the scope of the services which included preparing the
- The use of the word “salary” is not determinative of such a relationship either. Salary is
process terms of reference or basis of design for the BPSD Petroleum Refinery, to
defined as remuneration for services given. The contract details her salary and it serves
review and revise as necessary the existing conceptual process block diagram of the
between the parties was the law governing them. But the contract, as pointed out earlier,
process flow scheme, implement new process technologies, participate in
is bereft of proof of control of Infinite Loop over Almirez.
discussions, make recommendation reports to the company management team,
Disposition Petition is denied for lack of merit with costs against petitioner.
represent the company in meetings and perform other related works.
> She was to be paid a professional fee of US$2,000 per month (net of tax) to be
50/50 split in US dollars or equivalent peso every 15th of the month. She also had SEVILLA V CA (TOURIST WORLD SERVICES)
other benefits and bonuses along with equipment such as a laptop computer. 160 SCRA 171
- Almirez received a total of P77,000 the following amounts on the dates indicated:
SARMIENTO; April 15, 1998
- 11-23-09 – P20,000 (Salary for Nov. 1-15)
12-02-99 – P8,000 (Salary for Nov. 15-30)
12-15-99 – P2,000 (Full payment for Nov. 15-30 salary)
P10,000 (Salary for Dec. 1-15,) NATURE
1-17-00 – P12,000 (Salary for January 1-15) Appeal by certiorari
1-16-00 – P12,500 (Salary for January 16-31)
1-20-00 – P12,500 (Salary for January 1-15) FACTS
- Almirez then wrote a letter to the company, expressing her disappointment because she - On the strength of a contract, Tourist World Service Inc. (TWS) leased the premises
was receiving less than expected. She hdadexpected the amount to be net of taxes but belonging to Mrs. Segundina Noguera for the former’s use as a branch office. Lina
she was receiving less because of SSS deductions and tax deductions. She asked that Sevilla bound herself solidarily liable with TWS for the prompt payment of the monthly
her SSS dues be not deducted from her salary because she was voluntarily paying such rentals thereon.
obligations on her own. She further stated that she was willing to render her services at - When the branch office was opened, it was run by appellant Sevilla payable to TWS by
Infinite Loop based on the contract and that she was willing to serve as technical any airline for any fare brought in on the efforts of Sevilla, 4% was to go to Sevilla and
consultant on other relevant projects. 3% was to be withheld by TWS.
- Rabino said that Almirez’s letter was different from what they had previously agreed - TWS appears to have been informed that Sevilla was connected with a rival firm, the
upon. According to him, the BPSD project, like any other project, could be deferred and Philippine Travel Bureau, and, since the branch office was anyhow losing, the TWS
that since the engineering design for the proposed project was not yet available, it would considered closing down its office.
be prudent to suspend the professional services of Almirez as Senior Process Design - This was firmed up by two resolutions of the TWS board of directors to abolish the office
Engineer effective February 2007. of the manager and VP of the branch office and authorizing the corporate secretary to
- Almirez, through counsel, wrote a letter to Rabino, asking that she be properly receive the properties in the said branch office.
compensated with the total amount of her contract because the contract stated that her - The corporate secretary went to the branch office, and finding the premises locked and
tenure would last for 12 months but she had already been suspended by February of being unable to contact Sevilla, padlocked the premises to protect the interests of TWS.
2000. Almirez also noted that she had been paid only P74,229.17 which is way below - When neither Sevilla nor her employees could enter the locked premises, she filed a
the amount promised to her of US$2,000 a month net of tax. complaint against TWS with a prayer for the issuance of a mandatory preliminary
- Rabino responded by explaining to Almirez that the company and its partner injunction.
corporations were all experiencing financial difficulties with their projects and asked her - The trial court dismissed the case holding that TWS, being the true lessee, was within
to “bear with them.” its prerogative to terminate the lease and padlock the premises. It likewise found that
- December 12, 2000 – Almirez filed a complaint against Infinite Loop for breach of Sevilla was a mere employee of TWS and as such, was bound by the acts of her
contract of employment. Infinite Loop countered by saying that the NLRC had no employer.
jurisdiction to hear the case because there was no employer-employee relationship and - The CA affirmed. Hence this petition.
the contract was one of services, not employment.
- The Labor Arbiter ruled that there was an existence of an employer-employee ISSUE
relationship and ordered Infinite Loop to pay Almirez US$24,000 in its peso equivalent 1. WON there was an employer-employee relation between TWS and Sevilla
less advances of P77,000. Infinite Loop appealed to the NLRC but the appeal was 2. WON the padlocking of the premises by TWS without the knowledge and consent of
dismissed. Sevilla entitled the latter to the relief of damages prayed for
- The Court of Appeals found that the primary cause of Almirez’s action was that for a
sum of money on account of an alleged breach of contract to pay a professional fee. It HELD
held that there was no employer-employee relationship so the NLRC and the Labor 1. NO. It was a principal-agent relationship.
Arbiter have no jurisdiction over the said case. Thus Almirez’s petition was dismissed.
Labor Law 1 A2010 - 58 - Disini
Ratio In this jurisdiction, there has been no uniform test to determine the existence of any reason other than for cause, the acting unit manager would be reverted to agent
an employer-employee relation. In general, We have relied on the so-called right of status and assigned to any unit. As in the previous agency contract, De los Reyes
control test, “where the person for whom the services are performed reserves a right to together with his unit force was granted freedom to exercise judgment as to time, place
control not only the end to be achieved but also the means to be used in reaching such and means of soliciting insurance. Aside from being granted override commissions, the
end. In addition, the existing economic conditions prevailing between the parties , acting unit manager was given production bonus, development allowance and a unit
like the inclusion of the employee in the payrolls, are also considered in determining the development financing scheme euphemistically termed "financial assistance" consisting
existence of an employer-employee relationship. of payment to him of a free portion of P300.00 per month and a validate portion of
Reasoning P1,200.00. While the latter amount was deemed as an advance against expected
- Sevilla was not subject to control by TWS either as to the result of the enterprise or as commissions, the former was not and would be freely given to the unit manager by the
to the means used in connection therewith. company only upon fulfillment by him of certain manpower and premium quota
- Under the contract of lease, Sevilla bound herself in solidum for the rental payments; an requirements. The agents and underwriters recruited and trained by the acting unit
arrangement that would belie the claims of a master-servant relationship for a true manager would be attached to the unit but petitioner reserved the right to determine if
employee cannot be made to part with his own money in pursuance of his employer’s such assignment would be made or, for any reason, to reassign them elsewhere. Aside
business, or otherwise assume liability thereof. from soliciting insurance, De los Reyes was also expressly obliged to participate in the
- Sevilla was not in the company’s payroll. She retained 4% in commissions from airline company's conservation program, i.e., preservation and maintenance of existing
bookings, the remaining 3% going to TWS. Unlike an employee who usually earns a insurance policies, and to accept moneys duly receipted on agent's receipts provided the
fixed salary, she earned compensation in fluctuating amounts depending on her booking same were turned over to the company. As long as he was unit manager in an acting
successes. capacity, De los Reyes was prohibited from working for other life insurance companies or
- The fact that Sevilla has been designated “branch manager” does not make her, ergo, with the government. He could not also accept a managerial or supervisory position in
TWS’ employee. Employment is determined by the right of control test and certain any firm doing business in the Philippines without the written consent of petitioner.
economic parameters. Titles are weak indicators. - Private respondent worked concurrently as agent and Acting Unit Manager until he was
- When Sevilla agreed to man TWS’ Ermita branch office, she did so pursuant to a notified by petitioner on 18 November 1993 that his services were terminated effective 18
contract of agency. It is the essence of this contract that the agent renders services “in December 1993. He filed a complaint before the Labor Arbiter on the ground that he was
representation or on behalf of another”. In the case at bar, Sevilla solicited airline fares, illegally dismissed and that he was not paid his salaries and separation pay.
but she did so for and on behalf of her principal, TWS.
2. YES ISSUE
Ratio For its unwarranted revocation of the contact of agency, TWS should be WON there exist an employer-employee relationship between petitioner and respondent
sentenced to pay damages.
Reasoning HELD
- Sevilla had acquired a personal stake in the business itself, and necessarily, in the YES
equipment pertaining thereto. - It is axiomatic that the existence of an employer-employee relationship cannot be
- Sevilla was not a stranger to that contract of lease having been explicitly named therein negated by expressly repudiating it in the management contract and providing therein
as third party in charge of rental payments. She could not be ousted from possession that the "employee" is an independent contractor when the terms of the agreement
summarily as one would eject an interloper. clearly show otherwise. For, the employment status of a person is defined and prescribed
- The Court is satisfied with the chronicle of events, there was indeed some malevolent by law and not by what the parties say it should be. In determining the status of the
design to put the petitioner Sevilla in a bad light following the disclosures that she had management contract, the "four-fold test" on employment earlier mentioned has to be
worked for a rival firm. applied.
Disposition REVERSED. (a) selection and engagement of employee
> Petitioner contends that De los Reyes was ever required to go through the pre-
employment procedures and that the probationary employment status was reserved only
INSULAR LIFE ASSURANCE CO LTD V NLRC (DELOS to employees of petitioner. On this score, it insists that the first requirement of selection
REYES) and engagement of the employee was not met. A look at the provisions of the contract
287 SCRA 476 shows that private respondent was appointed as Acting Unit Manager only upon
BELLOSILLO; March 12, 1998 recommendation of the District Manager. This indicates that private respondent was hired
by petitioner because of the favorable endorsement of its duly authorized officer. But, this
approbation could only have been based on the performance of De los Reyes as agent
NATURE
under the agency contract so that there can be no other conclusion arrived under this
Petition for review on certiorari of the decision of the NLRC
premise than the fact that the agency or underwriter phase of the relationship of De los
Reyes with petitioner was nothing more than a trial or probationary period for his
FACTS
eventual appointment as Acting Unit Manager of petitioner. Then, again, the very
- On 21 August 1992 petitioner entered into an agency contract with respondent
designation of the appointment of private respondent as "acting" unit manager obviously
Pantaleon de los Reyes authorizing the latter to solicit within the Philippines applications
implies a temporary employment status which may be made permanent only upon
for life insurance and annuities for which he would be paid compensation in the form of
compliance with company standards such as those enumerated under the management
commissions. The contract was prepared by petitioner in its entirety and De los Reyes
contract.
merely signed his conformity thereto. It contained the stipulation that no employer-
(b) payment of wages
employee relationship shall be created between the parties and that the agent shall be
> Petitioner points out that respondent was compensated strictly on commission basis,
free to exercise his own judgment as to time, place and means of soliciting insurance. De
the amount of which was totally dependent on his total output. But, the manager's
los Reyes however was prohibited by petitioner from working for any other life insurance
contract, speaks differently. It unquestionably demonstrate that the performance
company, and violation of this stipulation was sufficient ground for termination of the
requirement imposed on De los Reyes was applicable quarterly while his entitlement to
contract. Aside from soliciting insurance for the petitioner, private respondent was
the free portion (P300) and the validated portion (P1,200) was monthly starting on the
required to submit to the former all completed applications for insurance within ninety
first month of the twelve (12) months of the appointment. Thus, it has to be admitted that
(90) consecutive days, deliver policies, receive and collect initial premiums and balances
even before the end of the first quarter and prior to the so-called quarterly performance
of first year premiums, renewal premiums, deposits on applications and payments on
evaluation, private respondent was already entitled to be paid both the free and validated
policy loans. Private respondent was also bound to turn over to the company immediately
portions of the UDF every month because his production performance could not be
any and all sums of money collected by him.
determined until after the lapse of the quarter involved. This indicates quite clearly that
- On 1 March 1993 petitioner and private respondent entered into another contract where
the unit manager's quarterly performance had no bearing at all on his entitlement at least
the latter was appointed as Acting Unit Manager under its office atthe Cebu DSO V. As
to the free portion of the UDF which for all intents and purposes comprised the salary
such, the duties and responsibilities of De los Reyes included the recruitment, training,
regularly paid to him by petitioner. Thus it cannot be validly claimed that the financial
organization and development within his designated territory of a sufficient number of
assistance consisting of the free portion of the UDF was purely dependent on the
qualified, competent and trustworthy underwriters, and to supervise and coordinate the
premium production of the agent. Be that as it may, it is worth considering that the
sales efforts of the underwriters in the active solicitation of new business and in the
payment of compensation by way of commission does not militate against the conclusion
furtherance of the agency's assigned goals. It was similarly provided in the management
that private respondent was an employee of petitioner. Under Art. 97 of the Labor Code,
contract that the relation of the acting unit manager and/or the agents of his unit to the
company shall be that of independent contractor. If the appointment was terminated for
Labor Law 1 A2010 - 59 - Disini
"wage" shall mean "however designated, capable of being expressed in terms of money, the records shows that the latter performed his work as truck driver under the
whether fixed or ascertained on a time, task, price or commission basis . . . .". respondents’ supervision and control. Their right of control was manifested by the
(c) power of dismissal and power of control following attendant circumstances:
> petitioner asserts that its termination of De los Reyes was but an exercise of its 1. The truck driven by the petitioner belonged to respondent company;
inherent right as principal under the contracts and that the rules and guidelines it set forth 2. There was an express instruction from the respondents that the truck shall be
in the contract cannot, by any stretch of the imagination, be deemed as an exercise of used exclusively to deliver respondent company’s goods; [19]
control over the private respondent as these were merely directives that fixed the desired 3. Respondents directed the petitioner, after completion of each delivery, to park
result without dictating the means or method to be employed in attaining it. The the truck in either of two specific places only, to wit: at its office in Metro Manila at
management contract, however, prescribes reveals that the company practically dictates 2320 Osmeña Street, Makati City or at BEPZ, Mariveles, Bataan;[20] and
the manner by which their jobs are to be carried out particularly exclusivity of service, 4. Respondents determined how, where and when the petitioner would perform
control of assignments and removal of agents under private respondent's unit, collection his task by issuing to him gate passes and routing slips. [21]
of premiums, furnishing of company facilities and materials as well as capital described - These circumstances, to the Court’s mind, prove that the respondents exercised control
as Unit Development Fund. over the means and methods by which the petitioner accomplished his work as truck
- These are but hallmarks of the management system in which herein private respondent driver of the respondent company.
worked. This obtaining, there is no escaping the conclusion that private respondent - The contract of service indubitably established the existence of an employer-employee
Pantaleon de los Reyes was an employee of herein petitioner. relationship between the respondent company and the petitioner. It bears stressing
Disposition Petition denied. that the existence of an employer-employee relationship cannot be negated by
expressly repudiating it in a contract and providing therein that the employee is
an independent contractor when, as in this case, the facts clearly show
CHAVEZ V NLRC (SUPREME PACKAGING INC, LEE) otherwise. Indeed, the employment status of a person is defined and
448 SCRA 478 prescribed by law and not by what the parties say it should be. [22]
CALLEJO, SR; January 17, 2005 - The employer-employee relationship being established, the Court rules that private
respondent is guilty of illegal dismissal.
NATURE
Petition for review on certiorari of the Resolution[1] dated December 15, 2000 of the Court SAN MIGUEL CORP V ABELLA
of Appeals (CA) reversing its Decision dated April 28, 2000 finding private respondents
461 SCRA 392
guilty of illegal dismissal.
CARPIO-MORALES; June 28 2005
FACTS
- The respondent company, Supreme Packaging, Inc. engaged the services of the NATURE
petitioner, Pedro Chavez, as truck driver on October 25, 1984. The respondent company Special Civil Action in the Supreme Court. Certiorari
furnished the petitioner with a truck.
- Sometime in 1992, the petitioner expressed to respondent Alvin Lee, respondent FACTS
company’s plant manager, his (the petitioner’s) desire to avail himself of the benefits that - Petitioner San Miguel Corporation (SMC) and Sunflower Multi-Purpose Cooperative
the regular employees were receiving such as overtime pay, nightshift differential pay, (Sunflower), entered into a one-year Contract of Services commencing on January 1,
and 13th month pay, among others. Although he promised to extend these benefits to 1993, to be renewed on a month to month basis until terminated by either party. The
the petitioner, respondent Lee failed to actually do so. pertinent provisions of the contract are:
- On February 20, 1995, the petitioner filed a complaint for regularization with the 1. The cooperative agrees and undertakes to perform and/or provide for the
Regional Arbitration Branch No. III of the NLRC in San Fernando, Pampanga. Before the company, on a non-exclusive basis for a period of one year the following services for
case could be heard, respondent company terminated the services of the petitioner. the Bacolod Shrimp Processing Plant:
Consequently, on May 25, 1995, the petitioner filed an amended complaint against the A. Messengerial/Janitorial
respondents for illegal dismissal, unfair labor practice and non-payment of overtime pay, B. Shrimp Harvesting/Receiving
nightshift differential pay, 13th month pay, among others. The case was docketed as C. Sanitation/Washing/Cold Storage
NLRC Case No. RAB-III-02-6181-95. 4. There is no employer-employee relationship between the company and the
- The respondents, for their part, denied the existence of an employer-employee cooperative, or the cooperative and any of its members, or the company and any
relationship between the respondent company and the petitioner. They averred that the members of the cooperative. The cooperative is an association of self-employed
petitioner was an independent contractor as evidenced by the contract of service which members, an independent contractor, and an entrepreneur. It is subject to the control
he and the respondent company entered into. The relationship of the respondent and direction of the company only as to the result to be accomplished by the work or
company and the petitioner was allegedly governed by this contract of service. services herein specified, and not as to the work herein contracted. The cooperative
- The respondents insisted that the petitioner had the sole control over the means and and its members recognize that it is taking a business risk in accepting a fixed service
methods by which his work was accomplished. He paid the wages of his helpers and fee to provide the services contracted for and its realization of profit or loss from its
exercised control over them. As such, the petitioner was not entitled to regularization undertaking, in relation to all its other undertakings, will depend on how efficiently it
because he was not an employee of the respondent company. The respondents, deploys and fields its members and how they perform the work and manage its
likewise, maintained that they did not dismiss the petitioner. Rather, the severance of his operations.
contractual relation with the respondent company was due to his violation of the terms - Pursuant to the contract, Sunflower engaged private respondents to, as they did, render
and conditions of their contract. services at SMC’s Bacolod Shrimp Processing Plant at Sta. Fe, Bacolod City. The
contract was deemed renewed by the parties every month after its expiration on January
1, 1994 and private respondents continued to perform their tasks until September 11,
ISSUE 1995. In July 1995, private respondents filed a complaint before the NLRC, Regional
WON there existed an employer-employee relationship between the respondent Arbitration Branch No. VI, Bacolod City, praying to be declared as regular employees of
company and the petitioner. SMC, with claims for recovery of all benefits and privileges enjoyed by SMC rank and file
employees. Private respondents subsequently filed on September 25, 1995 an Amended
HELD Complaint to include illegal dismissal as additional cause of action following SMC’s
YES closure of its Bacolod Shrimp Processing Plant on September 15, 1995which resulted in
- The elements to determine the existence of an employment relationship are: (1) the the termination of their services. SMC filed a Motion for Leave to File Attached Third
selection and engagement of the employee; (2) the payment of wages; (3) the power of Party Complaint dated November 27, 1995 to implead Sunflower as Third Party
dismissal; and (4) the employer’s power to control the employee’s conduct. [11] The most Defendant which was, by Order of December 11, 1995, granted by Labor Arbiter Ray
important element is the employer’s control of the employee’s conduct, not only as to the Alan T. Drilon. In the meantime, on September 30, 1996, SMC filed before the Regional
result of the work to be done, but also as to the means and methods to accomplish it. [12] Office at Iloilo City of the Department of Labor and Employment (DOLE) a Notice of
All the four elements are present in this case. Closure of its aquaculture operations effective on even date, citing serious business
- Of the four elements of the employer-employee relationship, the “control test” is the losses. By Decision of September 23, 1997, Labor Arbiter Drilon dismissed private
most important. Although the respondents denied that they exercised control over the respondents’ complaint for lack of merit.
manner and methods by which the petitioner accomplished his work, a careful review of
Labor Law 1 A2010 - 60 - Disini
- Private respondents appealed to the NLRC. By Decision of December 29, 1998, the resulting in the closure of its Calatrava Aquaculture Center in Negros Occidental,
NLRC dismissed the appeal for lack of merit, it finding that third party respondent P11,393,071.00 in 1993 and P80,325,608.00 in 1994 which led to the closure of its San
Sunflower was an independent contractor in light of its observation that “[i]n all the Fernando Shrimp Processing Plant in Pampanga and the Bacolod Shrimp Processing
activities of private respondents, they were under the actual direction, control and Plant in 1995. For termination due to retrenchment to be valid, however, the law requires
supervision of third party respondent Sunflower, as well as the payment of wages, and that written notices of the intended retrenchment be served by the employer on the
power of dismissal. By Decision of February 7, 2001, the appellate court reversed the worker and on the DOLE at least one (1) month before the actual date of the
NLRC decision and accordingly found for private respondents. Justifying its reversal of retrenchment in order to give employees some time to prepare for the eventual loss of
the findings of the labor arbiter and the NLRC, the appellate court reasoned:Although the their jobs, as well as to give DOLE the opportunity to ascertain the verity of the alleged
terms of the non-exclusive contract of service between SMC and [Sunflower] showed a cause of termination. Private respondents, however, were merely verbally
clear intent to abstain from establishing an employer-employee relationship between informed on September 10, 1995 by SMC Prawn Manager Ponciano Capay that
SMC and [Sunflower] or the latter’s members, the extent to which the parties successfully effective the following day or on September 11, 1995, they were no longer to
realized this intent in the light of the applicable law is the controlling factor in determining report for work as SMC would be closing its operations. Where the dismissal is
the real and actual relationship between or among the parties.There being a finding of based on an authorized cause under Article 283 of the Labor Code but the
“labor-only” contracting, liability must be shouldered either by SMC or [Sunflower] or employer failed to comply with the notice requirement, the sanction should be
shared by both (See Tabas vs. California Manufacturing, Inc ., supra, p. 502). SMC stiff as the dismissal process was initiated by the employer’s exercise of his
however should be held solely liable for [Sunflower] became non-existent with the management prerogative, as opposed to a dismissal based on a just cause
closure of the aquaculture business of SMC. under Article 282 with the same procedural infirmity where the sanction to be
imposed upon the employer should be tempered as the dismissal process was,
ISSUE in effect, initiated by an act imputable to the employee. In light of the factual
1. WON the respondents are employees of SMC circumstances of the case at bar, the Court awards P50,000.00 to each private
2. WON the retrenchment was valid and consequently, whether the respondents are respondent as nominal damages.The grant of separation pay as an incidence of
entitled to relief termination of employment due to retrenchment to prevent losses is a statutory obligation
on the part of the employer and a demandable right on the part of the employee. Private
HELD respondents should thus be awarded separation pay equivalent to at least one (1) month
1. YES pay or to at least one-half month pay for every year of service, whichever is higher, as
- Since private respondents who were engaged in shrimp processing performed tasks mandated by Article 283 of the Labor Code or the separation pay awarded by SMC to
usually necessary or desirable in the aquaculture business of SMC, they should be other regular SMC employees that were terminated as a result of the retrenchment,
deemed regular employees of the latter and as such are entitled to all the benefits and depending on which is most beneficial to private respondents.Considering that private
rights appurtenant to regular employment. They should thus be awarded differential pay respondents were not illegally dismissed, however, no backwages need be awarded. It
corresponding to the difference between the wages and benefits given them and those is well settled that backwages may be granted only when there is a finding of illegal
accorded SMC’s other regular employees. Respecting the private respondents who dismissal.[80] The appellate court thus erred in awarding backwages to private
were tasked with janitorial and messengerial duties, this Court quotes with approval the respondents. What was involved in that case was one of illegal dismissal
appellate court’s ruling thereon:
- Those performing janitorial and messengerial services however acquired regular status
LOPEZ V METROPOLITAN WATERWORKS AND
only after rendering one-year service pursuant to Article 280 of the Labor Code. Although
janitorial and messengerial services are considered directly related to the aquaculture SEWERAGE SYSTEM
business of SMC, they are deemed unnecessary in the conduct of its principal business; 462 SCRA 428
hence, the distinctionThe law of course provides for two kinds of regular employees, TINGA; June 30, 2005
namely: (1) those who are engaged to perform activities which are usually necessary or
desirable in the usual business or trade of the employer; and (2) those who have rendered
NATURE
at least one year of service, whether continuous or broken, with respect to the activity in
Petition for the review of the decision of the CA
which they are employed.
- The test to determine the existence of independent contractorship is whether one
FACTS
claiming to be an independent contractor has contracted to do the work according to his
- By virtue of an Agreement, petitioners were engaged by the MWSS as collectors-
own methods and without being subject to the control of the employer, except only as to
contractors, wherein the former agreed to collect from the concessionaires of MWSS,
the results of the work. As for those of private respondents who were engaged in
charges, fees, assessments of rents for water, sewer and/or plumbing services which the
janitorial and messengerial tasks, they fall under the second category and are thus
MWSS bills from time to time.
entitled to differential pay and benefits extended to other SMC regular employees from
- In 1997, MWSS entered into a Concession Agreement with Manila Water Service, Inc.
the day immediately following their first year of service.
and Benpress-Lyonnaise, wherein the collection of bills was transferred to said private
- In legitimate labor contracting, the law creates an employer-employee relationship for a
concessionaires, effectively terminating the contracts of service between petitioners and
limited purpose, i.e., to ensure that the employees are paid their wages. The principal
MWSS.
employer becomes jointly and severally liable with the job contractor, only for the
- Regular employees of the MWSS were paid their retirement benefits, but not
payment of the employees’ wages whenever the contractor fails to pay the same. Other
petitioners. Instead, they were refused said benefits, MWSS relying on a resolution of
than that, the principal employer is not responsible for any claim made by the employees.
[50] the CSC that contract-collectors of the MWSS are not its employees and therefore not
entitled to the benefits due regular government employees.
- In labor-only contracting, the statute creates an employer-employee relationship for a
- Petitioners filed a complaint with the CSC which denied their claims, stating that
comprehensive purpose: to prevent a circumvention of labor laws. The contractor is
petitioners were engaged by MWSS through a contract of service, which explicitly
considered merely an agent of the principal employer and the latter is responsible to the
provides that a bill collector-contractor is not an MWSS employee. Relying on Part V of
employees of the labor-only contractor as if such employees had been directly employed
CSC Memorandum Circular No. 38, Series of 1993, the CSC stated that contract
by the principal employer.[51]
services/job orders are not considered government services, which do not have to be
- The Contract of Services between SMC and Sunflower shows that the parties clearly
submitted to the CSC for approval, unlike contractual and plantilla appointments.
disavowed the existence of an employer-employee relationship between SMC and
Moreover, it found that petitioners were unable to show that they have contractual
private respondents. The language of a contract is not, however, determinative of the
appointments duly attested by the CSC. In addition, the CSC stated that petitioners, not
parties’ relationship; rather it is the totality of the facts and surrounding circumstances of
being permanent employees of MWSS and not included in the list submitted to the
the case.[52] A party cannot dictate, by the mere expedient of a unilateral declaration in a
concessionaire, are not entitled to severance pay. Petitioners’ claims for retirement
contract, the character of its business, i.e., whether as labor-only contractor or job
benefits and terminal leave pay were likewise denied.
contractor, it being crucial that its character be measured in terms of and determined by
- Petitioners sought reconsideration of the CSC Resolution, which was however denied
the criteria set by statute
- Petitioners filed a petition for review with the Court of Appeals which affirmed the ruling
2. SMC has thus proven substantial business reverses justifying retrenchment of its
of the CSC.
employees.
- In the case at bar, company losses were duly established by financial documents
ISSUE
audited by Joaquin Cunanan & Co. showing that the aquaculture operations of SMC’s
WON petitioners were employees of the MWSS and, consequently, entitled to the
Agribusiness Division accumulated losses amounting to P145,848,172.00 in 1992
benefits they claim
Labor Law 1 A2010 - 61 - Disini
FACTS
HELD - Private respondent Rosalina M. Laudato filed a petition before the SSC for social
YES security coverage and remittance of unpaid monthly social security contributions against
- The Court has invariably affirmed that it will not hesitate to tilt the scales of justice to the her three employers. Among the respondents was herein petitioner Angelito L. Lazaro,
labor class for no less than the Constitution dictates that “the State . . . shall protect the proprietor of Royal Star Marketing, which is engaged in the business of selling home
rights of workers and promote their welfare.” It is committed to this policy and has always appliances. Laudato alleged that despite her employment as sales supervisor of the
been quick to rise to defense in the rights of labor, as in this case. sales agents for Royal Star from April of 1979 to March of 1986, Lazaro had failed during
- Protection to labor, it has been said, extends to all of laborlocal and overseas, the said period, to report her to the SSC for compulsory coverage or remit Laudato's
organized and unorganized, in the public and private sectors. [52] Besides, there is no social security contributions.
reason not to apply this principle in favor of workers in the government. The government, - Lazaro denied that Laudato was a sales supervisor of Royal Star, averring instead that
including government-owned and controlled corporations, as employers, should set the she was a mere sales agent whom he paid purely on commission basis. Lazaro also
example in upholding the rights and interests of the working class. maintained that Laudato was not subjected to definite hours and conditions of work. As
- For purposes of determining the existence of employer-employee relationship, the such, Laudato could not be deemed an employee of Royal Star.
Court has consistently adhered to the four-fold test, namely: (1) whether the alleged - SSC ruled in favor of Laudato. Applying the "control test," it held that Laudato was an
employer has the power of selection and engagement of an employee; (2) whether he employee of Royal Star, and ordered Royal Star to pay the unremitted social security
has control of the employee with respect to the means and methods by which work is to contributions of Laudato in the amount of P5,007.35, together with the penalties totaling
be accomplished; (3) whether he has the power to dismiss; and (4) whether the P22,218.54. In addition, Royal Star was made liable to pay damages to the SSC in the
employee was paid wages. Of the four, the control test is the most important element. amount of P15,680.07 for not reporting Laudato for social security coverage, pursuant to
- A review of the circumstances surrounding the case reveals that petitioners are Section 24 of the Social Security Law. Lazaro's MR was denied, prompting him to file a
employees of MWSS. MWSS wielded its power of selection when it contracted with the petition for review with the CA. However, the CA affirmed the finding that Laudato was an
individual petitioners, undertaking separate contracts or agreements. The same goes employee of Royal Star, and hence entitled to coverage under the Social Security Law.
true for the power to dismiss. Although termed as causes for termination of the - Lazaro's Argument : that Laudato was not qualified for social security coverage, as
Agreement, a review of the same shows that the grounds indicated therein can similarly she was not an employee of Royal Star, her income dependent on a generation of sales
be grounds for termination of employment. and based on commissions; that Royal Star had no control over Laudato's activities, and
- On the issue of remuneration, MWSS claims that the compensation received by that under the so-called "control test," Laudato could not be deemed an employee.
petitioners does not fall under the definition of wages as provided in Section 2(i) of P.D.
1146. This assertion, however, simply begs the question. The provision is a simple ISSUE
statement of meaning, operating on the a priori premise or presumption that the recipient WON Laudato is an employee of Royal Star
is already classified as an employee, and does not lay down any basis or standard for
determining who are employees and who are not. HELD
- On the other hand, relevant and appropriate is the definition of wages in the Labor YES
Code, namely, that it is the remuneration, however designated, for work done or to be Doctrine For the purposes of coverage under the Social Security Act, the determination
done, or for services rendered or to be rendered. The “commissions” due petitioners of employer-employee relationship warrants the application of the " control test ," that is,
were based on the bills collected as per the schedule indicated in the Agreement. whether the employer controls or has reserved the right to control the employee, not only
Significantly, MWSS granted petitioners benefits usually given to employees, to wit: as to the result of the work done, but also as to the means and methods by which the
COLA, meal, emergency, and traveling allowances, hazard pay, cash gift, and other same is accomplished.
bonuses. Petitioners rendered services to MWSS for which they were paid and given - The fact that Laudato was paid by way of commission does not preclude the
similar benefits due the other employees of MWSS. establishment of an employer-employee relationship. In Grepalife v. Judico, the Court
- Now the aspect of control. MWSS makes an issue out of the proviso in the Agreement upheld the existence of an employer-employee relationship between the insurance
that specifically denies the existence of employer-employee relationship between it and company and its agents, despite the fact that the compensation that the agents on
petitioners. It is axiomatic that the existence of an employer-employee relationship commission received was not paid by the company but by the investor or the person
cannot be negated by expressly repudiating it in an agreement and providing therein that insured.
the employee is “not an MWSS employee” when the terms of the agreement and the - Neither does it follow that a person who does not observe normal hours of work cannot
surrounding circumstances show otherwise. The employment status of a person is be deemed an employee. In Cosmopolitan Funeral Homes, Inc. v. Maalat, the Supreme
defined and prescribed by law and not by what the parties say it should be. Court declared that there was an employer-employee relationship, noting that "[the]
- In addition, the control test merely calls for the existence of the right to control, and not supervisor, although compensated on commission basis, [is] exempt from the
the exercise thereof. It is not essential for the employer to actually supervise the observance of normal hours of work for his compensation is measured by the number of
performance of duties of the employee, it is enough that the former has a right to wield sales he makes.
the power. - The determination of an employer-employee relationship depends heavily on the
- Other manifestations of control are evident from the records. The power to transfer or particular factual circumstances attending the professional interaction of the parties. SC
reassign employees is a management prerogative exclusively enjoyed by employers. In sees no reversible error in the findings of fact of the courts below. Both SSC and CA
this case, MWSS had free reign over the transfer of bill collectors from one branch to found that Laudato was a sales supervisor and not a mere agent. As such, Laudato
another. MWSS also monitored the performance of the petitioners and determined their oversaw and supervised the sales agents of the company, and thus was subject to the
efficiency ratings. control of management as to how she implements its policies and its end results. This is
Disposition Petition was GRANTED IN PART. The Decision of the Court of Appeals in proven by several documentary evidence.
C.A.–G.R. SP No. 55263, as well as the Civil Service Commission’s Resolutions Nos. Disposition Petition is DENIED. CA Decision AFFIRMED. Costs against petitioner.
991384 and 992074, were REVERSED and SET ASIDE. MWSS is ordered to pay
terminal leave pay and separation pay and/or severance pay to each of herein petitioners
on the basis of remunerations/commissions, allowances and bonuses each were actually
receiving at the time of termination of their employment as contract collectors of MWSS. ALMIREZ V INFINITE LOOP TECHNOLOGY
The case was remanded to the Civil Service Commission for the computation of the CORPORATION
above awards and the appropriate disposition in accordance with the pronouncements in [PAGE 57]
this Decision.
LAZARO V SSS (LAUDATO)
LAZARO V SSS (LAUDATO) [PAGE 61]
435 SCRA 472
TINGA; July 30, 2004 DOMASIG V NLRC (CATA GARMENTS)
261 SCRA 779 (96)
NATURE
Petition for Review under ROC Rule 45, assailing the CA Decision, which affirmed two PADILLA; September 16, 1996
rulings of the Social Security Commission (SSC)
NATURE
Labor Law 1 A2010 - 62 - Disini
Petition for certiorari under Rule 65 of the Rules of Court to nullify and set aside the respondent corporation, he was also tasked to collect payments from his various
Resolution of respondent National Labor Relations Commission remanding the records customers. Sometime in 1998, petitioner encountered five customers/clients with bad
of the case to the arbitration branch of origin for further proceedings. accounts.
- Petitioner was confronted by respondent Lamadrid over the bad accounts and warned
that if he does not issue his own checks to cover the said bad accounts, his commissions
will not be released and he will lose his job. Not contented with the issuance of the
foregoing checks as security for the bad accounts, respondents "tricked" petitioner into
FACTS signing two documents, which he later discovered to be a Promissory Note and a Deed
- Complaint was instituted by Eddie Domasig against respondents Cata Garments of Real Estate Mortgage.
Corporation, a company engaged in garments business and its owner/manager Otto Ong - Due to financial difficulties, petitioner inquired about his membership with the SSS in
and Catalina Co for illegal dismissal, unpaid commission and other monetary claim(s). order to apply for a salary loan. To his dismay, he learned that he was not covered by the
- Complainant alleged that he started working with the respondent on July 6, 1986 as SSS and therefore was not entitled to any benefit. While doing his usual rounds as
Salesman; three (3) years ago, because of a complaint against respondent by its commission salesman, petitioner was handed by his customers a letter from the
workers, the company changed its name to Cata Garments Corporation; and that on respondent company warning them not to deal with petitioner since it no longer
August 29, 1992, he was dismissed when respondent learned that he was being pirated recognized him as a commission salesman. Petitioner thus filed a complaint for illegal
by a rival corporation which offer he refused. dismissal with money claims against respondent company and its president, Jose
- The Labor Arbiter held that complainant was illegally dismissed and entitled to Lamadrid, before the NLRC.
reinstatement and backwages as well as underpayment of salary; 13th month pay;
service incentive leave and legal holiday. The Arbiter also awarded complainant his ISSUE
claim for unpaid commission in the amount of P143,955.00. 1. WON an employer-employee relationship exists between plaintiff and respondent
- NLRC remanded the case for further proceedings. company
- Petitioner’s Claim 2. WON respondent intimidated and tricked plaintiff into providing security for the bad
> Petitioner claims he was an employee, and that he was illegally dismissed. accounts
- Respondent’s Comments HELD
> Respondents claim that Domasig was a mere commission worker, and not a regular 1. NO
employee (which would warrant backwages). Ratio To ascertain the existence of an employer-employee relationship, jurisprudence
has invariably applied the four-fold test, namely: (1) the manner of selection and
ISSUE engagement; (2) the payment of wages; (3) the presence or absence of the power of
WON Domasig is a regular employee (this case is under the topic of proof of dismissal; and (4) the presence or absence of the power of control. Of these four, the last
employment) one is the most important. Under the control test, an employer-employee relationship
exists where the person for whom the services are performed reserves the right to
HELD control not only the end achieved, but also the manner and means to be used in reaching
YES, Domasig is a regular employee. that end. Where a person who works for another does so more or less at his own
Ratio Substantial evidence is sufficient as a basis for judgment on the existence of pleasure and is not subject to definite hours or conditions of work, and in turn is
employer-employee relationship. compensated according to the result of his efforts and not the amount thereof, no
Reasoning relationship of employer-employee exists.
- Proof beyond reasonable doubt is not required as a basis for judgment on the legality of Reasoning
an employer’s dismissal of an employee, nor even preponderance of evidence for that - Petitioner Abante was a commission salesman who received 3% commission of his
matter, substantial evidence being sufficient. Any competent and relevant evidence to gross sales. No quota was imposed on him by the respondent. He was not required to
prove the relationship may be admitted. report to the office at any time or submit any periodic written report on his sales
- Substantial evidence performance and activities. He was not designated by respondent to conduct his sales
> relevant evidence as a reasonable mind might accept as adequate to support a activities at any particular or specific place. He pursued his selling activities without
conclusion, and its absence is not shown by stressing that there is contrary evidence on interference or supervision from respondent company and relied on his own resources to
record, direct or circumstantial, for the appellate court cannot substitute its own judgment perform his functions. Respondent company did not prescribe the manner of selling the
or criterion for that of the trial court in determining wherein lies the weight of evidence or merchandise; he was left alone to adopt any style or strategy to entice his customers.
what evidence is entitled to belief. Moreover, petitioner was free to offer his services to other companies engaged in similar
> In a business establishment, an identification card is usually provided not only as a or related marketing activities as evidenced by the certifications issued by various
security measure but mainly to identify the holder thereof as a bona fide employee of the customers.
firm that issues it. Together with the cash vouchers covering petitioner’s salaries for the 2. NO
months stated therein, these matters constitute substantial evidence adequate to support Ratio While petitioner may have been coerced into executing force to issue the said
a conclusion that petitioner was indeed an employee of private respondent. documents, it may equally be true that petitioner did so in recognition of a valid financial
> The list presented by private respondents would even support petitioner’s allegation obligation. He who claims that force or intimidation was employed upon him lies the onus
that, aside from a monthly salary of P1,500.00, he also received commissions for his probandi. He who asserts must prove.
work as a salesman of private respondents. Disposition The decision of the CA is AFFIRMED in toto.
- Having been in the employ of private respondents continuously for more than one year,
under the law, petitioner is considered a regular employee.
R TRANSPORT CORP V EJANDRA
Disposition The decision of the labor arbiter dated 19 May 1993 is REINSTATED and
AFFIRMED. [PAGE 55]

ABANTE V LAMADRID MANILA ELECTRIC COMPANY V QUISUMBING


430 SCRA 368 [PAGE 19]
YNARES-SANTIAGO; May 28, 2004
MANILA ELECTRIC CO V BENAMIRA
NATURE 302 SCRA 173
Petition for review assailing the Decision of the CA which affirmed the Resolution of the AUSTRIA-MARTINEZ; July 14, 2005
NLRC
NATURE
FACTS
Petition for review on certiorari of the Court of Appeals decision
- Petitioner was employed by respondent company Lamadrid Bearing and Parts
Corporation sometime in June 1985 as a salesman covering the whole area of
FACTS
Mindanao. His average monthly income was more or less P16,000.00, but later was
increased to approximately P20,269.50. Aside from selling the merchandise of
Labor Law 1 A2010 - 63 - Disini
- The individual respondents are licensed security guards formerly employed by People’s of tenure of individual respondents; individual respondents are regular employees of
Security, Inc. (PSI) and deployed as such at MERALCO’s head office in Ortigas Avenue, MERALCO since their services as security guards are usually necessary or desirable in
Pasig, Metro Manila. On November 30, 1990, the security service agreement between the usual business or trade of MERALCO and they have been in the service of
PSI and MERALCO was terminated. Immediately thereafter, fifty-six of PSI’s security MERALCO for no less than six years; an employer-employee relationship exists between
guards, including herein eight individual respondents, filed a complaint for unpaid MERALCO and the individual respondents because: (a) MERALCO had the final say in
monetary benefits against PSI and MERALCO. Meanwhile, the security service the selection and hiring of the guards, as when its advice was proved to have carried
agreement between respondent Armed Security & Detective Agency, Inc., (ASDAI) and weight in AFSISI’s decision not to absorb the individual respondents into its workforce;
MERALCO took effect on December 1, 1990. In the agreement, ASDAI was designated (b) MERALCO paid the wages of individual respondents through ASDAI and AFSISI; (c)
as the AGENCY while MERALCO was designated as the COMPANY. MERALCO’s discretion on matters of dismissal of guards was given great weight and
- Subsequently, the individual respondents were absorbed by ASDAI and retained at even finality since the record shows that the individual respondents were replaced upon
MERALCO’s head office. the advice of MERALCO; and, (d) MERALCO has the right, at any time, to inspect the
- Asuncion rendered a decision in NLRC-NCR Case No. 05-02746-90 in favor of the guards, to require without explanation the replacement of any guard whose behavior,
former PSI security guards, including the individual respondents. conduct or appearance is not satisfactory and ASDAI and AFSISI cannot pull out any
- Less than a month later, or on July 21, 1992, the individual respondents filed another security guard from MERALCO without the latter’s consent; and, a labor-only contract
complaint for unpaid monetary benefits, this time against ASDAI and MERALCO. existed between ASDAI and AFSISI and MERALCO, such that MERALCO is guilty of
- On July 25, 1992, the security service agreement between respondent Advance Forces illegal dismissal without just cause and liable for reinstatement of individual respondents
Security & Investigation Services, Inc. (AFSISI) and MERALCO took effect, terminating to its workforce.
the previous security service agreement with ASDAI. Except as to the number of security
guards, the amount to be paid the agency, and the effectivity of the agreement, the terms ISSUES
and conditions were substantially identical with the security service agreement with 1. WON there existed an employer-employee relationship
ASDAI. 2. WON individual respondents cannot be considered as regular employees as the
- The individual respondents amended their complaint to implead AFSISI as party duties performed by them as security guards are not necessary in the conduct of
respondent. They again amended their complaint to allege that AFSISI terminated their MERALCO’s principal business which is the distribution of electricity.
services on August 6, 1992 without notice and just cause and therefore guilty of illegal 3. WON MERALCO has a liability over the dismissed guards
dismissal.
- The individual respondents alleged that: MERALCO and ASDAI never paid their HELD
overtime pay, service incentive leave pay, premium pay for Sundays and Holidays, 1. It is a settled rule that in the exercise of the Supreme Court’s power of review, the
P50.00 monthly uniform allowance and underpaid their 13th month pay; on July 24, Court is not a trier of facts and does not normally undertake the re-examination of the
1992, when the security service agreement of ASDAI was terminated and AFSISI took evidence presented by the contending parties during the trial of the case considering that
over the security functions of the former on July 25, 1992, respondent security guard the findings of facts of the CA are conclusive and binding on the Court. However,
Benamira was no longer given any work assignment when AFSISI learned that the jurisprudence has recognized several exceptions in which factual issues may be resolved
former has a pending case against PSI, in effect, dismissing him from the service without by this Court.
just cause; and, the rest of the individual respondents were absorbed by AFSISI but were - In the present case, the existence of an employer-employee relationship is a question
not given any assignments, thereby dismissing them from the service without just cause. of fact which is well within the province of the CA. Nonetheless, given the reality that the
- ASDAI denied in general terms any liability for the claims of the individual respondents, CA’s findings are at odds to those of the NLRC, the Court is constrained to look deeper
claiming that there is nothing due them in connection with their services. into the attendant circumstances obtaining in the present case, as appearing on record.
- On the other hand, MERALCO denied liability on the ground of lack of employer- The individual respondents never alleged in their complaint in the Labor Arbiter, in their
employee relationship with individual respondents. It averred that the individual appeal in the NLRC and even in their petition for certiorari in the CA that MERALCO was
respondents are the employees of the security agencies it contracted for security their employer. They have always advanced the theory that AFSISI is their employer. A
services; and that it has no existing liability for the individual respondents’ claims since perusal of the records shows it was only in their Memorandum in the CA that this thesis
said security agencies have been fully paid for their services per their respective security was presented and discussed for the first time. We cannot ignore the fact that this
service agreement. position of individual respondents runs contrary to their earlier submission in their
- For its part, AFSISI asserted that: it is not liable for illegal dismissal since it did not pleadings filed in the Labor Arbiter, NLRC and even in the petition for certiorari in the CA
absorb or hire the individual respondents, the latter were merely hold-over guards from that AFSISI is their employer and liable for their termination. As the object of the
ASDAI; it is not obliged to employ or absorb the security guards of the agency it replaced pleadings is to draw the lines of battle, so to speak, between the litigants and to indicate
since there is no provision in its security service agreement with MERALCO or in law fairly the nature of the claims or defenses of both parties, a party cannot subsequently
requiring it to absorb and hire the guards of ASDAI as it has its own guards duly trained take a position contrary to, or inconsistent, with his pleadings.
to service its various clients. Moreover, it is a fundamental rule of procedure that higher courts are precluded from
- After the submission of their respective evidence and position papers, Labor Arbiter entertaining matters neither alleged in the pleadings nor raised during the proceedings
Pablo C. Espiritu, Jr. rendered a Decision holding ASDAI and MERALCO jointly and below, but ventilated for the first time only in a motion for reconsideration or on appeal.
solidarily liable to the monetary claims of individual respondents and dismissing the The individual respondents are bound by their submissions that AFSISI is their employer
complaint against AFSISI. Individual respondents’ partial appeal assailed solely the and they should not be permitted to change their theory. Such a change of theory cannot
Labor Arbiter’s declaration that ASDAI is their employer. They insisted that AFSISI is the be tolerated on appeal, not due to the strict application of procedural rules but as a
party liable for their illegal dismissal and should be the party directed to reinstate them. matter of fairness. A change of theory on appeal is objectionable because it is contrary
For its part, MERALCO attributed grave abuse of discretion on the part of the Labor to the rules of fair play, justice and due process.
Arbiter in failing to consider the absence of employer-employee relationship between - Thus, the CA should not have considered the new theory offered by the individual
MERALCO and individual respondents. respondents in their memorandum.
- On the other hand, ASDAI took exception from the Labor Arbiter’s finding that it is the - The present petition for review on certiorari is far from novel and, in fact, not without
employer of the individual respondents and therefore liable for the latter’s unpaid precedence. We have ruled in Social Security System vs. Court of Appeals that:
monetary benefits. ...The guards or watchmen render their services to private respondent by allowing
- The NLRC affirmed in toto the decision of the Labor Arbiter. The individual respondents themselves to be assigned by said respondent, which furnishes them arms and
filed a motion for partial reconsideration but it was denied by the NLRC. ammunition, to guard and protect the properties and interests of private respondent's
The individual respondents filed a petition for certiorari before the SC. They insisted that clients, thus enabling that respondent to fulfill its contractual obligations. Who the
they were absorbed by AFSISI and the latter effected their termination without notice and clients will be, and under what terms and conditions the services will be rendered, are
just cause. matters determined not by the guards or watchmen, but by private respondent. On
- After the submission of the responsive pleadings and memoranda, we referred the the other hand, the client companies have no hand in selecting who among the
petition, in accordance with St. Martin Funeral Homes vs. NLRC,[15] to the CA which, on guards or watchmen shall be assigned to them. It is private respondent that issues
September 27, 2000, modified the decision of the NLRC by declaring MERALCO as the assignment orders and instructions and exercises control and supervision over the
direct employer of the individual respondents. guards or watchmen, so much so that if, for one reason or another, the client is
- The CA held that: MERALCO changed the security agency manning its premises three dissatisfied with the services of a particular guard, the client cannot himself terminate
times while engaging the services of the same people, the individual respondents; the services of such guard, but has to notify private respondent, which either
MERALCO employed a scheme of hiring guards through an agency and periodically substitutes him with another or metes out to him disciplinary measures. That in the
entering into service contract with one agency after another in order to evade the security course of a watchman's assignment the client conceivably issues instructions to him,
Labor Law 1 A2010 - 64 - Disini
does not in the least detract from the fact that private respondent is the employer of agreement of MERALCO with ASDAI. As for the rest of the individual respondents, they
said watchman, for in legal contemplation such instructions carry no more weight than retained their post only as “hold-over” guards until the security guards of AFSISI took
mere requests, the privity of contract being between the client and private respondent, over their post on August 6, 1992.
not between the client and the guard or watchman. Corollarily, such giving out of - In the present case, respondent Benamira has been “off-detail” for seventeen days
instructions inevitably spring from the client's right predicated on the contract for while the rest of the individual respondents have only been “off- detail” for five days when
services entered into by it with private respondent. they amended their complaint on August 11, 1992 to include the charge of illegal
- In the matter of compensation, there can be no question at all that the guards or dismissal. The inclusion of the charge of illegal dismissal then was premature.
watchmen receive compensation from private respondent and not from the companies or Nonetheless, bearing in mind that ASDAI simply stopped giving the individual
establishments whose premises they are guarding. The fee contracted for to be paid by respondents any assignment and their inactivity clearly persisted beyond the six-month
the client is admittedly not equal to the salary of a guard or watchman; such fee is arrived period allowed by Article 286 of the Labor Code, the individual respondents were, in
at independently of the salary to which the guard or watchman is entitled under his effect, constructively dismissed by ASDAI from employment, hence, they should be
arrangements with private respondent. reinstated.
- Neither does the petitioner have any power to dismiss the security guards. In fact, We 3. YES, as an indirect employer.
fail to see any evidence in the record that it wielded such a power. It is true that it may - The fact that there is no actual and direct employer-employee relationship between
request the agency to change a particular guard. But this, precisely, is proof that the MERALCO and the individual respondents does not exonerate MERALCO from liability
power lies in the hands of the agency. as to the monetary claims of the individual respondents. When MERALCO contracted for
- Since the petitioner has to deal with the agency, and not the individual watchmen, on security services with ASDAI as the security agency that hired individual respondents to
matters pertaining to the contracted task, it stands to reason that the petitioner does not work as guards for it, MERALCO became an indirect employer of individual respondents
exercise any power over the watchmen's conduct. Always, the agency stands between pursuant to Article 107 of the Labor Code, which reads:
the petitioner and the watchmen; and it is the agency that is answerable to the petitioner ART. 107. Indirect employer - The provisions of the immediately preceding Article
for the conduct of its guards. shall likewise apply to any person, partnership, association or corporation which, not
- In this case, the terms and conditions embodied in the security service agreement being an employer, contracts with an independent contractor for the performance of
between MERALCO and ASDAI expressly recognized ASDAI as the employer of any work, task, job or project.
individual respondents. - When ASDAI as contractor failed to pay the individual respondents, MERALCO as
- Under the security service agreement, it was ASDAI which (a) selected, engaged or principal becomes jointly and severally liable for the individual respondents’ wages, under
hired and discharged the security guards; (b) assigned them to MERALCO according to Articles 106 and 109 of the Labor Code, which provide:
the number agreed upon; (c) provided the uniform, firearms and ammunition, nightsticks, ART. 106. Contractor or subcontractor. - Whenever an employer enters into a
flashlights, raincoats and other paraphernalia of the security guards; (d) paid them contract with another person for the performance of the former[‘s] work, the
salaries or wages; and, (e) disciplined and supervised them or principally controlled their employees of the contractor and of the latter[‘s] subcontractor, if any, shall be paid in
conduct. The agreement even explicitly provided that “[n]othing herein contained shall accordance with the provisions of this Code.
be understood to make the security guards under this Agreement, employees of the In the event that the contractor or subcontractor fails to pay the wages of his
COMPANY, it being clearly understood that such security guards shall be considered as employees in accordance with this Code, the employer shall be jointly and severally
they are, employees of the AGENCY alone.” Clearly, the individual respondents are the liable with his contractor or subcontractor to such employees to the extent of the work
employees of ASDAI. performed under the contract, in the same manner and extent that he is liable to
- Needless to stress, for the power of control to be present, the person for whom the employees directly employed by him.
services are rendered must reserve the right to direct not only the end to be achieved but ART. 109. Solidary liability - The provisions of existing laws to the contrary
also the means for reaching such end. Not all rules imposed by the hiring party on the notwithstanding, every employer or indirect employer shall be held responsible with
hired party indicate that the latter is an employee of the former. Rules which serve as his contractor or subcontractor for any violation of any provision of this Code. For
general guidelines towards the achievement of the mutually desired result are not purpose of determining the extent of their civil liability under this Chapter, they shall be
indicative of the power of control. considered as direct employers.
- Verily, the security service agreements in the present case provided that all specific - ASDAI is held liable by virtue of its status as direct employer, while MERALCO is
instructions by MERALCO relating to the discharge by the security guards of their duties deemed the indirect employer of the individual respondents for the purpose of paying
shall be directed to the agency and not directly to the individual respondents. The their wages in the event of failure of ASDAI to pay them. This statutory scheme gives
individual respondents failed to show that the rules of MERALCO controlled their the workers the ample protection consonant with labor and social justice provisions of
performance. the 1987 Constitution.
- Moreover, ASDAI and AFSISI are not “labor-only” contractors. There is “labor only” - However, as held in Mariveles Shipyard Corp. vs. Court of Appeals, the solidary liability
contract when the person acting as contractor is considered merely as an agent or of MERALCO with that of ASDAI does not preclude the application of Article 1217 of the
intermediary of the principal who is responsible to the workers in the same manner and Civil Code on the right of reimbursement from his co-debtor by the one who paid, which
to the same extent as if they had been directly employed by him. On the other hand, “job provides:
(independent) contracting” is present if the following conditions are met: (a) the ART. 1217. Payment made by one of the solidary debtors extinguishes the obligation.
contractor carries on an independent business and undertakes the contract work on his If two or more solidary debtors offer to pay, the creditor may choose which offer to
own account under his own responsibility according to his own manner and method, free accept.
from the control and direction of his employer or principal in all matters connected with - He who made the payment may claim from his co-debtors only the share which
the performance of the work except to the result thereof; and (b) the contractor has corresponds to each, with the interest for the payment already made. If the payment is
substantial capital or investments in the form of tools, equipment, machineries, work made before the debt is due, no interest for the intervening period may be demanded.
premises and other materials which are necessary in the conduct of his business.[29] When one of the solidary debtors cannot, because of his insolvency, reimburse his share
Given the above distinction and the provisions of the security service agreements to the debtor paying the obligation, such share shall be borne by all his co-debtors, in
entered into by petitioner with ASDAI and AFSISI, we are convinced that ASDAI and proportion to the debt of each.
AFSISI were engaged in job contracting. - ASDAI may not seek exculpation by claiming that MERALCO’s payments to it were
2. YES inadequate for the individual respondents’ lawful compensation. As an employer, ASDAI
- The individual respondents can not be considered as regular employees of the is charged with knowledge of labor laws and the adequacy of the compensation that it
MERALCO for, although security services are necessary and desirable to the business of demands for contractual services is its principal concern and not any other’s.[35]
MERALCO, it is not directly related to its principal business and may even be considered Disposition present petition is GRANTED. The assailed Decision, dated September
unnecessary in the conduct of MERALCO’s principal business, which is the distribution of 27, 2000, of the CA is REVERSED and SET ASIDE. The Decision of the Labor Arbiter
electricity. dated January 3, 1994 and the Resolution of the NLRC dated April 10, 1995 are
- Furthermore, the fact that the individual respondents filed their claim for unpaid AFFIRMED with the MODIFICATION that the joint and solidary liability of ASDAI and
monetary benefits against ASDAI is a clear indication that the individual respondents MERALCO to pay individual respondents’ monetary claims for underpayment of actual
acknowledge that ASDAI is their employer. regular hours and overtime hours rendered, and premium pay for holiday and rest day,
- We cannot give credence to individual respondents’ insistence that they were absorbed as well as attorney’s fees, shall be without prejudice to MERALCO’s right of
by AFSISI when MERALCO’s security service agreement with ASDAI was terminated. reimbursement from ASDAI.
The individual respondents failed to present any evidence to confirm that AFSISI
absorbed them into its workforce. Thus, respondent Benamira was not retained in his
SAN MIGUEL CORP V ABELLA
post at MERALCO since July 25, 1992 due to the termination of the security service
Labor Law 1 A2010 - 65 - Disini
[PAGE 59] 2. YES
- The consistent rule is that the employer must affirmatively show rationally adequate
evidence that the dismissal was for a justifiable cause, failing in which would make the
termination illegal, as in this case.
- Contrary to petitioner’s claim of abandonment as a valid just cause for termination,
herein respondents did not abandon their work. Petitioner failed to prove that (1) not only
of respondents’ failure to report for work or absence without valid reason, but (2) also of
respondents’ clear intention to sever employer-employee relations as manifested by
some overt acts.
- By filing the complaint for illegal dismissal within two days of their dismissal and by
BIG AA MANUFACTURER V ANTONIO seeking reinstatement in their position paper, respondents manifested their intention
against severing their employment relationship with petitioner and abandoning their jobs.
484 SCRA 392 It is settled that an employee who forthwith protests his layoff cannot be said to have
QUISUMBING; March 3, 2006 abandoned his work
Disposition Petition denied.
NATURE
Petition for review on certiorari of a decision of CA
COCA-COLA OTTLERS PHILS INC V NLRC
FACTS (CANONICATO)
- Petitioner Big AA Manufacturer is a sole proprietorship registered in the name of its 307 SCRA 131
proprietor, Enrico E. Alejo. Respondents filed a complaint for illegal lay-off and illegal BELLOSILLO; May 17, 1999
deductions
- Respondents
FACTS
> That as regular employees, they worked from 8:00 a.m. to 5:00 p.m. at petitioner’s
- On April 7, 1986 Coca-Cola entered into a contract of janitorial services with Bacolod
premises using petitioner’s tools and equipment and they received P250 per day.
Janitorial Services (BJS) as an independent contractor.
Eutiquio was employed as carpenter-foreman from 1991-99; Jay as carpenter from 1993-
- Private respondent Ramon Canonicato was hired as a janitor by the Bacolod Janitorial
99; Felicisimo as carpenter from 1994-99; and Leonardo, Sr. also as carpenter from
Services (BJS). He was assigned at the Coca Cola Bottlers, Inc. considering his
1997-99; That they were dismissed without just cause and due process; hence, their
familiarity with its premises, having been previous casual employee there.
prayer for reinstatement and full backwages.
- Goaded by information that COCA COLA employed previous BJS employees who filed
- Petitioner Big AA Manufacturer
a complaint against the company for regularization pursuant to a compromise
> That it is a sole proprietorship registered in the name of Enrico Alejo and engaged in
agreement, Canonicato submitted a similar complaint against COCA COLA to the Labor
manufacturing office furniture, but it denied that respondents were its regular employees.
Arbiter on 8 June 1993 and consequently did not report for work.
It claimed that Eutiquio Antonio was one of its independent contractors who used the
- On September 28,1993, BJS sent him a letter advising him to report to work within 3
services of the other respondents. It said that its independent contractors were paid by
days from receipt, otherwise he would be terminated.
results and were responsible for the salaries of their own workers. Allegedly, there was
- (there was no express mention of a termination but based on the fact I assume he did
no employer-employee relationship between petitioner and respondents. But it allowed
not return to work and was terminated)
respondents to use its facilities to meet job orders. It also denied that respondents were
- On July 23, 1993, respondent filed with the Labor Arbiter a complaint for illegal
laid-off by Big AA Manufacturer, since they were project employees only. It added that
dismissal and underpayment of wages. He included BJS therein as a co-respondent.
since Eutiquio Antonio had refused a job order of office tables, their contractual
The Labor Arbiter dismissed the complaint and ruled that a) there was no employer-
relationship ended.
employee relationship between Canonicato and Coca Cola (b) BJS was a legitimate job
- Labor Arbiter ruled againstpetitioners. Both appealed to NLRC. Respondents appealed
contractor, hence, any liability of COCA COLA as to Canonicato's salary or wage
for not ordering their reinstatement to their former positions. The NLRC modified the
differentials was solidary with BJS in accordance with pars. 1 and 2 of Art. 106, Labor
Labor Arbiter’s decision. It ordered petitioner to reinstate respondents to their former
Code; (c) COCA COLA and BJS must jointly and severally pay Canonicato his wage
positions or to pay them separation pay in case reinstatement was no longer feasible,
differentials amounting to P2,776.80 and his 13th month salary of P1,068.00, including
with full backwages in either case. The NLRC ruled that respondents were regular
ten (10%) percent attorney's fees in the sum of P384.48.
employees, not independent contractors. It further held that petitioner failed to justify its
- The NLRC rejected the decision of the Labor Arbiter on the ground that the janitorial
reason for terminating respondents and its failure to comply with the due process
services of Canonicato were found to be necessary in the usual trade of Coca Cola. In so
requirements. CA affirmed NLRC ruling.
holding, NLRC applied Art.280 of the Labor Code and declared that Canonito was a
regular employee of Coca-Cola. Its motion for reconsideration having been denied, Coca
ISSUES
Cola filed this petition.
1. WON respondents were regular employees
2. WON respondents were illegally dismissed
ISSUE
WON Canonito was a regular employee of Coca-cola and thus malking Coca-Cola liable
HELD
for illegal dismissal
1. YES
- Respondents were employed for more than 1 year and their work as carpenters was
HELD
necessary or desirable in petitioner’s usual trade or business of manufacturing office
NO
furniture. Under Art. 280 of the Labor Code, the applicable test to determine whether an
- In Kimberly Independent Labor Union v. Drilon where the Court took judicial notice of
employment should be considered regular or non-regular is the reasonable connection
the practice adopted in several government and private institutions and industries of
between the particular activity performed by the employee in relation to the usual
hiring janitorial services on an "independent contractor basis." In this respect, although
business or trade of the employer.
janitorial services may be considered directly related to the principal business of an
- True, certain forms of employment require the performance of usual or desirable
employer, as with every business, we deemed them unnecessary in the conduct of the
functions and exceed 1 year but do not necessarily result to regular employment under
employer's principal business.
Art. 280 of the Labor Code. Some specific exceptions include project or seasonal
- This judicial notice, of course, rests on the assumption that the independent contractor
employment. Yet, in this case, respondents cannot be considered project employees.
is a legitimate job contractor so that there can be no doubt as to the existence of an
Petitioner had neither shown that respondents were hired for a specific project the
employer-employee relationship between contractor and the worker. In this situation, the
duration of which was determined at the time of their hiring nor identified the specific
only pertinent question that may arise will no longer deal with whether there exists an
project or phase thereof for which respondents were hired.
employment bond but whether the employee may be considered regular or casual as to
Obiter on Requirements for an Independent contractor : a) he carries a distinct and
deserve the application of Art. 280 of the Labor Code.
independent business, b) possesses substantial capital or investment in tools,
- It was error therefore for the NLRC to apply Art. 280 of the Labor Code in determining
equipment, machinery or work premises, c) he does not work within another
the existence of an employment relationship of the parties herein, especially in light of
employer/company’s premises using the latter’s tools and materials, and d) he is not
our explicit holding in Singer Sewing Machine Company v. Drion that -
under the control and supervision of an employer or company
Labor Law 1 A2010 - 66 - Disini
“The Court agrees with the petitioner's argument that Article 280 is not the yardstick for 1. WON the individual private respondents are regular employees of PAL
determining the existence of an employment relationship because it merely distinguishes 2. WON petitioner is liable to them for separation pay
between two kinds of employees, i.e., regular employees and casual employees, for
purposes of determining the right of an employee to certain benefits, to join or form a HELD
union, or to security of tenure. Article 280 does not apply where the existence of an 1. No employer-employee relation between complainants and petitioner.
employment relationship is in dispute” Ratio a) Janitorial service agreement is not labor-only contacting AND
- In determining the existence of an employer-employee relationship it is necessary to b) Extension of service contract is not a source of employer-employee relation.
determine whether the following factors are present: (a) the selection and engagement Reasoning
of the employee; (b) the payment of wages; (c) the power to dismiss; and, (d) the power a) Prohibited labor-only contracting is defined in Article 106 of the Labor Code as follows:
to control the employee's conduct. Notably, these are all found in the relationship There is "labor-only" contracting where the person supplying workers to an employer
between BJS and Canonicato and not between Canonicato and petitioner COCA COLA. does not have substantial capital or investment in the form of tools, equipment,
As the Solicitor-General manifested machineries, work premises, among others, and the workers recruited and placed by
- BJS satisfied all the requirements of a job-contractor under the law, namely, (a) the such persons are performing activities which are directly related to the principal business
ability to carry on an independent business and undertake the contract work on its own of such employer. In such cases, the person or intermediary shall be considered merely
account under its own responsibility according to its manner and method, free from the as an agent of the employer who shall be responsible to the workers in the same manner
control and direction of its principal or client in all matters connected with the and extent as if the latter were directly employed by him.
performance of the work except as to the results thereof; and, (b) the substantial capital - This definition covers any person who undertakes to supply workers to an employer,
or investment in the form of tools, equipment, machinery, work premises, and other where such person:
materials which are necessary in the conduct of its business. (1) Does not have substantial capital or investment in the form of tools, equipment,
- All told, there being no employer-employee relationship between Canonicato and COCA [machinery], work premises and other materials; and
COLA, the latter cannot be validly ordered to reinstate the former and pay him back (2) The workers recruited and placed by such person are performing activities which are
wages. directly related to the principal business or operations of the employer in which workers
are habitually employed.
- On the other hand, permissible job contracting requires the following conditions:
PAL V NLRC (STELLAR INDUSTRIAL SERVICES INC) (1) The contractor carries on an independent business and undertakes the contract work
298 SCRA 430 on his own account under his own responsibility according to his own manner and
PANGANIBAN; November 9, 1998 method, free from the control and direction of his employer or principal in all matters
connected with the performance of the work except as to the results thereof; and
NATURE (2) The contractor has substantial capital or investment in the form of tools, equipment,
Special civil action for certiorari, seeking to nullify the July 13, 1994 Decision and the [machinery], work premises, and other materials which are necessary in the conduct of
June 27, 1996 Resolution of the National Labor Relations Commission, which held his business.
Philippines Airlines, Inc. liable for separation pay. - The employee-employer relation existed between the individual private respondents
and STELLAR, not PAL. STELLAR possessed these earmarks of an employer:
FACTS (1) the power of selection and engagement of employees
- Sometime in 1977, PAL, a local air carrier, entered into a service agreement with (2) the payment of wages
STELLAR, a domestic corporation engaged, among others, in the business of job (3) the power of dismissal, and
contracting janitorial services. Pursuant to their service agreement, which was impliedly (4) the power to control the employee's conduct
renewed year after year, STELLAR hired workers to perform janitorial and maintenance - A contract of employment existed between STELLAR and the individual private
services for PAL. The employees were assigned at PAL's various premises under the respondents, proving that it was said corporation which hired them. It was also STELLAR
supervision of STELLAR's supervisors/foremen and timekeepers. The workers were also which dismissed them, as evidenced by Complainant Parenas' termination letter, which
furnished by STELLAR with janitorial supplies, such as vacuum cleaner and polisher. was signed by Carlos P. Callanga, vice president for operations and comptroller of
- On December 31, 1990, the service agreement between PAL and STELLAR expired. STELLAR. Likewise, they worked under STELLAR's own supervisors, Rodel
PAL then called for [the] bidding of its janitorial requirements. This notwithstanding, Pagsulingan, Napoleon Parungao and Renato Topacio. STELLAR even had its own
STELLAR exerted efforts to maintain its janitorial contract with PAL which, in the collective bargaining agreement with its employees, including the individual private
meantime, allowed Manuel Parenas and others to work at the PAL's premises. respondents. Moreover, PAL had no power of control and dismissal over them.
- Subsequently, in a letter dated October 31, 1990, PAL formally informed STELLAR that - In fact, STELLAR claims that it falls under the definition of an independent job
the service agreement would no longer be renewed effective November 16, 1991, since contractor. Thus, it alleges that it has sufficient capital in the form of tools and equipment,
PAL's janitorial requirements were bidded to three other job contractors. Alleging that like vacuum cleaners and polishers, and substantial capitalization as proven by its
they were illegally dismissed, the aforenamed individual private respondents filed, from financial statements. Further, STELLAR has clients other than petitioner, like San Miguel
January to June 1992, five complaints against PAL and STELLAR for illegal dismissal Corporation, Hongkong and Shanghai Bank, Eveready, Benguet Management
and for payment of separation pay. Corporation and Japan Airlines.
- Labor Arbiter Manuel P. Asuncion rendered on October 29, 1993 a Decision which held - All these circumstances establish that STELLAR undertook said contract on its account,
PAL liable for the separation pay of terminated individual respondents. under its own responsibility, according to its own manner and method, and free from the
- In its Decision affirming the ruling of the labor arbiter, Respondent Commission held control and direction of the petitioner. Where the control of the principal is limited only to
petitioner, as an indirect employer, jointly and severally liable with STELLAR for the result of the work, independent job contracting exists. The janitorial service
separation pay. First, the individual private respondent's work, although not directly agreement between petitioner and STELLAR is definitely a case of permissible job
related to the business of petitioner, was necessary and desirable for the maintenance of contracting.
the petitioner's premises and airplanes. Second, the individual private respondents were b) What actually happened was that PAL and STELLAR impliedly renewed, as they had
retained for thirteen long years, despite the fact that the contract, which petitioner had previously done before, their service agreement until PAL's janitorial requirements were
entered into STELLAR in 1977, was only for one year. bidded to other job contractors. This explains why the individual private respondents
On reconsideration, the NLRC modified its earlier Decision by absolving STELLAR of remained working at PAL's premises even after December 31, 1990.
liability, thereby making PAL solely responsible for the award decreed by the labor arbiter. - It is evident that petitioner was engaged in permissible job contracting and that the
It held that, first, petitioner was the employer of the individual private respondents, for it individual private respondents, for the entire duration of their employ, were employees
engaged in labor-only contracting with STELLAR. This was shown by the failure of not of petitioner but of STELLAR. In legitimate job contracting, no employer-employee
petitioner to refute the factual finding that it continued to employ the individual private relation exists between the principal and the job contractor's employees. The principal is
respondents after the expiration of the service contract on December 31, 1990. Second, responsible to the job contractor's employees only for the proper payment of wages. But
the individual private respondents' admission in their Complaint that they were in labor-only contracting, an employer-employee relation is created by law between the
employees of STELLAR was not conclusive, as the existence of an employer-employee principal and the labor-only contractor's employees, such that the former is responsible
relation was a question of law that could not be the subject of stipulation. Respondent to such employees, as if he or she had directly employed them. Besides, the Court has
Commission concluded that their dismissal was without just and valid cause. Because already taken judicial notice of the general practice adopted in several government and
they were no longer seeking reinstatement, petitioner was liable for separation pay. private institutions of securing janitorial services on an independent contractor basis.
2. NO, STELLAR is the one liable for separation pay.
ISSUES
Labor Law 1 A2010 - 67 - Disini
Ratio Despite the protestations of STELLAR, the service agreement was not a project - The respondent was burdened to prove that the petitioner was the employer of Sido but
because its duration was not determined or determinable. failed to discharge this burden.
Reasoning - The respondent’s counsel admitted Sido was not employed by the petitioner
- In order to avoid liability for separation pay, STELLAR argues that it terminated the - Store manager Santos testified that Sido was not an employee of the petitioner, but of
services of the individual private respondents for a just and valid cause: the completion of BSSC, Black Shield Agency.
a specific project. Thus, they are not entitled to separation pay. - The petitioner adduced in evidence its contract with the BSSC, which contained the
- The Court is not convinced. The position of STELLAR that individual private following provisions: 1. THE AGENCY shall provide the CLIENT with the necessary
respondents were its project employees is totally unfounded. A regular employee is number of armed, uniformed and qualified security guards properly licensed by the Chief
distinguished from a project employee by the fact that the latter is employed to carry out of Philippine Constabulary; who shall provide security services to the CLIENT at its
a specific project or undertaking, the duration or scope of which was specified at the time establishment at –
the employees were engaged. A "project" has reference to a particular job or undertaking These security guards during the life of the Agreement shall be assigned in
that may or may not be within the regular or usual business of the employer. In either accordance with arrangements to be made between the CLIENT and the AGENCY.
case, the project must be distinct, separate and identifiable from the main business of the ...
employer, and its duration must be determined or determinable. 6. The AGENCY assumes full responsibility for any claim or cause of action which
- While the service agreement may have had a specific term, STELLAR disregarded it, may accrue in favor of any security guard by reason of employment with the AGENCY,
repeatedly renewed the service agreement, and continued hiring the individual private it being understood that security guards are employees of the AGENCY and not of the
respondents for thirteen consecutive years. Had STELLAR won the bidding, the alleged CLIENT.
"project" would have never ended. In any event, the aforesaid stipulations in the - Therefore, the respondent had no cause of action against the petitioner for damages for
employment contract are not included in Articles 282 and 283 of the Labor Code as valid Sido’s illegal and harmful acts. The respondent should have sued Sido and the BSSC for
causes for the dismissal of employees. damages, conformable to A2180.
Again, we must emphasize that the main business of STELLAR is the supply of - In Soliman, Jr. v. Tuazon the court held that where the security agency recruits, hires
manpower to perform janitorial services for its clients, and the individual private and assigns the works of its watchmen or security guards to a client, the employer of
respondents were janitors engaged to perform activities that were necessary and such guards or watchmen is such agency, and not the client, since the latter has no hand
desirable to STELLAR's enterprise. In this case, we hold that the individual private in selecting the security guards. Thus, the duty to observe the diligence of a good father
respondents were STELLAR's regular employees, and there was no valid cause for their of a family cannot be demanded from the said client
dismissal. - The petitioner had assigned Sido to help the management open and close the door of
Disposition petition is hereby GRANTED. The assailed Decision and Resolution are the drug store; inspect the bags of customers as they enter the store; and, check the
SET ASIDE insofar as they held PAL liable for separation pay. The July 13, 1994 receipts issued by the cashier to said customers for their purchases. Such circumstances
Decision is however reinstated insofar as it ORDERED STELLAR liable for such award. do not automatically make the security guard the employee of the petitioner, and, as
such, liable for the guard's tortious acts. The fact that a client company may give
instructions or directions to the security guards assigned to it, does not, by itself, render
MERCURY DRUG CORPORATION V LIBUNAO the client responsible as an employer of the security guards concerned and liable for
434 SCRA 404 their wrongful acts or omissions.
CALLEJO, SR; July 14, 2004 Disposition petition is hereby GRANTED. The Decision dated June 9, 2000 and the
Resolution dated August 9, 2000 of the Court of Appeals in CA-G.R. CV No. 59754 are
NATURE hereby REVERSED and SET ASIDE. The complaint filed by the respondent against
Petition for review on certiorari of a CA decision which modified an RTC decision, and the petitioner Mercury Drug Corporation in Civil Case No. Q-92-14114 is DISMISSED. The
Resolution of the CA denying the petitioner’s motion for reconsideration counterclaims of the latter are also DISMISSED. No costs.

FACTS MARIVELESSHIPYARD V CA
According to the plaintiff….
415 SCRA573
> Libunao and his friend bought some items at Mercury. He paid for his purchase and
placed his receipt in his pocket. As they exited, they were accosted by Sido, the security QUISUMBING; November 11, 2003
guard. Sido was armed with a service gun, and was 20 pounds heavier than Libunao. He
held Libunao’s upper right arm and demanded to see the receipt. Libunao searched but it FACTS
took time because Sido was holding his right arm. Sido then said “Wala yatang resibo - In October 1993, petitioner Mariveles Shipyard Corporation engaged the services of
yan!” Libunao finally found it, and asked Sido, “Satisfied ka na?” Sido reacted by lunging Longest Force Investigation and Security Agency, Inc. to render security services at its
at him and saying “Putang ina mo!” Sido was able to hit lubnao on the face, nose, chin, premises. Pursuant to their agreement, Longest Force deployed its security guards, the
and mouth. He then pointed his revolver at Libunao and said “Putang ina mo, pag hindi private respondents herein, at the petitioner’s shipyard in Mariveles, Bataan.
kayo lumabas ditto papuputukin ko to sa iyo!” Libunao eventually filed a criminal - According to petitioner, it found the services being rendered by the assigned guards
complaint against Sido. He was traumatized by the event, he had to consult a unsatisfactory and inadequate, causing it to terminate its contract with Longest Force on
psychiatrist, and was found to be suffering from post-traumatic depression syndrome. April 1995. Longest Force, in turn, terminated the employment of the security guards it
According to the defendants… had deployed at petitioner’s shipyard.
> Sido, the security guard at Mercury, noticed Libunao exiting the store with a plastic - Private respondents filed a case for illegal dismissal and underpayment of wages,
bag, and that no receipt was stapled to it. He asked for the receipt, but was given the among others. In turn, Longest Force filed a cross-claim against Mariveles Shipyard,
plastic bag. He found no receipt, and when Libunao finally found the receipt and shoved alleging that the service fee paid by the latter to it was way below the PNPSOSIA and
it in his face, he just explained he was doing his duty. Libunao said “Baka hindi mo ako PADPAO rate.
kilala, security guard ka lang! Ano ba talaga ang problema mo?” A violent argument - The petitioner denied any liability on account of the alleged illegal dismissal, stressing
ensued. that no employer-employee relationship existed between it and the security guards.
- The court rendered judgment in favor of the plaintiff, that the defendants Sido, Mercurly Petitioner likewise prayed that Longest Force’s cross-claim be dismissed for lack of
Drug Corporation, and Store Manager Vilma Santos, pay the plaintiff moral and merit. Petitioner averred that Longest Force had benefited from the contract, it was now
exemplary damages, to discourage disrespect of the public by such acts as were estopped from questioning said agreement on the ground that it had made a bad deal.
committed by defendants - The Labor Arbiter found Mariveles and Longest Force jointly and severally liable for
private respondents’ money claims and attorney’s fees. Longest Force was likewise
ISSUE ordered to reinstate private respondents without loss of seniority rights and privileges
WON the remedy of the petitioner is proper (that Mercury Drug be liable for Sido’s with full backwages. The NLRC affirmed the Labor Arbiter’s decision.
actions) - The Court of Appeals refused to give due course to Mariveles Shipyard’s appeal for
failure to comply with procedural requirements.
HELD
NO ISSUES
Ratio The petitioner was not Sido’s employer; hence, CC A 2180 should not be applied 1. WON the Court of Appeals’ dismissal of the petition was in order despite petitioner’s
against petitioner. subsequent compliance with the procedural requirements
Reasoning 2. WON petitioner was denied due process of law by the NLRC
Labor Law 1 A2010 - 68 - Disini
3. WON petitioner is jointly and severally liable with Longest Force for private 418 SCRA 411
respondents’ money claims
YNARES-SANTIAGO; December 11, 2003
HELD
1. NO FACTS
- The requirement in the Rules that the certification of non-forum shopping should be - New Golden City Builders and Development Corporation, a corporation engaged in the
executed and signed by the plaintiff or the principal means that counsel cannot sign said construction business, entered into a construction contract with Prince David
certification unless clothed with special authority to do so. The reason for this is that the Development Corporation for the construction of a 17-storey office and residential
plaintiff or principal knows better than anyone else whether a petition has previously condominium building along Katipunan Road, Loyola Heights, Quezon City, Metro
been filed involving the same case or substantially the same issues. Hence, a Manila.
certification signed by counsel alone is defective and constitutes a valid cause for - Petitioner engaged the services of Nilo Layno Builders to do the specialized “concrete
dismissal of the petition. In the case of the corporations, the physical act of signing may works, form works and steel rebar works”, for a total contract price of P5 Million. Nilo
be performed, on behalf of the corporate entity, only by specifically authorized individuals Layno Builders hired private respondents to perform work at the project. After the
for the simple reason that corporations, as artificial persons, cannot personally do the completion of the phase for which Nilo Layno Builders was contracted sometime in 1996,
task themselves. In this case, not only was the originally appended certification signed by private respondents filed a complaint case against petitioner and its president, Manuel
counsel, but in its motion for reconsideration, still petitioner utterly failed to show that Ms. Sy, with the Arbitration Branch of the NLRC for “unfair labor practice, non-payment of
Rosanna Ignacio, its Personnel Manager who signed the verification and certification of 13th month pay, non-payment of 5 days service incentive leave, illegal dismissal and
non-forum shopping attached thereto, was duly authorized for this purpose. severance pay in lieu of reinstatement.”
2. NO - The Labor Arbiter found that Nilo Layno Builders was a labor-only-contractor; thus,
- The essence of due process is simply an opportunity to be heard, or, as applied to private respondents were deemed employees of New Golden City. Both parties appealed
administrative proceedings, an opportunity to explain one’s side or an opportunity to seek the decision of the Labor Arbiter to the NLRC. Petitioner maintained that Nilo Layno
a reconsideration of the action or ruling complained of. Not all cases require a trial-type Builders was an independent contractor and that private respondents were not its
hearing. The requirement of due process in labor cases before a Labor Arbiter is employees. On the other hand, private respondents claimed that the Labor Arbiter erred
satisfied when the parties are given the opportunity to submit their position papers to in finding that they were not illegally dismissed and not entitled to recover monetary
which they are supposed to attach all the supporting documents or documentary claims like premium pay for rest days, regular holidays and special holiday. The NLRC
evidence that would prove their respective claims, in the event the Labor Arbiter affirmed with modification the Labor Arbiter’s decision. As modified, the NLRC held that
determines that no formal hearing would be conducted or that such hearing was not private respondents were illegally dismissed and ordered petitioner to reinstate them and
necessary. In any event, petitioner was given ample opportunity to present its side in to pay their full back wages. CA affirmed.
several hearings conducted before the Labor Arbiter and in the position papers and other
supporting documents that it had submitted. Such opportunity more than satisfies the ISSUES
requirement of due process in labor cases. 1. WON Nilo Layno Builders was an “independent contractor” and not a “labor-only”
3. YES contractor
- Petitioner’s liability is joint and several with that of Longest Force, pursuant to Articles 2. WON an employer-employee relationship existed between petitioner and private
106, 107 and 109 of the Labor Code. In this case, when petitioner contracted for security respondents
services with Longest Force as the security agency that hired private respondents to
work as guards for the shipyard corporation, petitioner became an indirect employer of HELD
private respondents pursuant to Article 107. Following Article 106, when the agency as 1. YES
contractor failed to pay the guards, the corporation as principal becomes jointly and Ratio The test to determine the existence of independent contractorship is whether one
severally liable for the guards’ wages. This is mandated by the Labor Code to ensure claiming to be an independent contractor has contracted to do the work according to his
compliance with its provisions, including payment of statutory minimum wage. The own methods and without being subject to the control of the employer, except only to the
security agency is held liable by virtue of its status as direct employer, while the results of the work.
corporation is deemed the indirect employer of the guards for the purpose of paying their Reasoning
wages in the event of failure of the agency to pay them. This statutory scheme gives the - Under Section 8, Rule VIII, Book III, of the Omnibus Rules Implementing the Labor
workers the ample protection consonant with labor and social justice provisions of the Code, an independent contractor is one who undertakes “job contracting,” i.e., a person
1987 Constitution. who: (a) carries on an independent business and undertakes the contract work on his
- Petitioner cannot evade its liability by claiming that it had religiously paid the own account under his own responsibility according to his own manner and method, free
compensation of guards as stipulated under the contract with the security agency. Labor from the control and direction of his employer or principal in all matters connected with
standards are enacted by the legislature to alleviate the plight of workers whose wages the performance of the work except as to the results thereof; and (b) has substantial
barely meet the spiraling costs of their basic needs. Labor laws are considered written in capital or investment in the form of tools, equipments, machineries, work premises, and
every contract. Stipulations in violation thereof are considered null. Similarly, legislated other materials which are necessary in the conduct of the business.
wage increases are deemed amendments to the contract. - Nilo Layno Builders hired its own employees, the private respondents, to do specialized
- However, we must emphasize that the joint and several liability imposed on petitioner is work in the Prince David Project of the petitioner. The means and methods adopted by
without prejudice to a claim for reimbursement by petitioner against the security agency the private respondents were directed by Nilo Layno Builders except that, from time to
for such amounts as petitioner may have to pay to complainants, the private respondents time, the engineers of the petitioner visited the site to check whether the work was in
herein. The security agency may not seek exculpation by claiming that the principal’s accord with the plans and specifications of the principal. As admitted by Nilo G. Layno,
payments to it were inadequate for the guards’ lawful compensation. As an employer, the he undertook the contract work on his own account and responsibility, free from
security agency is charged with knowledge of labor laws; and the adequacy of the interference from any other persons, except as to the results; that he was the one paying
compensation that it demands for contractual services is its principal concern and not the salaries of private respondents; and that as employer of the private respondents, he
any other’s. had the power to terminate or dismiss them for just and valid cause.
- On the issue of the propriety of the award of overtime pay despite the alleged lack of - As a licensed labor contractor, Nilo Layno Builders complied with the conditions set
proof thereof, suffice it to state that such involves a determination and evaluation of facts forth in Section 5, Rule VII-A, Book III, Rules to Implement the Labor Code, among
which cannot be done in a petition for review. others, proof of financial capability and list of equipment, tools, machineries and
- Upon review of the award of backwages and attorney’s fees, we discovered certain implements to be used in the business.
errors that happened in the addition of the amount of individual backwages that resulted 2. YES. [But for a limited purpose only]
in the erroneous total amount of backwages and attorney’s fees. These errors ought to Ratio In legitimate job contracting, the law creates an employer-employee relationship
be properly rectified now. Thus, the correct sum of individual backwages should be for a limited purpose, i.e., to ensure that the employees are paid their wages. The
P 126,648.40 instead of P126,684.40, while the correct sum of total backwages awarded principal employer becomes jointly and severally liable with the job contractor only for the
and attorney’s fees should be P 3,926,100.40 and P 392,610.04, instead of payment of the employees’ wages whenever the contractor fails to pay the same. Other
P3,927,216.40 and P392,721.64, respectively. than that, the principal employer is not responsible for any claim made by the employees.
Disposition The Court of Appeals’ Resolution is AFFIRMED with MODIFICATION. Reasoning
- The petitioner did not, as it could not, illegally dismissed the private complainants.
Hence, it could not be held liable for back wages and separation pay. Nevertheless, it is
NEW GOLDEN CITY BUILDERS V CA (GALLO ET AL) jointly and severally liable with Nilo Layno Builders for the private complainants’ wages,
Labor Law 1 A2010 - 69 - Disini
in the same manner and extent that it is liable to its direct employees. The pertinent 1. PMCI is a labor-only contractor.
provisions of the Labor Code read: Ratio In determining the existence of an independent contractor relationship, several
ART. 106. Contractor or subcontractor. – Whenever an employer enters into a factors might be considered such as, but not necessarily confined to, whether the
contract with another person for the performance of the former’s work, the employees contractor is carrying on an independent business; the nature and extent of the work; the
of the contractor and of the latter’s subcontractor, if any, shall be paid in accordance skill required; the term and duration of the relationship; the right to assign the
with the provisions of this Code. XXX In the event that the contractor or subcontractor performance of specified pieces of work; the control and supervision of the workers; the
fails to pay the wages of his employees in accordance with this Code, the employer power of the employer with respect to the hiring, firing and payment of the workers of the
shall be jointly and severally liable with his contractor or subcontractor to such contractor; the control of the premises; the duty to supply premises, tools, appliances,
employees to the extent of the work performed under the contract, in the same materials and labor; and the mode, manner and terms of payment.
manner and extent that he is liable to employees directly employed by him. Reasoning
ART. 107. Indirect employer. – The provisions of the immediately preceding Article - Labor-only contracting, a prohibited act, is an arrangement where the contractor or
shall likewise apply to any person, partnership, association or corporation which, not subcontractor merely recruits, supplies or places workers to perform a job, work or
being an employer, contracts with an independent contractor for the performance of service for a principal. The following elements are present: (a) The contractor or
any work, task, job or project. subcontractor does not have substantial capital or investment to actually perform the job,
- This liability covers the payment of service incentive leave and 13th month pay of the work or service under its own account and responsibility; (b) The employees recruited,
private complainants during the time they were working at petitioners’ Prince David supplied or placed by such contractor or subcontractor are performing activities which
Project. So long as the work, task, job or project has been performed for petitioners’ are directly related to the main business of the principal.
benefit or on its behalf, the liability accrues for such period even if, later on, the - Permissible job contracting or subcontracting refers to an arrangement whereby a
employees are eventually transferred or reassigned elsewhere. principal agrees to put out or farm out with a contractor or subcontractor the performance
Disposition Petition PARTLY GRANTED. Decision of the CA MODIFIED. Petitioner or completion of a specific job, work or service within a definite or predetermined period,
ABSOLVED from liability for back wages. However, he is ORDERED to pay, jointly and regardless of whether such job, work or service is to be performed or completed within or
severally with Nilo Layno Builders, private complainants’ Service Incentive Leave Pay outside the premises of the principal. A person is considered engaged in legitimate job
and 13th Month Pay. contracting or subcontracting if the following conditions concur: (a) The contractor or
subcontractor carries on a distinct and independent business and undertakes to perform
the job, work or service on its own account and under its own responsibility according to
VINOYA V NLRC (REGENT FOOD CORP) its own manner and method, and free from the control and direction of the principal in all
324 SCRA 469 matters connected with the performance of the work except as to the results thereof; (b)
KAPUNAN; February 2, 2000 The contractor or subcontractor has substantial capital or investment; and (c) The
agreement between the principal and contractor or subcontractor assures the contractual
NATURE employees entitlement to all labor and occupational safety and health standards, free
Petition for certiorari under Rule 65 seeking to annul NLRC decision exercise of the right to self-organization, security of tenure, and social and welfare
benefits.
FACTS - First of all, PMCI does not have substantial capitalization or investment in the form of
- Petition seeks to annul and set aside the decision of the NLRC which reversed the tools, equipment, machineries, work premises, among others, to qualify as an
decision of the Labor Arbiter, ordering RFC to reinstate Alexander Vinoya to his former independent contractor.
position and pay him backwages. -Second, PMCI did not carry on an independent business nor did it undertake the
Petitioner’s Claim performance of its contract according to its own manner and method, free from the
> Vinoya applied and was accepted by RFC as sales representative. RFC issued him an control and supervision of its principal, RFC. The evidence at hand shows that the
identification card. workers assigned by PMCI to RFC were under the control and supervision of the latter.
- He reported daily to the RFC office to take the van for the delivery of products. He was -Third, PMCI was not engaged to perform a specific and special job or service.. As stated
assigned to various supermarkets and grocery stores where he booked sales orders and in the Contract of Service, the sole undertaking of PMCI was to provide RFC with a
collected payments for RFC. He was required by RFC to put up a monthly bond of P200 temporary workforce able to carry out whatever service may be required by it. Apart from
as security deposit to guarantee his work performance. that, no other particular job, work or service was required from PMCI. Obviously, with
- After more than a year, he was transferred by RFC to Peninsula Manpower Company, such an arrangement, PMCI merely acted as a recruitment agency for RFC.
Inc., an agency which provides RFC with additional contractual workers pursuant to a - Lastly, in labor-only contracting, the employees recruited, supplied or placed by the
contract for the supply of manpower services. After this, petitioner was reassigned to contractor perform activities which are directly related to the main business of its
RFC as sales rep. principal. In this case, the work of petitioner as sales representative is directly related to
- 5 months later, he was informed by the personnel manager of RFC that his services the business of RFC. Being in the business of food manufacturing and sales, it is
were terminated and he was asked to surrender his ID card. Petitioner was told that his necessary for RFC to hire a sales representative like petitioner to take charge of booking
dismissal was due to the expiration of the Contract of Service between RFC and PMCI. its sales orders and collecting payments for such. Thus, the work of petitioner as sales
Petitioner claims that he was dismissed from employment despite the absence of any representative in RFC can only be categorized as clearly related to, and in the pursuit of
notice or investigation. the latter’s business. Logically, when petitioner was assigned by PMCI to RFC, PMCI
- He filed a case against RFC before the Labor Arbiter for illegal dismissal and non- acted merely as a labor-only contractor.
payment of 13th month pay. 2. Petitioner was an employee of RFC
Respondent’s Comments Ratio In determining the existence of employer-employee relationship the following
> RFC maintains that there is no employer-employee relationship. Petitioner is actually elements of the "four-fold test" are generally considered, namely: (1) the selection and
an employee of PMCI, an independent contractor, which had a Contract of Service with engagement of the employee or the power to hire; (2) the payment of wages; (3) the
RFC. RFC presented an Employment Contract signed by petitioner on 1 July 1991, power to dismiss; and (4) the power to control the employee. Of these four, the "control
wherein PMCI appears as his employer. RFC denies that petitioner was ever employed test" is the most important.
by it prior to 1 July 1991. Petitioner was issued an ID card so that its clients and - No particular form of proof is required to prove the existence of an employer-employee
customers would recognize him as a duly authorized representative of RFC. With regard relationship. Any competent and relevant evidence may show the relationship.
to the P200 monthly bond, RFC asserts that it was required in order to guarantee the Reasoning
turnover of his collection since he handled funds of RFC. While RFC admits that it had - PMC I as a labor-only contractor, cannot be considered as the employer of petitioner
control and supervision over petitioner, it argues that such was exercised in coordination - Even granting that PMCI is an independent contractor, still, a finding of the same will
with PMCI. Finally, RFC contends that the termination of its relationship with petitioner not save the day for RFC. A perusal of the Contract of Service entered into between RFC
was brought about by the expiration of the Contract of Service between itself and PMCI. and PMCI reveals that petitioner is actually not included in the enumeration of the
workers to be assigned to RFC. This only shows that petitioner was never intended to be
ISSUES a part of those to be contracted out.
1. WON PMCI is a labor-only contractor or an independent contractor -With regard to the first element, ID card is enough proof that petitioner was previously
2. WON petitioner was an employee of RFC or PMCI hired by RFC prior to his transfer as agency worker to PMCI. ID card issued by RFC to
3. WON petitioner was lawfully dismissed petitioner was dated more than one year before the Employment Contract was signed by
petitioner in favor of PMCI. While the Employment Contract indicates the word "renewal,"
HELD presumably an attempt to show that petitioner had previously signed a similar contract
Labor Law 1 A2010 - 70 - Disini
with PMCI, no evidence of a prior contract entered into between petitioner and PMCI was business of petitioner. And ACGI did not carry on an independent business according to
ever presented by RFC. It follows that it was RFC who actually hired and engaged its own manner.
petitioner to be its employee - ACGI was a labor-only contractor, an agent of the petitioner.
- With respect to the payment of wages, the Court takes judicial notice of the practice of - Then the workers are employees of the petitioner.
employers who, in order to evade the liabilities under the Labor Code, do not issue - Even the four-fold test (selection, payment of wages, dismissal power, control of
payslips directly to their employees. Even though the wages were coursed through conduct) indicate the relationship.
PMCI, we note that the funds actually came from the pockets of RFC. Thus, in the end, 2. YES
RFC is still the one who paid the wages of petitioner albeit indirectly. - The term fixed in the subsequent contract was used to defeat the tenurial security.
- As to the third element, the power to dismiss, the Contract of Service gave RFC the - Dismissal was illegal.
right to terminate the workers assigned to it by PMCI without the latter’s approval. The
dismissal of petitioner was indeed made under the instruction of RFC to PMCI.
GRANDSPAN DEVELOPMENT CORP V
-The power of control refers to the authority of the employer to control the employee not
only with regard to the result of work to be done but also to the means and methods by BERNARDO
which the work is to be accomplished. The "control test" calls merely for the existence of 470 SCRA 461
the right to control the manner of doing the work, and not necessarily to the actual SANDOVAL-GUTIERREZ; September 21, 2005
exercise of the right. The Labor Arbiter found that petitioner was under the direct control
and supervision of the personnel of RFC and not PMCI.
NATURE
3. YES
Certiorari under Rule 45
Ratio The requirements for the lawful dismissal of an employee are two-fold, the
substantive and the procedural aspects. Not only must the dismissal be for a valid or
FACTS
authorized cause, the rudimentary requirements of due process - notice and hearing–
- The instant controversy stemmed from a complaint for illegal dismissal and non-
must, likewise, be observed before an employee may be dismissed.
payment of benefits filed with the LA by Ricardo Bernardo, Antonino Ceñidoza and Edgar
Reasoning
Del Prado, against Grandspan and/or its warehouse manager, Manuel G. Lee
- Since petitioner, due to his length of service, already attained the status of a regular
- In their complaint, respondents alleged that sometime in 1990, they were employed as
employee, he is entitled to the security of tenure provided under the labor laws. Hence,
truck scale monitors by Grandspan with a daily salary of P104.00 each. Eventually, they
he may only be validly terminated from service upon compliance with the legal requisites
were assigned at its Truck Scale Section of the Warehouse/Materials Department. They
for dismissal.
were issued identification cards signed.
- RFC never pointed to any valid or authorized cause under the Labor Code which
- Oct 28, 1992-Grandspan sent them a notice terminating their services effective October
allowed it to terminate the services of petitioner. Its lone allegation that the dismissal was
29, 1992 for using profane or offensive language, in violation of Article VI (2) (a) of the
due to the expiration or completion of contract is not even one of the grounds for
company’s Rules and Regulations.
termination allowed by law.
- Grandspan denied the allegations and claimed that respondents are employees of J.
- Neither did RFC show that petitioner was given ample opportunity to contest the legality
Narag Construction.
of his dismissal. In fact, no notice of such impending termination was ever given him.
- Sometime in the 3rd quarter of 1992, Canad Japan Co., Ltd. engaged Grandspan’s
- An employee who has been illegally dismissed is entitled to reinstatement to his former
services for fabrication works of several round and rectangular steel tanks needed for the
position without loss of seniority rights and to payment of full backwages corresponding
HCMG or Sogo project due for completion in September, 1992. As a consequence,
to the period from his illegal dismissal up to actual reinstatement.
Grandspan subcontracted the services of J. Narag Construction which, in turn, assigned
Disposition Petition granted.
its 3 helpers (herein respondents) to work for its project.
- Sometime in October, 1992, Manuel G. Lee, manager of Grandspan’s Warehouse
MANILA WATER COMPANY V PENA Department received a report from supervisor Robert Ong that respondents vandalized
434 SCRA 52 the company’s log book and chairs.
- This prompted petitioner to send J. Narag Construction a memorandum terminating the
YNARES-SANTIAGO; July 8, 2004 services of respondents for violation of the company’s Rules and Regulations.
- June 30, 1994- LA dismissed respondents’ complaint; concluded that respondents were
NATURE validly dismissed from employment; held too that respondent were project employees
Petition for review on certiorari whose services were terminated upon completion of the project for which they were
hired.
FACTS - NLRC- remanded the case to the LA for appropriate proceedings to determine whether
Manila Water is one of the 2 concessionaires contracted by MWSS to manage water there is an employer-employee relationship between the parties.
distribution. Pursuant to RA No. 8041, petitioner undertook to absorb ex-employees of - Both parties filed MRs but were denied by the NLRC in separate Resolutions
MWSS whose names were in the list, and those not in the list were terminated. - Meantime, Del Prado died and was substituted by his surviving parent, Edgardo Del
Respondents are contractual collectors, not listed, but petitioner still engaged their Prado.
services. They signed a 3 month contract. - Sept 17, 1999- CA set aside the NLRC’s Resolutions and ordered Grandspan (1) to
121 collectors incorporated the Association Collectors Group Inc. or ACGI. Petitioner reinstate respondents Bernardo and Ceñidoza to their former positions and pay, jointly
continued to transact with ACGI, but eventually terminated its contract. Respondents and severally with J. Narag Construction, their backwages and other benefits, and (2) to
filed complaint for illegal dismissal, saying they were petitioner’s employees. Petitioner pay respondent Del Prado his separation pay.
asserts respondents were employees of ACGI, an independent contractor. - CA found that respondents are employees of petitioner; that they were non-project
Arbiter found dismissal illegal. NLRC reversed. Respondents filed certiorari petition with workers; and that they were denied due process, thus:
CA, which reversed NLRC decision and reinstated with modification the Arbiter decision. “They(Respondents Bernardo et al) worked in Grandspan’s premises using the
Hence this petition. materials, supplies and equipment of Grandspan. They were under the supervision
of Grandspan as to the manner and results of their work, and performed services
ISSUES directly connected to the usual business of respondent Grandspan for the fabrication
1. WON there’s employer-employee relationship of heavy structural components”
2. WON respondents were illegally dismissed - Oct 8, 1999- Grandspan filed MR. Respondents also filed a motion for reconsideration
and/or clarification praying that the Appellate Court’s Decision be modified by awarding
HELD respondent Del Prado his backwages.
1. YES - Jan 6, 2000- CA denied Grandspan’s MR but order Grandspan and J. Narag
- We must resolve WON ACGI is an independent contractor or a labor-only contractor. Construction to pay respondent Del Prado his separation pay and backwages.
- Labor-only contracting refers to arrangement where contractor merely recruits and
places workers for a principal. Elements ISSUE
- contractor doesn’t have substantial capital WON the CA erred in holding that respondents are employees of Grandspan
- contractor doesn’t control performance of contractual employee
- Arbiter correctly ruled that ACGI was not an independent contractor. ACGI doesn’t have HELD
substantial capital. It has no office. The work of the respondents was directly related to YES
Labor Law 1 A2010 - 71 - Disini
Grandspan’s Argument : PHILIPPINE AIRLINES INC V NLRC (VILLENA,
> it has no employer-employee relationship with respondents since they are employees
NATIONAL ORGANIZATION OF WORKING MEN,
of J. Narag Construction, an independent contractor.
- Miguel vs. JCT Group - “The test for determining an employer-employee relationship VILLACRUZ)
hinges on resolving who has the power to select employees, who pays for their wages, 296 SCRA 214
who has the power to dismiss them, and who exercises control in the methods and the QUISUMBING; September 25, 1998
results by which the work is accomplished.”
- SC agrees with CA when it found that J. Narag Construction assigned respondents to
NATURE
perform activities directly related to the main business of petitioner . . These
Special Civil Action in the Supreme Court. Certiorari.
circumstances confirm the existence of an employer-employee relationship between
petitioner and respondents.
FACTS
> They worked in petitioner’s premises, using its equipment, materials and supplies. J.
- Petitioner Philippine Airlines Inc. (PAL) is a domestic corporation principally engaged in
Narag Construction’s payroll worksheets covering the period from December 21, 1990
the air transportation industry for both domestic and foreign markets. Private respondent
to July 31, 1991 show that the payment of their salaries was approved by petitioner.
National Organization of the Workingmen (NOWM) is a labor union, while the other
> The manager and supervisor of petitioner’s Warehouse Department supervised the
private respondents are members of respondent union and complainants in
manner and results of their work.
aforementioned cases.
> It was petitioner who terminated their services after finding them guilty of using
- PAL contracted the services G. C. Services Enterprises, to undertake specific projects.
profane or offensive language in violation of Article VI (2) (a) of the company’s Rules
Accordingly, G. C. Services recruited and hired carpenters, painters, and electricians and
and Regulations
assigned them to different PAL shops, namely: Carpentry Shop, Electrical Shop,
- SC also agrees with the CA that J. Narag Construction is a labor-only contractor.
Technical Center Shop and Inflight Center Shop, all under PAL’s Construction and
> A106 LC as amended, provides that “there is ‘labor-only’ contracting where the
Corporate Services Department.
person supplying workers to an employer does not have substantial capital or
- PAL terminated its contract with G. C. Services. As a result, all G.C. employees
investment in the form of tools, equipment, machineries, work premises, among
assigned as PAL project workers were notified by G.C. Services not to report anymore to
others, and the workers recruited and placed by such person are performing activities
PAL. Later, PAL decided to give G.C. Services employees the opportunity to apply as
which are directly related to the principal business of such employer. x x x.”
regular employees, in accordance with its practice of giving employment priority to
- J. Narag Construction is indeed a labor-only contractor. These are the reasons:
qualified persons who had been connected with PAL. Due to lack of vacant positions
(1) it is not registered as a building contractor with the SEC;
and also due to alleged unsatisfactory work performance records of some, not all G.C.
(2) it has no contract with petitioner; and
Services employees were hired. Those who were not hired instituted the instant
(3) there is no proof of its financial capability and has no list of equipment, tools,
complaint for illegal dismissal. The complainants were represented in their case by the
machineries and implements used in the business.
NOWM.
- Kiamco vs. NLRC: “The principal test for determining whether particular employees are
- Initially, there were 36 complainants in these three consolidated cases. In the course of
properly characterized as ‘project employees,’ as distinguished from ‘regular employees,’
the proceedings, PAL agreed to employ 23 qualified complainants. Only 12 complainants
is whether or not the ‘project employees’ were assigned to carry out a ‘specific project or
were left.
undertaking,’ the duration and scope of which were specified at the time the employees
- The rest of the complainants alleged that they applied for employment with G.C.
were engaged for that project. As defined, project employees are those workers hired (1)
Services; that after they were accepted they were made to work at PAL Maintenance
for a specific project or undertaking, and (2) the completion or termination of such project
Department where each of them worked as carpenters, welders, or electricians; that they
or undertaking has been determined at the time of engagement of the employee.
were not considered employees of PAL but that of G.C. Services; that their work are
- Grandspan could not present employment contracts signed by respondents showing
necessary and directly related to PAL’s principal business. In pointing at PAL as their real
that their employment was for the duration of the HCMG or Sogo project and failed to
employer, they averred that G.C. Services is only an agent of PAL because it does not
present any report terminating the services of respondents when its projects were
have substantial capital in the form of cash investments, tools, equipment or work
actually finished pursuant to Sec2.2 (e) of the Labor Department Order No. 19
premises; that it merely supplied workers to PAL and these workers were supervised,
SC: The failure of the employer to file termination reports after every project completion
directed and controlled by PAL regular employees; that PAL actually decided when,
with the nearest public employment office is an indication that respondents were not
where and what to work; that PAL decided how many of them were to be taken in, when
project employees. TF: respondents are Grandspan’s regular employees. As such, they
they would start, and when they would not. Complainants, thus, argued that G.C.
are entitled to security of tenure and can only be dismissed for a just or authorized
Services being a mere agent, the real employer was PAL pursuant to Art.106 of the Labor
cause, as provided by Article 279 of the Labor Code.
Code which prohibits the employment of persons through labor only contracting
- Bolinao Security and Investigation Service, Inc. vs. Toston- “it is incumbent upon the
agencies, like the G.C. Services Enterprise.
employer to prove by the quantum of evidence required by law that the dismissal of an
- In claiming that they were illegally dismissed, complainants alleged that they were
employee is not illegal, otherwise, the dismissal would be unjustified.”
dismissed from employment witho²ut just cause and without due process and without any
- SC: Termination is ILLEGAL. Grandspan violated respondents’ right (both substantive
prior notice. They thus prayed for reinstatement with full backwages from the date of
and procedural) to due process as records show that respondents were not served by
their dismissal on March 31, 1990 up to the date of their actual reinstatement.
notices of any kind nor were asked to explain the misconduct imputed to them.
- Respondent PAL denied the existence of employer-employee relationship between it
> Loadstar Shipping Co., Inc. vs. Mesano: “The law requires that an employee sought
and the complainants. It averred that G.C. Services Enterprises, as a duly licensed
to be dismissed must be served two written notices before termination of his
independent contractor, contracted on its own account under its own responsibility; that
employment. The first notice is to apprise the employee of the particular acts or
the contractor has substantial capital or investment in the form of tools, equipment and
omissions by reason of which his dismissal has been decided upon; and the second
other materials necessary in the conduct of its business; that complainants were being
notice is to inform the employee of the employer’s decision to dismiss him. Failure to
paid their wages by G.C. Services and not PAL; and that they were terminated by G.C.
comply with the requirement of two notices makes the dismissal illegal. The
Services. PAL further argued that even granting arguendo that complainants are entitled
procedure is mandatory. Non-observance thereof renders the dismissal of an
to be regularized, it is not obliged to employ all the complainants; and that there are no
employee illegal and void.
more positions or substantially equivalent positions within its organization for which they
- SC: they are entitled to reinstatement without loss of seniority rights, full backwages,
maybe (sic) qualified.
inclusive of allowances, and other benefits or their monetary equivalent computed from
- The Labor Arbiter ruled that G.C. Services Enterprises is a labor-only contractor and
the time their compensation was withheld from them up to the time of their
mere agent of PAL (petitioner herein), thus, the private respondents are deemed
actual reinstatement.
employees of petitioner. The Labor Arbiter then declared the termination of private
Disposition CA’s decision AFFIRMED with modification. Reinstatement in this case is
respondents’ services illegal, and held petitioner and G.C. Services Enterprises jointly
N/A because of antagonism. Respondents are entitled to a separation pay of
and severally liable to pay private respondents their separation pay, backwages as well
P4,992.00 plus their respective full backwages, and other privileges and benefits, or their
as attorney’s fees.
monetary equivalent, during the period of their dismissal up to their supposed actual
- Both parties appealed to the NLRC, which, affirmed the Labor Arbiter’s decision with
reinstatement.
modification as to the computation of the monetary award.
- Its motion for reconsideration having been denied, petitioner filed the instant petition.

ISSUES
Labor Law 1 A2010 - 72 - Disini
1. WON the public respondents committed grave abuse of discretion in declaring the Warehouse owned by MAERC. They washed and segregated various kinds of empty
dismissal of private respondents illegal despite the finding of redundancy bottles used by SMC to sell and distribute its beer beverages to the consuming public.
2. WON private respondents are entitled to separation pay as well as backwages They were paid on a per piece or pakiao basis except for a few who worked as checkers
3. WON petitioner should be held jointly and severally liable and were paid on daily wage basis. Complainants alleged that long before SMC
contracted the services of MAERC a majority of them had already been working for SMC
HELD under the guise of being employees of another contractor, Jopard Services, until the
1. YES services of the latter were terminated on 31 January 1988. SMC informed MAERC of the
- The petitioner “regularized” and/or re-employed 23 original complainants as there were termination of their service contract by the end of June 1991. SMC cited its plans to
vacant positions to which they could qualify. However, the remaining 12 complainants phase out its segregation activities starting 1 June 1991 due to the installation of labor
(private respondents herein) could no longer be absorbed into petitioner’s regular and cost-saving devices. When the service contract was terminated, complainants
workforce as there were no longer vacant positions as evidenced by the Table of claimed that SMC stopped them from performing their jobs; that this was tantamount to
Organization of PAL Construction and Corporate Services Department. Simply put, the their being illegally dismissed by SMC who was their real employer as their activities
services of private respondents were already in excess of what is reasonably demanded were directly related, necessary and desirable to the main business of SMC; and, that
by the actual manpower requirement of petitioner. It is settled that where there is need MAERC was merely made a tool or a shield by SMC to avoid its liability under the Labor
for reduction of workforce, management has the right to choose whom to layoff, Code
depending on the work still required to be done and the qualities of the workers to be - MAERC for its part admitted that it recruited the complainants and placed them in the
retained. bottle segregation project of SMC but maintained that it was only conveniently used by
- Under Article 203 (must be 283) of the Labor Code, the employer may terminate an SMC as an intermediary in operating the project or work directly related to the primary
employee due to redundancy or retrenchment. business concern of the latter with the end in view of avoiding its obligations and
- In Wilshire (sic) File Co., Inc. v. NLRC, 193 SCRA 672 the Supreme Court aptly ruled: responsibilities towards the complaining workers.
“Redundancy, for purposes of our Labor code, exists where the services of an -The Labor Arbiter rendered a decision holding that MAERC was an independent
employee are in excess of what is reasonably demanded by the actual requirements contractor. He dismissed the complaints for illegal dismissal but ordered MAERC to pay
of the enterprise. Succinctly put, a position is redundant where it is superfluous and complainants' separation benefits in the total amount of P2,334,150.00. MAERC and
the superfluity of a position or positions may be the outcome of a number of factors, SMC were also ordered to jointly and severally pay complainants their wage differentials
such as over-hiring of workers xxx. The employer has no legal obligation to keep in in the amount of P845,117.00 and to pay attorney's fees in the amount of P317,926.70.
its payroll more employees than are necessary for the operation of its business.” - The National Labor Relations Commission (NLRC) ruled that MAERC was a labor-only
(underscoring supplied) contractor and that complainants were employees of SMC. The NLRC also held that
- Clearly, the Labor Arbiter recognized the existence of redundancy. Despite said whether MAERC was a job contractor or a labor-only contractor, SMC was still solidarily
findings the Labor Arbiter ruled as follows: liable with MAERC for the latter's unpaid obligations, citing Art. 109 4 of the Labor Code.
“xxx In consonance therefore under Art. 280 of the Labor Code of the Philippines, Thus, the NLRC modified the judgment of the Labor Arbiter and held SMC jointly and
herein complainants are regular employees. For being so, they are protected by the severally liable with MAERC for complainants' separation benefits. In addition, both
Security of Tenure provision of law (Art. 279, Labor Code) the complainant dismissal respondents were ordered to pay jointly and severally an indemnity fee of P2,000.00 to
being not in contemplation with Art. 282 of the Labor Code it is therefore illegal. xxx” each complainant.
- The reference to Article 282 is misplaced. Article 282 enumerates the causes for - SMC filed petition for certiorari
termination “by reason of some blameworthy act or omission on the part of the
employee.” ISSUE
- In the instant case, the cause of termination is redundancy which is an authorized WON the complainants are employees of petitioner SMC or of respondent MAERC
cause for termination under Article 283. In any event, it is absurd for the Labor Arbiter to
declare a finding of redundancy, on one hand, and to conclude, on the other, that the HELD
termination of private respondents’ services is illegal. There being redundancy, the Employees of SMC
dismissal of private respondents is valid - In ascertaining an employer-employee relationship, the following factors are
2. NO considered: (a) the selection and engagement of employee; (b) the payment of wages;
- Since private respondents were validly dismissed under Art. 283, they are not entitled to (c) the power of dismissal; and, (d) the power to control an employee's conduct, the last
backwages. Apparently, public respondents awarded backwages to private respondents being the most important. Application of the aforesaid criteria clearly indicates an
to penalize PAL for engaging in a “labor-only” scheme. However, the law does not give employer-employee relationship between petitioner and the complainants.
public respondents such authority. The only effect of labor-only contracting is that ‘the - Evidence discloses that petitioner played a large and indispensable part in the hiring of
person or intermediary shall be considered merely as an agent of the employer who shall MAERC's workers. It also appears that majority of the complainants had already been
be responsible to the workers in the same manner and extent as if the latter were directly working for SMC long before the signing of the service contract between SMC and
employed by him’ (Art. 106, Labor Code)”. MAERC.
- Thus, private respondents are entitled to separation pay only. The award of backwages - The incorporators of MAERC admitted having supplied and recruited workers for SMC
to them has no basis in law. even before MAERC was created. The NLRC also found that when MAERC was
3. YES organized into a corporation, the complainants who were then already working for SMC
- Petitioner and G.C. Services Enterprises are jointly and severally liable to the private were made to go through the motion of applying for work with Ms. Olga Ouano, President
respondents for the latter’s monetary claims. The reason is that G.C. Services and General Manager of MAERC, upon the instruction of SMC through its supervisors to
Enterprises, being a ‘labor only’ contractor, is merely an agent of the petitioner (the make it appear that complainants were hired by MAERC. This was testified to by two (2)
employer); the resultant “liability must be shouldered by either one or shared by both”. of the workers who were segregator and forklift operator assigned to the Beer Marketing
Hence, petitioner cannot avoid liability by invoking its Service Agreement with G.C. Division at the SMC compound and who had been working with SMC under a purported
Services Enterprises considering that here the liability is imposed by law. contractor Jopard Services since March 1979 and March 1981, respectively. Both
witnesses also testified that together with other complainants they continued working for
SMC without break from Jopard Services to MAERC.
SAN MIGUEL V MAERC INTEGRATED SERVICES - As for the payment of workers' wages, it is conceded that MAERC was paid in lump
405 SCRA 579 sum but records suggest that the remuneration was not computed merely according to
BELLOSILLO; July 10, 2003 the result or the volume of work performed. The memoranda of the labor rates bearing
the signature of a Vice-President and General Manager for the Vismin Beer Operations
FACTS as well as a director of SMC appended to the contract of service reveal that SMC
- TWO HUNDRED NINETY-ONE (291) workers filed their complaints (nine [9] complaints assumed the responsibility of paying for the mandated overtime, holiday and rest day
in all) against San Miguel Corporation (petitioner herein) and Maerc Integrated Services, pays of the MAERC workers. SMC also paid the employer's share of the SSS and
Inc. (respondent herein), for illegal dismissal, underpayment of wages, non-payment of Medicare contributions, the 13th month pay, incentive leave pay and maternity benefits.
service incentive leave pays and other labor standards benefits, and for separation pays In the lump sum received, MAERC earned a marginal amount representing the
from 25 June to 24 October 1991. The complainants alleged that they were hired by San contractor's share. These lend credence to the complaining workers' assertion that while
Miguel Corporation (SMC) through its agent or intermediary Maerc Integrated Services, MAERC paid the wages of the complainants, it merely acted as an agent of SMC.
Inc. (MAERC) to work in two (2) designated workplaces in Mandaue City: one, inside the - Petitioner insists that the most significant determinant of an employer-employee
SMC premises at the Mandaue Container Services, and another, in the Philphos relationship, i.e., the right to control, is absent. The contract of services between MAERC
Labor Law 1 A2010 - 73 - Disini
and SMC provided that MAERC was an independent contractor and that the workers from said effectivity, subject to renewal of this contract should the employee's
hired by it "shall not, in any manner and under any circumstances, be considered performance be satisfactory."
employees of the Company, and that the Company has no control or supervision - Said period having elapsed, respondent was allowed to work until PFCCI presented to
whatsoever over the conduct of the Contractor or any of its workers in respect to how her another employment contract for a period of one year commencing on January 2,
they accomplish their work or perform the Contractor's obligations under the Contract." 1991 until December 31, 1991, after which period, her employment was terminated.
- In deciding the question of control, the language of the contract is not determinative of - LA dismissed her complaint for illegal dismissal against PFCCI.
the parties' relationship; rather, it is the totality of the facts and surrounding - NLRC set aside LA’s decision and ordered her reinstated to her last position held (RFO)
circumstances of each case. or to an equivalent position, with full backwages from Jan 1, 1992 until she is reinstated.
- Despite SMCs disclaimer, there are indicia that it actively supervised the complainants.
SMC maintained a constant presence in the workplace through its own checkers. Its ISSUE
asseveration that the checkers were there only to check the end result was belied by the WON Abril was a regular employee and thus illegally dismissed
testimony of Carlito R. Singson, head of the Mandaue Container Service of SMC, that the HELD
checkers were also tasked to report on the identity of the workers whose performance or YES
quality of work was not according to the rules and standards set by SMC. According to - It is an elementary rule in the law on labor relations that a probationary employee who
Singson, "it (was) necessary to identify the names of those concerned so that the is engaged to work beyond the probationary period of six months, as provided under Art.
management [referring to MAERC] could call the attention to make these people improve 281 of the Labor Code, as amended, or for any length of time set forth by the employer,
the quality of work." shall be considered a regular employee.
- Other instances attesting to SMC's supervision of the workers are found in the minutes - Article 281 of the Labor Code, as amended, allows the employer to secure the services
of the meeting held by the SMC officers on 5 December 1988. Among those matters of an employee on a probationary basis which allows him to terminate the latter for just
discussed were the calling of SMC contractors to have workers assigned to segregation cause or upon failure to qualify in accordance with reasonable standards set forth by the
to undergo and pass eye examination to be done by SMC EENT company doctor and a employer at the time of his engagement. A probationary employee is one who is on trial
review of compensation/incentive system for segregators to improve the segregation by an employer during which the employer determines whether or not he is qualified for
activities. permanent employment. A probationary employment is made to afford the employer an
- But the most telling evidence is a letter by Mr. Antonio Ouano, Vice-President of opportunity to observe the fitness of a probationer while at work, and to ascertain
MAERC dated 27 May 1991 addressed to Francisco Eizmendi, SMC President and Chief whether he will become a proper and efficient employee. Probationary employees,
Executive Officer, asking the latter to reconsider the phasing out of SMC's segregation notwithstanding their limited tenure, are also entitled to security of tenure. Thus, except
activities in Mandaue City. The letter was not denied but in fact used by SMC to advance for just cause as provided by law, or under the employment contract, a probationary
its own arguments. Briefly, the letter exposed the actual state of affairs under which employee cannot be terminated.
MAERC was formed and engaged to handle the segregation project of SMC. It provided - PFCCI refutes the findings of the NLRC arguing that, after respondent had allegedly
an account of how in 1987 Eizmendi approached the would-be incorporators of MAERC abandoned her secretarial position for eight (8) months, she applied for the position of
and offered them the business of servicing the SMC bottle-washing and segregation Regional Field Officer for Region IV, which appointment, as petitioner would aptly put it,
department in order to avert an impending labor strike. After initial reservations, MAERC "had been fixed for a specific project or undertaking the completion or termination of
incorporators accepted the offer and before long trial segregation was conducted by which had been determined at the time of the engagement of said private respondent
SMC at the PHILPHOS warehouse. and therefore considered as a casual or contractual employment under Article 280 of the
- In legitimate job contracting, the law creates an employer-employee relationship for a Labor Code."
limited purpose, i.e., to ensure that the employees are paid their wages. The principal - Abril cannot be classified as casual or contractual. (This is why the Court went into a
employer becomes jointly and severally liable with the job contractor only for the discussion of the kinds of employment recognized in this jurisdiction)
payment of the employees' wages whenever the contractor fails to pay the same. Other "Art. 280. Regular and casual employment . — The provisions of written agreement to
than that, the principal employer is not responsible for any claim made by the employees. the contrary notwithstanding and regardless of the oral agreement of the parties, an
- On the other hand, in labor-only contracting, the statute creates an employer- employment shall be deemed to be regular where the employee has been engaged to
employee relationship for a comprehensive purpose: to prevent a circumvention of labor perform activities which are usually necessary or desirable in the usual business or
laws. The contractor is considered merely an agent of the principal employer and the trade of the employer, except where the employment has been fixed for a specific
latter is responsible to the employees of the labor-only contractor as if such employees project or undertaking the completion or termination of which has been determined at
had been directly employed by the principal employer. The principal employer the time of the engagement of the employee or where the work or services to be
therefore becomes solidarily liable with the labor-only contractor for all the performed is seasonal in nature and the employment is for the duration of the season.
rightful claims of the employees. - An employment shall be deemed to be casual if it is not covered by the preceding
- This distinction between job contractor and labor-only contractor, however, will not paragraph: Provided, That, any employee who has rendered at least one year of service,
discharge SMC from paying the separation benefits of the workers, inasmuch as MAERC whether such service is continuous or broken, shall be considered a regular employee
was shown to be a labor-only contractor; in which case, petitioner's liability is that of a with respect to the activity in which he is employed and his employment shall continue
direct employer and thus solidarily liable with MAERC. while such activity exists."
- SMC also failed to comply with the requirement of written notice to both the employees - This provision of law comprehends three kinds of employees:
concerned and the Department of Labor and Employment (DOLE) which must be given (a) regular employees or those whose work is necessary or desirable to the usual
at least one (1) month before the intended date of retrenchment. The fines imposed for business of the employer;
violations of the notice requirement have varied. The measure of this award depends on (b) project employees or those whose employment has been fixed for a specific
the facts of each case and the gravity of the omission committed by the employer. For its project or undertaking the completion or termination of which has been determined at
failure, petitioner was justly ordered to indemnify each displaced worker P2,000.00. the time of the engagement of the employee or where the work or services to be
Disposition Petition is DENIED. performed is seasonal in nature and the employment is for the duration of the season;
and
(c) casual employees or those who are neither regular nor project employees.
PHIL. FEDERATION OF CREDIT COOPERATIVES INC - For contractual employees, stipulations in employment contracts providing for term
(PFCCI) V NLRC (ABRIL) employment or fixed period employment are valid when
300 SCRA 72 (1) the period were agreed upon knowingly and voluntarily by the parties without
ROMERO; December 11, 1998. force, duress or improper pressure being brought to bear upon the employee and
absent any other circumstances vitiating his consent, or
(2) where it satisfactorily appears that the employer and employee dealt with each
FACTS
other on more or less equal terms with no moral dominance whatever being exercised
- Victoria Abril was employed by PFCCI in different capacities from 1982 to 1988, when
by the former over the latter.
she went on leave until she gave birth. When she went back in 1989, after 8 months,
- The contract (see facts) contains stipulations so ambiguous as to preclude a precise
another employee had been permanently appointed to her former position of office
application of pertinent labor laws. Since contract of employment is contract of adhesion,
secretary. She accepted a position of Regional Field Officer. The contract reads:
ambiguity is construed strictly against the party who prepared it. Also, Art. 1702 of CC
"That the employer hires the employee on contractual basis to the position of
provides that in case of doubt, all labor contracts shall be construed in favor of the
Regional Field Officer of Region 4 under PFCCI/WOCCU/Aid Project No. 8175 and to
laborer. The interpretation which the respondent company seeks to wiggle out is wholly
do the function as stipulated in the job description assigned to him (her): on
unacceptable, as it would result in a violation of petitioner's right to security of tenure
probationary status effective February 17/90 for a period not to exceed six (6) months
Labor Law 1 A2010 - 74 - Disini
guaranteed in Section 3 of Article XIII of the Constitution and in Articles 279 and 281 of - A regular employee is one who is engaged to perform activities which are necessary
the Labor Code. and desirable in the usual business or trade of the employer as against those which are
- Regardless of the designation petitioner may have conferred upon respondent's undertaken for a specific project or are seasonal.
employment status, it is, however, uncontroverted that the latter, having completed the - There are two separate instances whereby it can be determined that an employment is
probationary period and allowed to work thereafter, became a regular employee who may regular: (1) if the particular activity performed by the employee is necessary or desirable
be dismissed only for just or authorized causes under Articles 282, 283 and 284 of the in the usual business or trade of the employer; and, (2) if the employee has been
Labor Code, as amended. Therefore, the dismissal, premised on the alleged expiration of performing the job for at least a year.
the contract, is illegal and entitles respondent to the reliefs prayed for. - In the case of S t. Theresa's School of Novaliches Foundation vs. NLRC , we held that
Disposition The petition is hereby DISMISSED and the decision of the National Labor Article 280 of the Labor Code does not proscribe or prohibit an employment
Relations Commission dated November 28. 1994 is AFFIRMED. contract with a fixed period. We furthered that it does not necessarily follow that
where the duties of the employee consist of activities usually necessary or
desirable in the usual business of the employer, the parties are forbidden from
PANGILINAN V GENERAL MILLING CORPORATION agreeing on a period of time for the performance of such activities. There is
434 SCRA 159 thus nothing essentially contradictory between a definite period of employment
CALLEJO, SR; July 12, 2004 and the nature of the employee's duties.
- In the case of Brent School Inc. v. Zamora, the SC laid down the guideline before a
NATURE contract of employment may be held as valid, to wit: Stipulations in employment
Petition for review on certiorari of a decision of the Court of Appeals contracts providing for term employment or fixed period employment are valid when the
period were agreed upon knowingly and voluntarily by the parties without force, duress or
FACTS improper pressure, being brought to bear upon the employee and absent any other
- Respondent General Milling Corporation is a domestic corporation engaged in the circumstances vitiating his consent, or where it satisfactorily appears that the employer
production and sale of livestock and poultry. It is, likewise, the distributor of dressed and employee dealt with each other on more or less equal terms with no moral
chicken to various restaurants and establishments nationwide. dominance whatever being exercised by the former over the latter.
- Petitioners were employed by the respondent as emergency workers under separate - An examination of the contracts entered into by the petitioners showed that their
"temporary/casual contracts of employment" for a period of five months. employment was limited to a fixed period, usually five or six months, and did not go
- Upon the expiration of their respective contracts, their services were terminated. beyond such period.
- They later filed separate complaints for illegal dismissal and non-payment of holiday - The records reveal that the stipulations in the employment contracts were knowingly
pay, 13th month pay, night-shift differential and service incentive leave pay against the and voluntarily agreed to by the petitioners without force, duress or improper pressure, or
respondent before the Arbitration Branch of the National Labor Relations Commission, any circumstances that vitiated their consent. Similarly, nothing therein shows that these
- Petitioners alleged that their work as chicken dressers was necessary and desirable in contracts were used as a subterfuge by the respondent GMC to evade the provisions of
the usual business of the respondent, and added that although they worked from 10:00 Articles 279 and 280 of the Labor Code.
p.m. to 6:00 a.m., they were not paid night-shift differential. - The petitioners were hired as "emergency workers" and assigned as chicken dressers,
- They stressed that based on the nature of their work, they were regular employees of packers and helpers at the Cainta Processing Plant.
the respondent; hence, could not be dismissed from their employment unless for just - While the petitioners' employment as chicken dressers is necessary and desirable in
cause and after due notice. the usual business of the respondent, they were employed on a mere temporary basis,
- Labor Arbiter Voltaire A. Balitaan rendered a decision in favor of the petitioners since their employment was limited to a fixed period. As such, they cannot be said to be
declaring that they were regular employees. regular employees, but are merely "contractual employees."
- Finding that the termination of their employment was not based on any of the just - Consequently, there was no illegal dismissal when the petitioners' services were
causes provided for in the Labor Code, the Labor Arbiter declared that they were terminated by reason of the expiration of their contracts.
allegedly illegally dismissed. - Lack of notice of termination is of no consequence, because when the contract
- On May 25, 1998, the NLRC rendered a decision reversing that of the Labor Arbiter specifies the period of its duration, it terminates on the expiration of such period. A
- The NLRC held that the petitioners, who were temporary or contractual employees of contract for employment for a definite period terminates by its own term at the end of
the respondent, were legally terminated upon the expiration of their respective contracts. such period.
Citing the case of Brent School, Inc. vs. Zamora , the NLRC explained that while the Disposition Petition is denied.
petitioners' work was necessary and desirable in the usual business of GMC, they cannot
be considered as regular employees since they agreed to a fixed term. DE LEON V NLRC (LA TONDENA)
- The petitioners' motion for reconsideration of the decision having been denied by the
176 SCRA 615
NLRC, they filed a petition for certiorari before the Court of Appeals.
- On September 29, 2000, the CA rendered a decision affirming decision of the NLRC FERNAN; August 21, 1989
- The CA ruled that where the duties of the employee consist of activities usually
necessary or desirable in the usual business of the employer, it does not necessarily NATURE
follow that the parties are forbidden from agreeing on a period of time for the Petition for certiorari seeking to annul and set aside: (1) majority decision of the NLRC,
performance of such activities. which reversed the Order of Labor Arbiter Hernandez; and, (2) the Resolution denying
- Petitioners’ MFR was denied, hence, this petition petitioner's MFR

ISSUE FACTS
WON the petitioners were regular employees of the respondent GMC when their - DE LEON was employed by LA TONDENA (business of manufacture and distillery of
employment was terminated wines and liquors) on Dec 11, 1981, at the Maintenance Section of its Engineering Dept
in Tondo.
HELD - His work consisted mainly of painting company building and equipment, and other odd
NO jobs relating to maintenance. He was paid on a daily basis through petty cash vouchers.
- Petitioners were employees with a fixed period, and, as such, were not regular - After service of more than 1 year, DE LEON requested that he be included in the payroll
employees. of regular workers. LA TONDENA responded by dismissing him from work.
- Article 280 of the Labor Code comprehends three kinds of employees: (a) regular - Weeks after this, he was re-hired indirectly through the Vitas-Magsaysay Village
employees or those whose work is necessary or desirable to the usual business Livelihood Council, a labor agency of respondent, and was made to perform tasks he
of the employer; (b) project employees or those whose employment has been used to do.
fixed for a specific project or undertaking the completion or termination of which - Having been refused reinstatement despite repeated demands, petitioner filed a
has been determined at the time of the engagement of the employee or where complaint before the Office of the Labor Arbiter.
the work or services to be performed is seasonal in nature and the employment - LA TONDENA claimed he was a casual worker hired only to paint a certain bldg in the
is for the duration of the season; and, (c) casual employees or those who are company premises, and such work terminated upon completion of the painting job.
neither regular nor project employees. - Labor Arbiter Hernandez ordered reinstatement and payment of backwages to
petitioner. “Complainant's being hired on casual basis did not dissuade from the cold fact
Labor Law 1 A2010 - 75 - Disini
that such jobs he performed related to maintenance as a maintenance man is necessary giving him again another contract of employment for another specific period cannot be
and desirable to the better operation of the business company.” countenanced. This is one way of doing violence to the employee's constitutional right to
- On appeal, NLRC reversed such decision because his job cannot be considered security of tenure under which even employees under probationary status are amply
necessary in the usual trade of employer: "Painting the business or factory building is not protected.
a part of the respondent's manufacturing or distilling process of wines and liquors.” - SMC’s MFR was denied by NLRC. Hence, this petition.

ISSUE ISSUES
WON petitioner is a regular employee 1. WON De Guzman is a regular employee
2. WON De Guzman was illegally dismissed

HELD
1. NO
Art. 280 of the Labor Code defines regular, project and casual employment as follows:
“An employment shall be deemed to be regular where the employee has been
HELD engaged to perform activities which are usually necessary or desirable in the usual
1. YES business or trade of the employer, except where the employment has been fixed for a
Ratio An employment shall be deemed to be casual if it is not covered by Art.281 6 of specific project or undertaking the completion or termination of which has been
Labor Code: provided, That any employee who has rendered at least one year of service, determined at the time of the engagement of the employee or where the work or
whether such service is continuous or broken, shall be considered a regular employee services to be performed is seasonal in nature and the employment is for the duration
with respect to the activity in which he is employed and his employment shall continue of the season.
while such actually exists. An employment shall be deemed to be casual if it is not covered by the preceding
Reasoning paragraph: Provided, That, any employee who has rendered at least one year of
- During petitioner's period of employment, the records reveal that the tasks assigned to service, whether such service is continuous or broken, shall be considered a regular
him included not only painting of company buildings, equipment and tools but also employee with respect to the activity in which he is employed and his employment
cleaning and oiling machines, even operating a drilling machine, and other odd jobs shall continue while such actually exists.
assigned to him when he had no painting job. - The above mentioned provision reinforces the Constitutional mandate to protect the
- It is not the will and word of the employer that determines whether a certain interest of labor as it sets the legal framework for ascertaining one's nature of
employment is regular or casual, to which the desperate worker often accedes, but the employment, and distinguishing different kinds of employees. Its language manifests the
nature of the activities performed in relation to the particular business or trade intent to safeguard the tenurial interest of worker who may be denied the enjoyment of
considering all circumstances, and in some cases the length of time of its performance the rights and benefits due to an employee, regardless of the nature of his employment,
and its continued existence. by virtue of lopsided agreements which the economically powerful employer who can
Disposition Petition is GRANTED. maneuver to keep an employee on a casual or contractual status for as long as it is
convenient to the employer.
- Thus, under Article 280 of the Labor Code, an employment is deemed regular when the
SAN MIGUEL CORPORATION V NLRC (GUZMAN) activities performed by the employee are usually necessary or desirable in the usual
297 SCRA 277 business or trade of the employer even if the parties enter into an agreement stating
QUISUMBING; October 7, 1998 otherwise. But considered not regular under said Article (1) the so-called "project
employment" the termination of which is more or less determinable at the time of
NATURE employment, such as those connected, which by its nature is only for one season of the
Petition for certiorari. year and the employment is limited for the duration of that season, such as the Christmas
holiday season. Nevertheless, an exception to this exception is made: any employee who
FACTS has rendered at least 1 year of service, whether continuous or intermitent, with respect to
- In November 1990, Francisco De Guzman, JR. was hired by SMC as helper/bricklayer the activity he performed and while such activity actually exists, must be deemed regular.
for a specific project, the repair and upgrading of furnace C at its Manila Glass Plant. His - Following Article 280, whether one is employed as a project employee or not would
contract of employment provided that said temporary employment was for a specific depend on whether he was hired to carry out a "specific project or undertaking", the
period of approximately 4 months. On April 30, 1991, De Guzman was able to complete duration and scope of which were specified at the time his services were engaged for
the repair and upgrading of furnace C. Thus, his services were terminated on that same that particular project. Another factor that may be undertaken by the employee in relation
day as there was no more work to be done. His employment contract also ended that to the usual trade or business of the employer, if without specifying the duration and
day. scope, the work to be undertaken is usually necessary or desirable in the usual business
- On May 10, 1991, De Guzman was again hired for a specific job which involved the or trade of the employer, then it is regular employment and not just "project" must less
draining/cooling down of fuenace F and the emergency repair of furnace E. This project "casual" employment.
was for a specific period of approximately 3 months. After the completion of this task, at - Thus, the nature of one's employment does not depend on the will or word of the
the end of July 1991, DE Guzman's services were terminated. employer. Nor on the procedure of hiring and the manner of designating the employee,
- On Aug.1, 1991, complainant saw his name in a Memorandum posted at the but on the nature of the activities to be performed by the employee, considering the
Company's Bulletin Board as among those who were considered dismissed. employer's nature of business and the duration and scope of the work to be done.
- On Aug.12, 1994, or after the lapse of more than 3 years from the completion of the - Project could refer to 2 distinguishable types of activity. Firstly, a project could refer to a
last undertaking for which De Guzman was hired, he filed a complaint for illegal dismissal particular job or undertaking that is within the regular or usual business of the employer
against SMC. company, but which is distinct at separate, and identifiable as such, from the other
- On June 30, 1995, labor Arbiter Felipe Garduque II rendered the decision dismissing undertakings of the company. Such job or undertaking begins and ends at determined or
said complaint for lack of merit, sustaining SMC's argument that DE Guzman was a determinable times. . . . Secondly, a project could refer to a particular job or undertaking
project employee. The position of a helper does not fall within the classification of regular that is not within the regular business of the corporation. Such a job or undertaking must
employees. Hence, complainant never attained regular employment status. Moreover, his also be identifiably separate and distinct from the ordinary or regular business operations
silence for more than three (3) years without any reasonable explanation tended to of the employer. The job or undertaking also begins and ends at determined or
weaken his claim. determinable times . .
- Upon appeal, NLRC reversed Labor Arbiter Garduque's decision. In its ruling, NLRC - The plant where De Guzman was employed for only 7 months is engaged in the
stated that SMC’s scheme of subsequently re-hiring complainant after only 10 days from manufacturer of glass, an integral component of the packaging and manufacturing
the last day of the expiration of his contract of employment for a specific period, and business of petitioner. The process of manufacturing glass requires a furnace, which has
a limited operating life. SMC resorted to hiring project or fixed term employees in having
said furnaces repaired since said activity is not regularly performed. Said furnaces are to
6
Art. 281. Regular and casual employment. The provisions of a written agreement to the contrary notwithstanding and be repaired or overhauled only in case of need and after being used continuously for a
regardless of the oral agreements of the parties, an employment shall be deemed to be regular where the employee varying period of 5-10 years. In 1990, one of the furnaces of petitioner required repair
has been engaged to perform activities which are usually necessary or desirable in the usual business or trade of the
employer, except where the employment has been fixed for a specific project or undertaking the completion or and upgrading. This was an undertaking distinct and separate from SMC's business of
termination of which has been determined at the time of the engagement of the employee or where the work or manufacturing glass. For this purpose, SMC must hire workers to undertake the said
services to be performed is seasonal in nature and the employment is for the duration of the season.
Labor Law 1 A2010 - 76 - Disini
repair and upgrading. De Guzman was, thus, hired by SMC on November 28, 1990 on a California then amended their complaint charging California with illegal dismissal.
"temporary status for a specific job" for a determined period of approximately four Thereafter, Livi reabsorbed them into its labor pool on a “wait-in or standby” status.
months - Respondents claim: they are not the petitioner’s employer (Livi is, therefore, no
- Upon completion of the undertaking, or on April 30, 1991, DE Guzman's services were employer-employee relationship between them) and that the "retrenchment" had been
terminated. A few days, thereafter, two of SMC's furnaces required "draining/coolong forced by business losses as well as expiration of contracts ( "unfavorable political
down" and "emergency repair". De Guzman was again hired on May 10, 1991 to help in and economic atmosphere coupled by the February Revolution.")
the new undertaking, which would take approximately 3 months to accomplish. Upon - LA: no employer-employee relationship in the light of the manpower supply contract;
completion of the second undertaking, private respondent's services were likewise California not liable for the money claims demanded. Livi also absolved from any
terminated. He was not hired a third time, and his two engagements taken together did obligation because retrenchment was allegedly beyond its control, but were to pay
not total one full year in order to qualify him as an exception to the exception falling under separation pay and attorney’s fees.
the cited proviso in the second paragraph of Art. 280 of the Labor Code. - NLRC: affirm labor arbiter’s deci
2. NO
- De Guzman was hired for a specific project that was not within the regular business of ISSUES
the corporation. For SMC is not engaged in the business of repairing furnaces. Although 1. WON the petitioners are employees of California Manufacturing Company
the activity was necessary to enable petitioner to continue manufacturing glass, the 2. WON the petitioners were illegally dismissed
necessity therefor arose only when a particular furnace reached the end of its life or HELD
operating cycle. Or, as on the second undertaking, when a particular furnace required an 1. YES
emergency repair. In other words, the undertakings where he was hired primarily as Ratio The existence of an employer-employees relation is a question of law and being
helper/bricklayer have specified goals and purpose which are fulfilled once the such, it cannot be made the subject of agreement. The determination of whether or not
designated work was completed. Moreover, such undertakings were also identifiably there is an employer-employee relation depends upon four standards: (1) the manner of
separate and distinct from the usual, ordinary or regular business operations of petitioner, selection and engagement of the putative employee; (2) the mode of payment of wages;
which is glass manufacturing. These undertakings, the duration and scope of which had (3) the presence or absence of a power of dismissal; and (4) the presence or absence of
been determined and made known to private respondent at the time of his employment a power to control the putative employee's conduct. Of the four, the right-of-control test
clearly indicated the nature of his employment as a project employee. Thus, his services has been held to be the decisive factor.
were terminated legally after the completion of the project. Reasoning
- If NLRC’s decision is upheld, it would amount to negating the distinction made in Article - IN RELATION TO THE MANPOWER SUPPLY AGREEMENT: The fact that the manpower supply
280 of the Labor Code. It would shunt aside the rule that since a project employee's work agreement between Livi and California had specifically designated the former as the
defends on the availability of a project, necessarily, the duration of his employment is petitioners' employer and had absolved the latter from any liability as an employer, will
coterminous with the project to which he is assigned. It would become a burden for an not erase either party's obligations as an employer, if an employer-employee relation
employer to retain an employee and pay him his corresponding wages it there was no otherwise exists between the workers and either firm. At any rate, since the agreement
project for him to work on. was between Livi and California, they alone are bound by it, and the petitioners cannot
- While the Constitution is committed to the policy of social justice and the protection of be made to suffer from its adverse consequences.
the working class, it should not be supposed that every dispute will be automatically - Art. 106 of the Labor Code 7 still imposes responsibility on both firms:
decided in favor of labor. Management has also rights, which, as such, are entitled to Notwithstanding the absence of a direct employer-employee relationship between the
respect and enforcement in the interest of fair play. Although the SC has inclined more employer in whose favor work had been contracted out by a "labor-only" contractor, and
often than not toward the worker and has upheld has cause in his conflicts with the the employees, the former has the responsibility, together with the "labor-only" contractor,
employer, such favoritism has no blinded the Court to the rule that justice is in avery case for any valid labor claims by operation of law. The reason, so we held, is that the "labor-
for the deserving, to be dispensed in the light of the established facts and the applicable only" contractor is considered "merely an agent of the employer," and liability must be
law and doctrine. shouldered by either one or shared by both.
Disposition Petition is hereby GRANTED. The decision of respondent NLRC is hereby - Livi, as a placement agency, had simply supplied California with the manpower
REVERSED, and the judgment of the Labor Arbiter REINSTATED. necessary to carry out California’s merchandizing activities, using the latter’s premises
and equipment.
- O N PETITIONERS BEING “ DIRECT EMPLOYEES OF L IVI : Not conclusive – will not absolve
TABAS V NLRC (CALIFORNIA MANUFACTURING) California from liability imposed by law and relations of parties are not determined by
169 SCRA 497 their declarations
SARMIENTO; January 26, 1989 - O N TEMPORARY OR SEASONAL BASIS HIRING : temporary or casual employee, under
Article 218 of the Labor Code, becomes regular after service of one year, unless he has
NATURE been contracted for a specific project. Merchandising is not a specific project, it is an
PETITION to review the decision and resolution of the National Labor Relations activity related to the day-to-day operations of California.
Commission. *The Court need not therefore consider whether it is Livi or California which exercises
control over the petitioner vis-a-vis the four barometers reffered to earlier, since by fiction
FACTS of law, either or both shoulder responsibility.
- Petitioners were employees of Livi Manpower Services, Inc. (Livi). Livi subsequently -R EASONING FOR DISMISSING THE TERMS AND CONDITIONS OF THE MANPOWER SUPPLY
assigned them to work as “promotional merchandisers” for California Manufacturing Co. AGREEMENT : not illegal, under the Labor Code, genuine job contracts are
(California) pursuant to a manpower supply agreement. The agreement provided the permissible, provided they are genuine job contracts. But when such arrangements
following, among others: (1) that California had no control/supervision over the are resorted to "in anticipation of, and for the very purpose of making possible, the
petitioners with respect to how they accomplish their work; (2) that Livi is an secondment"of the employees from the true employer, the Court will be justified in
“independent contractor” and that the relationship between Livi and California should not
be construed to be of principal-agent or employer-employee; (3) that California is free
and harmless (!?!) from any liability arising from such laws or from any accident that may 7
ART. 106. Contractor or subcontractor.-Whenever an employee enters into a contract with another person for
befall the workers and employees of Livi while in the performance of their duties for
the performance of the former's work, the employees of the contractor and of the letter's subcontractor, if any,
California; (4) that the assignment of workers to California shall be on a “seasonal and shall be paid in accordance with the provisions of this Code.
contractual basis”; (5) that most of living allowance and the 10 legal holidays will be In the event that the contractor or subcontractor fails to pay wages of his employees in accordance with this
charged directly to California at cost; and (6) that the payroll for the preceding week shall Code, the employer shall be jointly and severally liable with his contractor or subcontractor to such employees to
the extent of the work performed under the contract, in the same manner and extent that he is liable to employees
be delivered by LIvi at California’s premises. directly employed by him.
- Petitioners were then made to sign employment contracts with durations of six The Secretary of Labor may, by appropriate regulations, restrict or prohibit the contracting out of labor to protect
months, upon the expiration of which they signed new agreements with the the rights of workers established under this Code. In so prohibiting or restricting, he may make appropriate
distinctions between labor-only contracting and job contracting as well as differentiations within these types of
same period, and so on . Unlike regular California employees, who received not less contracting and determine who among the parties involved shall be considered the employer for purposes of this
than P2,823.00 a month in addition to a host of fringe benefits and bonuses, they Code, to prevent any violation or circumvention of any provisions of this Code.
received P38.56 plus P15.00 in allowance daily. There is "labor-only" contracting where the person supplying workers to an employer does not have substantial
capital or investment in the form of tools, equipment, machineries, work premises, among others, and the workers
- Petitioners filed complaints, demanding to have similar benefits as regular employees; recruited and placed by such person are performing activities which are directly related to the principal business of
but pending their claims, California notified them that they would not be rehired. such employer. In such cases, the person or intermediary shall be considered merely as an agent of the employer
who shall be responsible to the workers in the same manner and extent as if the latter were directly employed by
him.
Labor Law 1 A2010 - 77 - Disini
expressing its concern. For then that would compromise the rights of the workers, ISSUES
especially their right to security of tenure. 1. WON the NLRC and Labor Arbiter erred in not finding respondent to be a regular
2. YES employee
Ratio Retrenchment of workers, unless clearly warranted, has serious consequences 2. WON the CBA applies to respondent
not only on the State's initiatives to maintain a stable employment record for the country, 3. WON respondent was deprived of due process
but more so, on the workingman himself, amid an environment that is desperately scarce 4. WON dismissal was a just penalty
in jobs.
Reasoning The 6-month contracts of the petitioners were renewed, and accordingly, HELD
under Article 281 (Labor Code), they had become regular employees of California and 1. YES
had acquired a secure tenure. Hence, they cannot be separated without due process of - According to Article 280 of the Labor Code, there are 2 kinds of regular employees: (1)
law. those engaged to perform activities which are necessary or desirable in the usual
- O N VALIDITY OF RETRENCHMENT : California has not shown enough evidence that it had business or trade of the employer; and (2) those casual employees who have rendered
in fact suffered serious business reverses as a result alone of the prevailing political and at least one year of service, whether continuous or broken, with respect to the activities
economic climate; attribution to February Revolution as cause of alleged losses in which they are employed. The respondent obviously falls under the first type of regular
gratuitous and without basis in fact. employee. She had been working continuously for the petitioner for over a year,
Disposition petition is GRANTED. Judgment is hereby RENDERED: (1) SETTING evidencing the necessity and indispensability of her services to the petitioner’s business.
ASIDE the decision, dated March 20, 1987, and the resolution, dated August 19, 1987; By operation of law, respondent had attained regular status and was thus entitled to
(2) ORDERING the respondent, the California Manufacturing Company, to REINSTATE security of tenure as provided in Art. 279 of the code. The said article requires a just
the petitioners with fall status and rights of regular employees; and (3) ORDERING the cause before termination, and entitles the employee to reinstatement and other privileges
respondent, the California Manufacturing Company, and the respondents, Livi Manpower in absence of one.
Service, Inc. and/or Lily-Victoria A. Azarcon, to PAY, jointly and severally, unto the - Petitioner’s hiring policy for contract employees is contrary to the spirit of Articles 279
petitioners: (a) backwages and differential pays effective as and from the time they had and 280 of the code; it is but an excuse to prevent regularization and circumvent the law
acquired a regular status under the second paragraph, of Section 281, of the Labor on security of tenure. This is echoed in Sec. 3 Art XVI of the Constitution which deems
Code, but not to exceed three (3) years, and (b) all such other and further benefits as security of tenure a State policy to guarantee social justice. The fact is that the operation
may be provided by existing collective bargaining agreement(s) or other relations, or by of every business depends on supply and demand—the cyclical nature of one’s trade
law, beginning such time; and (4) ORDERING the private respondents to PAY unto the cannot be invoked as a reason to place an employee’s status on shaky ground.
petitioners attorney's fees equivalent to ten (10%) percent of all money claims hereby - This is not to say that term employment is illegal outright. In Romares v NLRC it was
awarded, in addition to those money claims. said that term employment does not circumvent the law when the fixed period was
knowingly and voluntarily agreed upon by both parties and that such agreement was
made with no party holding moral dominance over the other. However, none of these
PHILIPS SEMICONDUCTORS V FADRIQUELA requisites are present in the instant case.
427 SCRA 408 2. NO
CALLEJO, SR; April 14, 2004 - Petitioner’s reliance on the CBA is misplaced. The CBA constitutes the law between the
employer and regular employees, but cannot be binding on contractual employees who
FACTS are not represented by the bargaining union. The CBA provision requiring 17 months for
- On May 8, 1992, respondent Eloisa Fadriquela executed a Contract of Employment regularization runs contrary to what is clearly stipulated in law, which provides that
with petitioner Philips Semiconductors as a production operator, initially for 3 months. regularization requires only 1 year.
Because her performance constantly met petitioner’s ratings requirements, her contract 3. YES
was renewed several times, extending to 12 months. However, over the last few months, - Respondent was dismissed without the requisite notice and formal investigation.
respondent incurred several absences for which she offered no valid justification despite Dismissals must not be arbitrary and capricious; a mere dialogue between the
a prompting to do so by the line supervisor. As a consequence, her performance rating respondent and line supervisor cannot possibly suffice as a substitute for actual notice
dropped, and respondent’s contract was no longer renewed. and hearing.
- Respondent filed a complaint with the NLRC for illegal dismissal, claiming she had not 4. NO
been duly notified; she furthered that having rendered over 6 months of service, she was - Dismissal is too harsh a penalty for mere absences, especially since the repeated
already a regular employee and could not be terminated without just cause. renewal of respondent’s contract proves her efficiency as a worker. The SC mandates
- Petitioner contended that respondent had not been dismissed; rather, her contract that “where a penalty less punitive would suffice, whatever missteps may be committed
merely expired and was not renewed. by labor ought not to be visited with a consequence so severe”.
- The Labor Arbiter dismissed the complaint for lack of merit but awarded her severance Disposition IN LIGHT OF ALL THE FOREGOING, the assailed decision of the
of 1 month’s pay. He stated that petitioner and its union’s CBA required one to render 17 appellate court is AFFIRMED. The petition at bar is DENIED.
months of service to be considered regular. He also added that respondent could not
complain of being deprived of notice and hearing as the line supervisor had asked her to MAGSALIN V NATIONAL ORGANIZATION OF
explain her absences. An appeal with the NLRC yielded the same results. It was pointed
WORKING MEN (NOWM)
out that as a contractual employee respondent was bound by the stipulations of her
contract of employment, which in this case was a satisfactory performance rating. 403 SCRA 199
- Dissatisfied, respondent filed a petition for certiorari before the CA, which reversed the VITUG; May 9, 2003
decisions of the NLRC and the Labor Arbiter. The appellate court argued that the NLRC
and the Labor Arbiter employed inappropriate bases for their decisions, since the CBA FACTS
did not apply to contractual employees like Fadriquela. The CA cited Art. 280 of the Labor - Coca-Cola Bottlers Phils., Inc., herein petitioner, engaged the services of respondent
Code which states that regardless of any written or oral agreements between employer workers as “sales route helpers” for a limited period of five months. After five months,
and employee, “an employment shall be deemed to be regular where the employee has respondent workers were employed by petitioner company on a day-to-day basis to
been engaged to perform activities which are usually necessary or desirable in the usual substitute for regular sales route helpers whenever the latter would be unavailable or
business or trade of the employer”. Petitioner’s contention that employment was obtained when there would be an unexpected shortage of manpower in any of its work places or
as the need arose was illogical, as this would mean the employee would never attain an unusually high volume of work. The practice was for the workers to wait every
regular status. The CA further held that a less punitive penalty would suffice for morning outside the gates of the sales office of petitioner company. If thus hired, the
absenteeism. Finally, it held that the dialogue between the respondent and line workers would then be paid their wages at the end of the day.
supervisor was insufficient as to amount to notice, and thus the former was deprived of - Ultimately, respondent workers asked petitioner company to extend to them regular
due process. appointments. Petitioner company refused.
- Petitioner filed a motion for reconsideration in which petitioner claimed that its hiring - November 7, 1997 - twenty-three (23) of the ”temporary” workers (herein respondents)
policy was neither new nor prohibited and that it was a valid exercise of its management filed with the National Labor Relations Commission (NLRC) a complaint for the
prerogative since demand for its semiconductors is cyclical in nature. It added that it had regularization of their employment with petitioner company. The complaint was
the prerogative to set reasonable standards of employment qualification as provided by amended a number of times to include other complainants that ultimately totaled fifty-
law. The motion was denied, hence this petition for review. eight (58) workers. Claiming that petitioner company meanwhile terminated their
services, respondent workers filed a notice of strike and a complaint for illegal dismissal
Labor Law 1 A2010 - 78 - Disini
and unfair labor practice with the NLRC. NATURE
- 01 April 1998 - voluntary arbitration Before the Court is a Petition for Review under Rule 45 of the Rules of Court, seeking to
- 18 May 1998 - the voluntary arbitrator rendered a decision dismissing the complaint on set aside CA Decision denying petition for certiorari the Decision of NLRC. NLRC set
the thesis that respondents (then complainants) were not regular employees of petitioner aside and vacated the Labor Arbiter’s finding that there was no illegal dismissal.
company.
- 11 August 2000, the Court of Appeals reversed and set aside the ruling of the FACTS
voluntary arbitrator ; (Petitioners were declared regular employees of Coca Cola Bottlers; - According to the Labor arbiter, the respondents refused to work and/or were choosy in
dismissal illegal; ordered to reinstate the workers) the kind of jobs they wanted to perform. NLRC found that the record is replete with the
workers’ persistence and determination of going back to work.
ISSUES - When the union was certified as the collective bargaining representative in the
1. WON the nature of work of respondents in the company is of such nature as to be certification elections, Hacienda Fatima under the pretext that the result was on appeal,
deemed necessary and desirable in the usual business or trade of petitioner that could refused to sit down with the union for the purpose of entering into a CBA. Moreover, the
qualify them to be regular employees workers were not given work for more than one month. In protest, the Union staged a
2. WON the quitclaims executed by the 36 individual respondents were valid strike which was however settled upon the signing of a Memorandum of Agreement.
- When Company again reneged on its commitment, Union filed the complaint. For all
HELD their persistence, the risk they had to undergo in conducting a strike, complainants now
1. YES find themselves being accused of ‘refusing to work and being choosy in the kind of work
Ratio In determining whether an employment should be considered regular or non- they have to perform’.
regular, the applicable test is the reasonable connection between the particular activity - The CA affirmed that while the work of respondents was seasonal in nature, they were
performed by the employee in relation to the usual business or trade of the employer. considered to be merely on leave during the off-season and were therefore still employed
Reasoning by petitioners. Moreover, the workers enjoyed security of tenure. Any infringement upon
a. Intentionalist approach - Even while the language of law (Art 280) 8 might have been this right was deemed by the CA to be tantamount to illegal dismissal. Hence this
more definitive, the clarity of its spirit and intent, i.e., to ensure a “regular” worker’s Petition.
security of tenure, however, can hardly be doubted.
b. Although the work to be performed is only for a specific project or seasonal, where a ISSUES
person thus engaged has been performing the job for at least one year, even if the 1. WON CA erred in holding that respondents, admittedly seasonal workers, were
performance is not continuous or is merely intermittent, the law deems the repeated and regular employees, contrary to the clear provisions of Article 280 9 of the Labor Code,
continuing need for its performance as being sufficient to indicate the necessity or which categorically state that seasonal employees are not covered by the definition of
desirability of that activity to the business or trade of the employer. The employment of regular employees under paragraph 1, nor covered under paragraph 2 which refers
such person is also then deemed to be regular with respect to such activity and while exclusively to casual employees who have served for at least one year
such activity exists. 2. WON CA committed grave abuse of discretion in upholding the NLRC’s conclusion
c. “The postproduction activities” done by sales route helpers are important. The nature that private respondents were illegally dismissed, that petitioner[s were] guilty of unfair
of the work performed must be viewed from a perspective of the business or trade in its labor practice, and that the union be awarded moral and exemplary damages.”
entirety and not on a confined scope.
d. The repeated rehiring of respondent workers and the continuing need for their HELD
services clearly attest to the necessity or desirability of their services in the regular 1. NO, the CA did not err when it held that respondents were regular employees.
conduct of the business or trade of petitioner company. - The fact that respondents do not work continuously for one whole year but only for the
e. A contract of employment is impressed with public interest. The provisions of duration of the season does not detract from considering them in regular employment
applicable statutes are deemed written into the contract, and “the parties are not at liberty since in a litany of cases this Court has already settled that seasonal workers who are
to insulate themselves and their relationships from the impact of labor laws and called to work from time to time and are temporarily laid off during off-season are not
regulations by simply contracting with each other.” separated from service in said period, but merely considered on leave until re-employed.
2. YES - For respondents to be excluded from those classified as regular employees, it is not
Ratio While quitclaims executed by employees are commonly frowned upon as being enough that they perform work or services that are seasonal in nature. They must have
contrary to public policy and are ineffective to bar claims for the full measure of their legal also been employed only for the duration of one season. The evidence proves the
rights, there are, however, legitimate waivers that represent a voluntary and reasonable existence of the first, but not of the second, condition. The fact that respondents -- with
settlement of laborers’ claims which should be so respected by the Court as the law the exception of Luisa Rombo, Ramona Rombo, Bobong Abriga and Boboy Silva --
between the parties. Where the person making the waiver has done so voluntarily, with a repeatedly worked as sugarcane workers for petitioners for several years is not denied
full understanding thereof, and the consideration for the quitclaim is credible and by the latter. Evidently, petitioners employed respondents for more than one season.
reasonable, the transaction must be recognized as being a valid and binding Therefore, the general rule of regular employment is applicable.
undertaking. “Dire necessity” is not an acceptable ground for annulling the release, when - The test of WON an employee is a regular employee has been laid down in De
it is not shown that the employee has been forced to execute it. Leon v. NLRC, in which this Court held:
Disposition Questioned decision of the Court of Appeals, is AFFIRMED with - The primary standard of determining regular employment is the reasonable connection
MODIFICATION in that the “Release, Waiver and Quitclaim” executed by the thirty-six between the particular activity performed by the employee in relation to the usual trade or
(36) individual respondents are hereby declared VALID and LEGAL. business of the employer. The test is whether the former is usually necessary or
desirable in the usual trade or business of the employer. The connection can be
determined by considering the nature of the work performed and its relation to the
HACIENDA FATIMA V NATIONAL FEDERATION OF scheme of the particular business or trade in its entirety. Also if the employee has been
SUGARCANE WORKERS-FOOD AND performing the job for at least a year, even if the performance is not continuous and
GENERAL TRADE merely intermittent, the law deems repeated and continuing need for its performance as
396 SCRA 518 sufficient evidence of the necessity if not indispensability of that activity to the business.
Hence, the employment is considered regular, but only with respect to such activity and
PANGANIBAN; January 28, 2003 while such activity exists. (Abasolo v. National Labor Relations Commission)
- The sudden changes in work assignments reeked of bad faith. These changes were
implemented immediately after respondents had organized themselves into a union and
8
“Art. 280. Regular and Casual Employment. – The provisions of written agreement to
the contrary notwithstanding and regardless of the oral agreement of the parties, an employment shall be 9
deemed to be regular where the employee has been engaged to perform activities which are usually Art. 280. Regular and Casual Employment. - The provisions of written agreement to the contrary notwithstanding and
necessary or desirable in the usual business or trade of the employer, except where the employment has regardless of the oral agreement of the parties, an employment shall be deemed to be regular where the employee has
been fixed for a specific project or undertaking the completion or termination of which has been determined been engaged to perform activities which are usually necessary or desirable in the usual business or trade of the
at the time of the engagement of the employee or where the work or services to be performed is seasonal in employer, except where the employment has been fixed for a specific project or undertaking the completion or
nature and the employment is for the duration of the season. termination of which has been determined at the time of the engagement of the employee or where the work or
“An employment shall be deemed to be casual if it is not covered by the preceding paragraph: Provided, services to be performed is seasonal in nature and the employment is for the duration of the season.
That, any employee who has rendered at least one year of service, whether such service is continuous or An employment shall be deemed to be casual if it is not covered by the preceding paragraph: Provided, That, any
broken, shall be considered a regular employee with respect to the activity in which he is employed and his employee who has rendered at least one year of service, whether such service is continuous or broken, shall be
employment shall continue while such activity exists.” considered a regular employee with respect to the activity in which he is employed and his employment shall continue
while such activity exist.”
Labor Law 1 A2010 - 79 - Disini
started demanding collective bargaining. Those who were union members were where the work or services to be performed is seasonal in nature and the employment is
effectively deprived of their jobs. Petitioners’ move actually amounted to unjustified for the duration of the season. We need not depart from the rulings of the Court in the
dismissal of respondents, in violation of the Labor Code. two aforementioned cases which indeed constitute stare decisis with respect to the
2. NO employment status of seafarers. Petitioners make much of the fact that they have been
- Factual findings of labor officials, who are deemed to have acquired expertise in continually re-hired or their contracts renewed before the contracts expired (which has
matters within their respective jurisdictions, are generally accorded not only respect but admittedly been going on for twenty (20) years). By such circumstance they claim to
even finality. Their findings are binding on the Supreme Court. Verily, their conclusions have acquired regular status with all the rights and benefits appurtenant to it.
are accorded great weight upon appeal, especially when supported by substantial - Such contention is untenable. Undeniably, this circumstance of continuous re-hiring
evidence. Consequently, the Court is not duty-bound to delve into the accuracy of their was dictated by practical considerations that experienced crew members are more
factual findings, in the absence of a clear showing that these were arbitrary and bereft of preferred. Petitioners were only given priority or preference because of their experience
any rational basis. and qualifications but this does not detract the fact that herein petitioners are contractual
- The NLRC found herein petitioners guilty of unfair labor practice. It ruled that from employees. They can not be considered regular employees.
respondents’ refusal to bargain, to their acts of economic inducements resulting in the Disposition IN VIEW OF THE FOREGOING, THE COURT Resolved to Partially
promotion of those who withdrew from the union, the use of armed guards to prevent the GRANT Private Respondent’s Second Motion for Reconsideration and Intervenor
organizers to come in, and the dismissal of union officials and members, one cannot but FAMES’ Motion for Reconsideration in Intervention. The Decision of the National Labor
conclude that respondents did not want a union in their hacienda—a clear interference in Relations Commission dated June 1, 1993 is hereby REINSTATED with
the right of the workers to self-organization. MODIFICATION. The Private Respondents, Trans-Global Maritime Agency, Inc. and
Disposition Petition is hereby DENIED and the assailed Decision AFFIRMED. Costs Esso International Shipping Co.,Ltd. are hereby jointly and severally ORDERED to pay
against petitioners. petitioners One Hundred Percent (100%) of their total credited contributions as provided
under the Consecutive Enlistment Incentive Plan(CEIP).
MILLARES V NLRC (TRANS-GLOBAL MARITIME
AGENCY, ESSO INTERNATIONAL SHIPPING PETROLEUM SHIPPING LIMITED V NLRC (TANCHICO)
COMPANY) 491 SCRA 35
485 SCRA 307 CARPIO; June 16, 2006
KAPUNAN; July 29, 2002 FACTS
- On 6 March 1978, Esso International Shipping (Bahamas) Co., Ltd., ("Esso") through
FACTS
Trans-Global Maritime Agency, Inc. ("Trans-Global") hired Florello W. Tanchico
- Petitioner Douglas Millares was employed by private respondent ESSO International
("Tanchico") as First Assistant Engineer. In 1981, Tanchico became Chief Engineer.
Shipping Company LTD. (Esso Int.) through its local manning agency, private respondent
- On 13 October 1992, Tanchico returned to the Philippines for a two-month vacation
Trans-Global Maritime Agency, Inc. (Trans-Global) as a machinist. In 1975, he was
after completing his eight-month deployment.
promoted as Chief Engineer. He was then receiving a monthly salary of US $1,939.00.
- On 8 December 1992, Tanchico underwent the required standard medical examination
On June 13, 1989, Millares applied for a leave of absence for the period July 9 to August
prior to boarding the vessel. The medical examination revealed that Tanchico was
7, 1989. Trans-global approved it. He then informed Esso Int. of his intention to avail of
suffering from "Ischemic Heart Disease, Hypertensive Cardio-Muscular Disease and
the optional retirement plan under the Consecutive Enlistment Incentive Plan (CEIP)
Diabetes Mellitus." Tanchico took medications for two months and a subsequent stress
considering that he had already rendered more than twenty (20) years of continuous
test showed a negative result. However, Esso no longer deployed Tanchico. Instead,
service. But the denied petitioner Millares’ request for optional retirement on the
Esso offered to pay him benefits under the Career Employment Incentive Plan. Tanchico
following grounds, to wit: (1) he was employed on a contractual basis; (2) his contract of
accepted the offer.
enlistment (COE) did not provide for retirement before the age of sixty (60) years; and (3)
- On 26 April 1993, Tanchico filed a complaint against Esso, Trans-Global and Malayan
he did not comply with the requirement for claiming benefits under the CEIP, i.e., to
Insurance Co., Inc. ("Malayan") before the Philippine Overseas Employment
submit a written advice to the company of his intention to terminate his employment
Administration (POEA) for illegal dismissal with claims for backwages, separation pay,
within thirty (30) days from his last disembarkation date. He then requested for an
disability and medical benefits and 13th month pay.
extension of his leave of absence from August 9 to 24, 1989. But the company told him
that they have promoted a First Assistant Engineer to his position as a result of his
ISSUES
previous leave of absence which expired last August 8, 1989. The adjustment in said
1. WON Tanchico is a regular employee of petitioners
rank was required in order to meet manpower schedules as a result of his inability. Esso
2. WON Tanchico is entitled to 13th month pay, disability benefits and attorney’s fees
International told Millares that in view of his absence without leave, which is equivalent to
abandonment of his position, he had been dropped from the roster of crew members
HELD
effective September 1, 1989.
1. NO
- On the other hand, petitioner Lagda was employed by private respondent Esso
- The Court squarely passed upon the issue in Millares v. NLRC17 where one of the
International as wiper/oiler in June 1969. He was promoted as Chief Engineer in 1980, a
issues raised was whether seafarers are regular or contractual employees whose
position he continued to occupy until his last COE expired on April 10, 1989. He also filed
employment are terminated everytime their contracts of employment expire. The Court
for a leave of absence and informed the company of his intention to avail the early
explained:
retirement. His request was denied on the same grounds and he too was dropped from
[I]t is clear that seafarers are considered contractual employees. They can not be
work.
considered as regular employees under Article 280 of the Labor Code. Their
- On October 5, 1989, petitioners Millares and Lagda filed a complaint-affidavit for illegal
employment is governed by the contracts they sign everytime they are rehired and
dismissal and non-payment of employee benefits against private respondents Esso
their employment is terminated when the contract expires. Their employment is
International and Trans-Global, before the POEA. POEA dismissed it for lack of merit.
contractually fixed for a certain period of time. They fall under the exception of Article
NLRC affirmed.
280 whose employment has been fixed for a specific project or undertaking the
completion or termination of which has been determined at the time of engagement of
ISSUE
the employee or where the work or services to be performed is seasonal in nature and
WON the petitioners are contractual employees whose employment are terminated
the employment is for the duration of the season. We need not depart from the rulings
everytime their contracts expire
of the Court in the two aforementioned cases which indeed constitute stare decisis
with respect to the employment status of seafarers.
HELD
- The circumstance of continuous re-hiring was dictated by practical considerations that
YES
experienced crew members are more preferred. Petitioners were only given priority or
- it is clear that seafarers are considered contractual employees. They can not be
preference because of their experience and qualifications but this does not detract the
considered as regular employees under Article 280 of the Labor Code. Their
fact that herein petitioners are contractual employees. They can not be considered
employment is governed by the contracts they sign everytime they are rehired and their
regular employees.
employment is terminated when the contract expires. Their employment is contractually
2. On 13th Month Pay
fixed for a certain period of time. They fall under the exception of Article 280 whose
- The Court of Appeals premised its grant of 13th month pay on its ruling that Tanchico
employment has been fixed for a specific project or undertaking the completion or
was a regular employee. The Court of Appeals also ruled that petitioners are not exempt
termination of which has been determined at the time of engagement of the employee or
Labor Law 1 A2010 - 80 - Disini
from the coverage of PD 851 which requires all employers to pay their employees a 13th dismissed seafarer is entitled to indemnity equivalent to his salary for the unexpired term
month pay. of his employment contract or three months for every year of the unexpired term,
- We do not agree with the Court of Appeals. Again, Tanchico was a contractual, not a whichever is less.
regular, employee. Further, PD 851 does not apply to seafarers. - The award by the Arbiter of the peso equivalent of the dollar awards cannot be enforced
- Tanchico’s employment is governed by his Contract of Enlistment. The Contract has as the same is contrary to law. The peso equivalent must be computed at the exchange
been approved by the POEA in accordance with Title I, Book One of the Labor Code and rate computed at the time of payment as provided for by RA 8183.
the POEA Rules Governing Employment. Hence, in the absence of any provision in his Disposition The questioned decision is affirmed with the modification that the dollar
Contract governing the payment of 13th month pay, Tanchico is not entitled to the benefit. award should be payable in its peso equivalent computed at the prevailing rate of
On Disability Benefits exchange at the time of payment.
- Since Tanchico received compensation during his vacation, the Contract did not
terminate on the day he returned to Manila. The Contract remained in force during
PENTAGON INTERNATIONAL SHIPPING INC V
Tanchico’s vacation period.
- However, the Court of Appeals erred when it ruled that Tanchico is entitled to disability ADELANTAR
benefits of 18 days for every year of service. The Court of Appeals ruled that Tanchico’s 435 SCRA 342
employment was continuous and that his tenure with petitioners was for 14 years. Again, YNARES-SANTIAGO; July 27, 2001
the Court of Appeals assumed that Tanchico was a regular employee. The Court of
Appeals failed to consider that Tanchico’s employment terminated with the end of each NATURE
contract. Petition for review on certiorari of CA decision which modified an NLRC decision
- Indications that Tanchico was suffering from ischemia were detected on 8 December
1992 during Tanchico’s vacation period. Thus, petitioners paid him disability benefits for FACTS
18 days in accordance with the Contract. Tanchico cannot claim that he only acquired the - August 16, 1997 > William B. Adelantar was hired by Dubai Ports Authority of Jebel Ali
illness during his last deployment since the Medical Report 26 he submitted to the NLRC under an employment contract (first contract) which provided for an unlimited period of
showed that he has been hypertensive since 1983 and diabetic since 1987. In the employment with a monthly salary of Dhs 5,500.
absence of concrete proof that Tanchico acquired his disability during his last deployment - September 3, 1997 > Adelantar and Pentagon International Shipping, Inc, for and in
and not during his vacation, he is only entitled to disability benefits for 18 days. behalf of Dubai Ports Authority of Jebel Ali, entered into a Philippine Overseas
Employment Administration (POEA) standard employment contract (second contract),
SKIPPERS UNITED PACIFIC INC V NLRC (CA & this time providing for a 12-month period with basic monthly salary of US$380.00 and
fixed overtime pay of US$152.00.
ROSAROSO) - April 5, 1998 > Adelantar’s basic salary was increased to Dhs 5,890 and overtime pay
494 SCRA 66 was increased to Dhs 2,356
AUSTRIA-MARTINEZ; July 12, 2006 - June 11, 1998 > Dubai Ports barred Adelantar from entering the port due to a previous
dispute with his superior. On the same date, he was given a letter as he was terminated
for assaulting his superior officer, although he was promised employment in another
NATURE company.
Appeal from a decision of the CA - Adelantar filed a complaint for illegal dismissal with money claim against Pentagon with
the NLRC
FACTS - LABOR ARBITER: found the dismissal of Adelantar was illegal and ordered Pentagon
- Private respondent Gervasio Rosaroso was employed as a Third Engineer with to pay Adelantar the amount of Dhs 24,738.00 representing the latter’s three (3) months
Nicolakis Shipping, S.A., a foreign firm through its recruitment and manning agency, basic salary inclusive of overtime pay
petitioner Skippers. The employment contract was for the period of one year beginning - NLRC: affirmed the Labor Arbiter’s decision and held that in Section 10 of RA8042
July 10, 1997 with a salary of $800 per month and other benefits. Rosaroso boarded M/V (Migrant Workers and Overseas Filipinos Act of 1995) an illegally dismissed contract
Naval Gent on July 15, 1997. He was however ordered to disembark in Bulgaria on worker is entitled to the salaries corresponding to the unexpired portion of his contract, or
August 7, 1997 and repatriated to the Philippines. for three (3) months for every year of the unexpired term, whichever is less. They
- Soon after arrival in Manila, respondent filed a complaint for illegal dismissal and awarded backwages to Adelantar equivalent to three (3) months of his basic salary, but
monetary claims. The Labor Arbiter found the respondent was in fact illegally dismissed exclusive of overtime pay
and issued an order directing petitioner, Skippers, to pay Rosaroso separation pay of - CA: September 26, 2002, CA modified the amounts awarded by the Labor Arbiter and
$2,4000 or the equivalent of P100,000, representing three months pay and unpaid salary the NLRC and instead awarded full backwages computed from the time of the dismissal
for seven days of $186.69 or the equivalent of P7,840.98. Atorney’s fees of P5,000 was up to the finality of the decision because Section 10 of R.A. No 8042 is not applicable
also awarded. The NLRC and the CA affirmed en toto the ruling of the Arbiter. because said provision only contemplates a fixed period of employment and that A279
- Hence this appeal to the SC. LC should apply considering that Adelantar’s first contract provided for an unlimited
period of employment.
ISSUE
WON private respondent Rosaroso was illegal dismissed ISSUE
WON A279 LC should apply given the first contract provided for an unlimited period of
HELD employment
YES
- The employer of Rosaroso did not provide the quantum of evidence needed to prove HELD
that dismissal was in fact for cause. The evidence presented was just a telefax coming NO. Sec 10 RA 8042 should apply because the second contract (with POEA), which
from the alleged Chief Engineer of the vessel which the Arbiter up to the CA considered provided for a fixed period of 1 year as employment, is applicable at bar. Also, landmark
as mere hearsay. While the Master of the vessel was grated under Paragraph D of case of Millares v NLRC applies
Section 17 of the Philippine Overseas employment Administration (POEA) Standard Ratio It is clear that seafarers are considered contractual employees. They can not be
Employment condition governing the employment of Filipino Seafarers on Board Ocean considered as regular employees under A280 LC. Their employment is governed by the
Going Vessels the power to dismiss for just cause without furnishing the seafarer with a contracts they sign every time they are rehired and their employment is terminated when
notice of dismissal if doing so will prejudice the safety of the crew and the vessel, the SC the contract expires. Their employment is contractually fixed for a certain period of time.
noted that the complete report on the circumstances of the dismissal was not forwarded They fall under the exception of A280 LC whose employment has been fixed for a
to the manning agency as called for under the same provision. specific project or undertaking the completion or termination of which has been
Minor issues determined at the time of engagement of the employee or where the work or services to
- The award of backwages and separation pay in lieu of reinstatement as provided for in be performed is seasonal in nature and the employment is for the duration of the season.
Article 279 of the Labor Code is not applicable in this case. The Seafarer is a contractual (Millares v NLRC)
employee whose rights and obligations are governed by the POEA Employment Contract Reasoning
and by RA 8042 (1995). The Employment contract does not provide for the award of - Coyoca v NLRC: Filipino seamen are governed by the Rules and Regulations of the
separation or termination pay. However, under Section 10 of RA 8042 the award of POEA. The Standard Employment Contract governing the Employment of All Filipino
money claims in cases of illegal dismissal is allowed. Under this provision, an illegal Seamen on Board Ocean-Going Vessels of the POEA, particularly in Part I, Sec. C
Labor Law 1 A2010 - 81 - Disini
specifically provides that the contract of seamen shall be for a fixed period. In no case that his exposure to their field of operation was as fabricator, helper/electrician,
should the contract of seamen be longer than 12 months and any extension of the stockman/timekeeper. This proves that he was regularly and continuously employed by
Contract period shall be subject to the mutual consent of the parties. Audion in various job assignments from 1976 to 1989, for a total of 13 years. The alleged
- It should be stressed that whatever status of employment or increased benefits that the gap in employment service does not defeat his regular status as he was rehired for many
complainant may have gained while under the employ of Dubai Ports Authority, the more projects without interruption and performed functions which are vital, necessary
undisputed fact remains that prior to his deployment, he agreed to be hired under a 12- and indispensable to the usual business of petitioner.
month POEA contract, the duration of which is the basis for the determination of the - Audion could have presented substantial evidence to support its claim that Madolid was
extent of the respondent’s liability. a project worker, like the employment contract (which stated the employee’s nature of
- Moreover, it is an accepted maritime industry practice that employment of seafarers is employment) or reports of termination (which were required by DOLE upon termination of
for a fixed period only. Constrained by the nature of their employment which is quite the project, and failure to submit this is an indication of regular status of an employee as
peculiar and unique in itself, it is for the mutual interest of both the seafarer and the held in cases), but it did not.
employer why the employment status must be contractual only or for a certain period of 2. NO
time. Seafarers spend most of their time at sea and understandably, they can not stay Ratio Due process is not denied when one is afforded the opportunity to be heard and
for a long and an indefinite period of time at sea. Limited access to shore society during present his case, but the same decided not to take the opportunity.
the employment will have an adverse impact on the seafarer. The national, cultural and Reasoning
lingual diversity among the crew during the COE is a reality that necessitates the - Madolid clearly specified in his affidavit the specific dates in which he was not paid
limitation of its period. overtime pay, project allowances, 13 th month pay, and wage adjustments. The claim of
Disposition Petition is partly GRANTED and CA decision is REVERSED and SET Audion that it paid him such must be proved by evidence, which it did not do (despite of
ASIDE. Petitioner Pentagon International Shipping, Inc. is ORDERED to pay private having the burden to prove the claim).
respondent William B. Adelantar the amount equivalent to the unexpired portion of the - In fact, records show that the company did not appear in hearings, which the court took
September 3, 1997 POEA Standard Contract of Employment plus ten percent (10%) of to be a waiver of its right to be heard.
the award as attorney’s fees. - However, award to moral and exemplary damages and attorney’s fees are deleted for
being devoid of moral basis.
Disposition Petition denied, resolutions affirmed with modifications (deletion of award
LOPEZ V MWSS of damages and attorney’s fees)
[PAGE 60]
BETA ELECTRIC CORP V NLRC (BETA ELECTRIC
AUDION ELECTRIC CO INC V NLRC (MADOLID) EMPLOYEES ASSOCIATION, PETILLA)
308 SCRA 340 182 SCRA 384
GONZAGA-REYES; June 17, 1999 SARMIENTO; February 15, 1990
NATURE NATURE
Petition for certiorari, seeking annulment of resolution of the NLRC (of which the Petition to review the decision of the National Labor Relations Commission affirming the
presiding officer was our very own Dean Carale ) judgment of the labor arbiter reinstating the private respondent with backwages.

FACTS FACTS
- Madolid was employed by Audion Electric Co. on June 30, 1976 as fabricator and - The petitioner hired the private respondent as clerk typist for one month, which
continuously rendered service in different offices and projects as helper technician, appointment was extended five times in five months (one month /contract).Her
stockman, and timekeeper. He rendered 13 years of service with a clean record. On appointments were covered by corresponding written contracts. On June 22, 1987, her
August 3, Madolid received a letter informing him that he will be considered terminated services were terminated without notice or investigation. On the same day, she went to
after the turnover of materials, including company’s tools and equipments not later than the labor arbiter on a complaint for illegal dismissal. Both the labor arbiter and the
August 15, 1989. respondent National Labor Relations Commission ruled for her.Petioner claims the
- Madolid claims that he was dismissed without justifiable cause and due process and private respondent’s appointment was temporary and hence she may be terminated at
that his dismissal was done in bad faith which renders the dismissal illegal. For this will.
reason, he claims that he is entitled to reinstatement with full backwages, and moral and
exemplary damages. He also includes payment of his overtime pay, project allowance, ISSUES
minimum wage increase adjustment, proportionate 13th month pay and attorney's fees. WON the fact that private respondent’s employment has been a contract-to-contract
Audion rebuts his allegations by saying that the employment contract of Madolid was one basis alters the character of her employment as a regular employee
that was co-terminus with the project, thus he should not be considered as a regular
employee. Also, the company contends that it had paid all the alleged unpaid wages. HELD
- The Labor arbiter decided the case in favor of Madolid, ordering Audion to pay him NO
backwages, OT pay, project allowances, min. wage increase adjustment, 13 th month pay, Ratio . The fact that her employment has been a contract-to-contract basis can not alter
and awarding him moral and exemplary damages and attorney’s fees. Appeal to NLRC the character of employment, because contracts can not override the mandate of law..
was dismissed. Reasoning
- private employee was employed from December 15, 1986 until June 22, 1987 when
ISSUES she was ordered laid off. Her tenure having exceeded six months, she attained regular
1. WON Madolid was a regular employee, thus entitling him to backwages, etc. employment.
2. WON Audion was denied due process with the award of all the claims of Madolid - petitioner can not rightfully say that since the private respondent's employment hinged
from contract to contract, it was ergo, "temporary", depending on the term of each
HELD agreement. Under the Labor Code, an employment may only be said to be "temporary"
1. YES "where [it] has been fixed for a specific undertaking the completion of or termination of
Ratio Where the employment of project employees is extended long after the supposed which has been determined at the time of the engagement of the employee or where the
project has been finished, the employees are removed from the scope of project work or services to be performed is seasonal in nature and the employment is for the
employees and considered regular employees. duration of the season." Quite to the contrary, the private respondent's work, that of
Reasoning "typist-clerk" is far from being "specific" or "seasonal", but rather, one, according to the
- (citing NLRC’s decision): Audion’s assigning Madolid to its various projects did not Code, "where the employee has been engaged to perform activities which are usually
make him a project worker. As found by the Labor Arbiter, “it appears that complainant necessary or desirable in the usual business." And under the Code, where one performs
was employed by respondent xxx as fabricator and or projects as helper electrician, such activities, he is a regular employee, "[t]he provisions of written agreement to the
stockman and timekeeper.' Simply put, complainant was a regular non-project worker.” contrary notwithstanding.
- Madolid’s employment status was established by the Certification of Employment dated Disposition Petition DISMISSED. Private respondent is ordered REINSTATED with
April 10, 1989 issued by Audion which certified that private respondent is a bonafide backwages equivalent to three years with no qualification or deductions.
employee from June 30, 1976 up to the time of issuance on April 10, 1989. This showed
Labor Law 1 A2010 - 82 - Disini
Disposition petition is DENIED DUE COURSE. The Decision of the Court of Appeals is
AFFIRMED.
UNIVERSAL ROBINA CORPORATION V CATAPANG
473 SCRA 189
CALLEJO, SR; October 14, 2005 MARAGUINOT V NLRC (DEL ROSARIO, VIVA FILMS)
284 SCRA 539
FACTS DAVIDE; January 22, 1998
- Petitioner Universal Robina Corporation is a corporation duly organized and existing
under the Philippine laws, while petitioner Randy Gregorio is the manager of the NATURE
petitioner company’s duck farm in Calauan, Laguna. Special civil action for certiorari seeking to annul the decision of NLRC and its Resolution
- The individual respondents were hired by the petitioner company on various dates from
1991 to 1993 to work at its duck farm in Barangay Sto. Tomas, Calauan, Laguna. The FACTS
respondents were hired under an employment contract which provided for a five-month - Petitioner Alejandro Maraguinot, Jr. maintains that he was employed by private
period. After the expiration of the said employment contracts, the petitioner company respondents as part of the filming crew. About 4 months later, he was designated Asst.
would renew them and re-employ the respondents. This practice continued until Electrician. He was then promoted to the rank of Electrician.
sometime in 1996, when the petitioners informed the respondents that they were no - Petitioner Paulino Enero claims that private respondents employed him as a member of
longer renewing their employment contracts. the shooting crew.
- In October 1996, the respondents filed separate complaints for illegal dismissal, - Petitioners’ tasks consisted of loading, unloading and arranging movie equipment in the
reinstatement, backwages, damages and attorney’s fees against the petitioners. The shooting area as instructed by the cameraman, returning the equipment to Viva Films’
complaints were later consolidated. On March 30, 1999, after due proceedings, the warehouse, assisting in the “fixing” of the lighting system, and performing other tasks that
Labor Arbiter rendered a decision in favor of the respondents, which NLRC and the CA the cameraman and/or director may assign.
affirmed. - Petitioners requested that private respondents adjust their salary in accordance with the
- On appeal, the petitioners submit that the respondents are not regular employees. They minimum wage law. Petitioners were informed that Mr. Vic del Rosario would agree to
aver that it is of no moment that the respondents have rendered service for more than a increase their salary only if they signed a blank employment contract. As petitioners
year since they were covered by the five-month individual contracts to which they duly refused to sign, private respondents forced Enero to go on leave then refused to take him
acquiesced. The petitioners contend that they were free to terminate the services of the back when he reported for work. Meanwhile, Maraguinot was dropped from the company
respondents at the expiration of their individual contracts. The petitioners maintain payroll but was returned and again asked to sign a blank employment contract, and when
that, in doing so, they merely implemented the terms of the contracts. he still refused, private respondents terminated his services. Petitioners thus sued for
- The petitioners assert that the respondents’ contracts of employment were not intended illegal dismissal before the Labor Arbiter.
to circumvent security of tenure. They point out that the respondents knowingly and - Private respondents claim that Viva Films is primarily engaged in the distribution and
voluntarily agreed to sign the contracts without the petitioners having exercised any exhibition of movies, but not in the business of making movies; in the same vein, private
undue advantage over them. Moreover, there is no evidence showing that the petitioners respondent Vic del Rosario is merely an executive producer, i.e., the financier who
exerted moral dominance on the respondents.[\ invests a certain sum of money for the production of movies distributed and exhibited by
VIVA; that they contract persons called “producers” -- also referred to as “associate
ISSUE producers”-- to “produce” or make movies for private respondents; and that petitioners
WON the respondent employees of the corporation are regular employees and therefore are project employees of the associate producers who, in turn, act as independent
their termination for causes outside of the Labor Code is patently illegal contractors. As such, there is no employer-employee relationship between petitioners
and private respondents; that it was the associate producer of a film who hired
HELD Maraguinot.and he was released upon payment of his last salary, as his services were no
YES longer needed; that Enero was hired for a movie, went on vacation and by the time he
Ratio An employee shall be deemed to be of regular status when he has been reported back to work the move had been completed.
performing a job for at least one year even if the performance is not continuous and - The Labor Arbiter found that:
merely intermittent. -- complainants are the employees of the respondents. The producer cannot be
Reasoning considered as an independent contractor but should be considered only as a labor-only
- In any case, we find that the CA, the NLRC and the Labor Arbiter correctly categorized contractor and as such, acts as a mere agent of the real employer, the herein
the respondents as regular employees of the petitioner company. In Abasolo v. National respondents. Also, it is an admitted fact that the complainants received their salaries from
Labor Relations Commission, the Court reiterated the test in determining whether one is the respondents. It is very clear also that complainants are doing activities which are
a regular employee: necessary and essential to the business of the respondents, that of movie-making.
- The primary standard, therefore, of determining regular employment is the Complainant Maraguinot worked as an electrician while complainant Enero worked as a
reasonable connection between the particular activity performed by the crew [member]. Hence, the complainants were illegally dismissed.
employee in relation to the usual trade or business of the employer. The test is - Private respondents appealed to the NLRC. In its decision, it said that:
whether the former is usually necessary or desirable in the usual business or 1. Complainants were hired for specific movie projects and their employment was co-
trade of the employer. The connection can be determined by considering the terminus with each movie project the completion/termination of which are pre-
nature of work performed and its relation to the scheme of the particular determined, such fact being made known to complainants at the time of their
business or trade in its entirety. Also, if the employee has been performing the engagement.
job for at least a year, even if the performance is not continuous and merely 2. Each shooting unit works on one movie project at a time. And the work of the shooting
intermittent, the law deems repeated and continuing need for its performance units, which work independently from each other, are not continuous in nature but
as sufficient evidence of the necessity if not indispensability of that activity to depends on the availability of movie projects.
the business. Hence, the employment is considered regular, but only with 3. Further shown by respondents is the irregular work schedule of complainants on a
respect to such activity and while such activity exists. daily basis. Maraguinot was supposed to report on 05 August 1991 but reported only on
- It is obvious that the said five-month contract of employment was used by petitioners as 30 August 1991, or a gap of 25 days. Complainant Enero worked on 10 September 1991
a convenient subterfuge to prevent private respondents from becoming regular and his next scheduled working day was 28 September 1991, a gap of 18 days.
employees. Such contractual arrangement should be struck down or disregarded as 4. The extremely irregular working days and hours of complainants’ work explain the
contrary to public policy or morals. To uphold the same would, in effect, permit petitioners lump sum payment for complainants’ services for each movie project. Hence,
to avoid hiring permanent or regular employees by simply hiring them on a temporary or complainants were paid a standard weekly salary regardless of the number of working
casual basis, thereby violating the employees’ security of tenure in their jobs. Petitioners’ days and hours they logged in. Otherwise, if the principle of “no work no pay” was strictly
act of repeatedly and continuously hiring private respondents in a span of … 3 to 5 years applied, complainants’ earnings for certain weeks would be very negligible.
to do the same kind of work negates their contention that private respondents were hired 5. Respondents also alleged that complainants were not prohibited from working with
for a specific project or undertaking only. other movie companies.
- Further, factual findings of labor officials who are deemed to have acquired expertise in The NLRC, in reversing the Labor Arbiter, then concluded that these circumstances,
matters within their respective jurisdiction are generally accorded not only respect but taken together, indicated that complainants (herein petitioners) were “project employees.”
even finality, and bind us when supported by substantial evidence. Petitioners’ Claim To support their claim that they were regular (and not project)
employees of private respondents, petitioners cited their performance of activities that
Labor Law 1 A2010 - 83 - Disini
were necessary or desirable in the usual trade or business of private respondents and preceding standards, the associate producers of VIVA cannot be considered labor-only
added that their work was continuous, i.e., after one project was completed they were contractors as they did not supply, recruit nor hire the workers.
assigned to another project. Reasoning (On control test)
Respondents Private respondents reiterate their version of the facts and stress that - VIVA’s control is evident in its mandate that the end result must be a “quality film
their evidence supports the view that petitioners are project employees; point to acceptable to the company.” The means and methods to accomplish the result are
petitioners’ irregular work load and work schedule; emphasize the NLRC’s finding that likewise controlled by VIVA, viz., the movie project must be finished within schedule
petitioners never controverted the allegation that they were not prohibited from working without exceeding the budget, and additional expenses must be justified; certain scenes
with other movie companies; and ask that the facts be viewed in the context of the are subject to change to suit the taste of the company; and the Supervising Producer, the
peculiar characteristics of the movie industry. “eyes and ears” of VIVA and del Rosario, intervenes in the movie-making process by
The Office of the Solicitor General (OSG) is convinced that this petition is improper assisting the associate producer in solving problems encountered in making the film.
since petitioners raise questions of fact; and submits that petitioners’ reliance on Article - Aside from control, the element of selection and engagement is likewise present in the
280 of the Labor Code to support their contention that they should be deemed regular instant case and exercised by VIVA. A sample appointment slip was offered by private
employees is misplaced, as said section “merely distinguishes between two types of respondents “to prove that members of the shooting crew except the driver are project
employees, i.e., regular employees and casual employees, for purposes of determining employees of the Independent Producers. Notably, nowhere in the appointment slip does
the right of an employee to certain benefits.” The OSG likewise rejects petitioners’ it appear that it was the producer or associate producer who hired the crew members;
contention that since they were hired not for one project, but for a series of projects, they moreover, it is VIVA’s corporate name which appears on the heading of the appointment
should be deemed regular employees. In closing, the OSG disagrees with petitioners’ slip. What likewise tells against VIVA is that it paid petitioners’ salaries as evidenced by
claim that the NLRC’s classification of the movie producers as independent contractors vouchers, containing VIVA’s letterhead, for that purpose.
had no basis in fact and in law, since, on the contrary, the NLRC “took pains in explaining 3. YES
its basis” for its decision. Ratio A project employee or a member of a work pool may acquire the status of a regular
employee when the following concur: 1) There is a continuous rehiring of project
ISSUES employees even after cessation of a project; and 2) The tasks performed by the alleged
1. WON this is a proper action “project employee” are vital, necessary and indispensable to the usual business or trade
2. WON an employer-employee relationship existed between the petitioners and private of the employer. However, the length of time during which the employee was
respondents or any one of them continuously re-hired is not controlling, but merely serves as a badge of regular
3. WON petitioners were illegally dismissed employment.
- In the instant case, the evidence on record shows that petitioner Enero was employed
HELD for a total of two (2) years and engaged in at least eighteen (18) projects, while petitioner
1. YES Maraguinot was employed for some three (3) years and worked on at least twenty-three
Ratio We rule that a special civil action for certiorari under Rule 65 of the Rules of Court (23) projects. Moreover, as petitioners’ tasks involved, among other chores, the loading,
is the proper remedy for one who complains that the NLRC acted in total disregard of unloading and arranging of movie equipment in the shooting area as instructed by the
evidence material to or decisive of the controversy. In the instant case, petitioners allege cameramen, returning the equipment to the Viva Films’ warehouse, and assisting in the
that the NLRC’s conclusions have no basis in fact and in law, hence the petition may not “fixing” of the lighting system, it may not be gainsaid that these tasks were vital,
be dismissed on procedural or jurisdictional grounds. necessary and indispensable to the usual business or trade of the employer. As regards
2. YES the underscored phrase, it has been held that this is ascertained by considering the
Ratio The relationship between VIVA and its producers or associate producers seems to nature of the work performed and its relation to the scheme of the particular business or
be that of agency, as the latter make movies on behalf of VIVA, whose business is to trade in its entirety.
“make” movies. As such, the employment relationship between petitioners and Reasoning
producers is actually one between petitioners and VIVA, with the latter being the direct - It may not be ignored, however, that private respondents expressly admitted that
employer. petitioners were part of a work pool; and, while petitioners were initially hired possibly as
The employer-employee relationship between petitioners and VIVA can further be project employees, they had attained the status of regular employees in view of VIVA’s
established by the “control test.” While four elements are usually considered in conduct.
determining the existence of an employment relationship, namely: (a) the selection and - At this time, we wish to allay any fears that this decision unduly burdens an employer by
engagement of the employee; (b) the payment of wages; (c) the power of dismissal; and imposing a duty to re-hire a project employee even after completion of the project for
(d) the employer’s power to control the employee’s conduct, the most important element which he was hired. The import of this decision is not to impose a positive and sweeping
is the employer’s control of the employee’s conduct, not only as to the result of the work obligation upon the employer to re-hire project employees. What this decision merely
to be done but also as to the means and methods to accomplish the same. These four accomplishes is a judicial recognition of the employment status of a project or work pool
elements are present here. employee in accordance with what is fait accompli, i.e., the continuous re-hiring by the
Reasoning (On job contracting) employer of project or work pool employees who perform tasks necessary or desirable to
It is settled that the contracting out of labor is allowed only in case of job contracting. 10 the employer’s usual business or trade. Let it not be said that this decision “coddles”
- Assuming that the associate producers are job contractors, they must then be engaged labor, for as Lao12 has ruled, project or work pool employees who have gained the status
in the business of making motion pictures. As such, and to be a job contractor under the of regular employees are subject to the “no work-no pay” principle.
preceding description, associate producers must have tools, equipment, machinery, work - The Court’s ruling here is meant precisely to give life to the constitutional policy of
premises, and other materials necessary to make motion pictures.The associate strengthening the labor sector, but, we stress, not at the expense of management. Lest it
producer did not have substantial capital nor investment in the form of tools, equipment be misunderstood, this ruling does not mean that simply because an employee is a
and other materials necessary for making a movie. If private respondents insist that their project or work pool employee even outside the construction industry, he is deemed, ipso
associate producers are labor contractors, then these producers can only be “labor-only” jure, a regular employee. All that we hold today is that once a project or work pool
contractors.11 employee has been: (1) continuously, as opposed to intermittently, re-hired by the same
- As labor-only contracting is prohibited, the law considers the person or entity engaged employer for the same tasks or nature of tasks; and (2) these tasks are vital, necessary
in the same a mere agent or intermediary of the direct employer. But even by the and indispensable to the usual business or trade of the employer, then the employee
must be deemed a regular employee, pursuant to Article 280 of the Labor Code and
jurisprudence.
10
Section 8, Rule VIII, Book III of the Omnibus Rules Implementing the Labor Code describes permissible job Disposition instant petition is GRANTED.
contracting in this wise:
Sec. 8. Job contracting. -- There is job contracting permissible under the Code if the following conditions are met:
(1) The contractor carries on an independent business and undertakes the contract work on his own account under his ABESCO CONSTRUCTION AND DEVELOPMENT
own responsibility according to his own manner and method, free from the control and direction of his employer or
principal in all matters connected with the performance of the work except as to the results thereof; and
CORPORATION V RAMIREZ
(2) The contractor has substantial capital or investment in the form of tools, equipment, machineries, work premises,
and other materials which are necessary in the conduct of his business. 12
11 A work pool may exist although the workers in the pool do not receive salaries and are free to seek other
Art. 106. Contractor or subcontractor.-- x x x There is “labor-only” contracting where the person supplying employment during temporary breaks in the business, provided that the worker shall be available when called
workers to an employer does not have substantial capital or investment in the form of tools, equipment, to report for a project. Although primarily applicable to regular seasonal workers, this set-up can likewise be
machineries, work premises, among others, and the workers recruited and placed by such persons are applied to project workers insofar as the effect of temporary cessation of work is concerned. This is
performing activities which are directly related to the principal business of such employer. In such cases, the beneficial to both the employer and employee for it prevents the unjust situation of “coddling labor at the
person or intermediary shall be considered merely as an agent of the employer who shall be responsible to expense of capital” and at the same time enables the workers to attain the status of regular employees.
the workers in the same manner and extent as if the latter were directly employed by him.
Labor Law 1 A2010 - 84 - Disini
487 SCRA 9 - Furthermore, petitioners cannot belatedly argue that respondents continue to be their
employees (so as to escape liability for illegal dismissal). Before the LA, petitioners
CORONA; April 10, 2006 staunchly postured that respondents were only “project employees” whose employment
tenure was coterminous with the projects they were assigned to. However, before the
NATURE CA, they took a different stance by insisting that respondents continued to be their
Appeal by certiorari employees. Petitioners’ inconsistent and conflicting positions on their true relation with
respondents make it all the more evident that the latter were indeed their regular
FACTS employees.
- Petitioner company was engaged in a construction business where respondents were 2. YES
hired on different dates from 1976 to 1992 either as laborers, road roller operators, Ratio The law requires that the employer furnish the employee 2 written notices: (1) a
painters or drivers. notice informing them of the particular acts for which they are being dismissed and (2) a
- In 1997, respondents filed 2 separate complaints for illegal dismissal against the notice advising them of the decision to terminate the employment, before termination can
company and its General Manager before the Labor Arbiter (LA). Petitioners allegedly be validly effected.
dismissed them without a valid reason and without due process of law. The complaints Reasoning
also included claims for non-payment of the 13th month pay, five days’ service incentive - In resolving the issue of illegal dismissal, the SC simply stated that petitioners failed to
leave pay, premium pay for holidays and rest days, and moral and exemplary damages. adhere to the “two-notice rule,” and said that respondents were never given such notices.
The LA later ordered the consolidation of the two complaints. Disposition Petition denied
- Petitioners denied liability and countered that respondents were “project employees”
since their services were necessary only when the company had projects to be
completed. Petitioners argued that, being project employees, respondents’ employment ALU-TUCP V NLRC (NATIONAL STEEL CORP)
was coterminous with the proj. to which they were assigned. They were’nt regular 234 SCRA 678
employees who enjoyed security of tenure and entitlement to separation pay upon FELICIANO; August 2, 1994
termination from work.
- After trial, the LA declared respondents as regular employees because they belonged to
NATURE
a “work pool” from which the company drew workers for assignment to different projects,
Petition for Certiorari to review the resolutions of the NLRC
at its discretion. He ruled that respondents were hired and re-hired over a period of 18
years, hence, they were deemed to be regular employees. He likewise found that their
FACTS
employment was terminated without just cause. Thus, in its judgment, the LA declared
- Petitioners claim that they have been employed by respondent National Steel
petitioner company guilty of illegal dismissal and ordered it to reinstate the respondents
Corporation (NSC) in connection with its Five Year Expansion Program (FAYEP I and II)
to their former positions with backwages and other benefits and that if reinstatement was
for varying lengths of time when they were separated from NSC’s service.
not feasible, that separation pay be awarded.
- Petitioners filed separate complaints for unfair labor practice, regularization and
- Petitioners appealed to the NLRC which affirmed the LA’s decision. They later filed a
monetary benefits. The Labor Arbiter declared petitioners “ regular project employees who
petition for review in the CA arguing that they were not liable for illegal dismissal since
shall continue their employment as such for as long as such (project) activity exists,” but
respondents’ services were merely put on hold until the resumption of their business
entitled to the salary of a regular employee pursuant to the provisions in the collective
operations. They also averred that they had paid respondents their full wages and
bargaining agreement. It also ordered payment of salary differentials.
benefits as provided by law, hence, the latter had no more right to further benefits.
- Both parties appealed. Petitioners argued they were regular, not project employees.
- The CA, taking note of the fact that petitioners previously used the defense that the
NSC claimed petitioners are project employees as they were employed to undertake a
respondents were project employees who were not entitled to security of tenure and now
specific project.
say that the respondents were not dismissed but their employment merely suspended,
- The NLRC modified the Labor Arbiter’s decision, affirming the holding that they were
dismissed the appeal and dismissed the MFR as well.
project employees since they were hired to perform work in a specific undertaking. It,
however, set aside the award to petitioners of the same benefits enjoyed by regular
ISSUES
employees for lack of legal and factual basis.
1. WON the respondents were regular employees
- Petitioners appealed to the SC, arguing that they are “regular” employees of NSC
2. WON respondents were illegally dismissed
because: (i) their jobs are “necessary, desirable and work-related to private respondent’s
main business, steel-making”; and (ii) they have rendered service for six (6) or more
HELD
years to NSC.
1. YES
Ratio In determining the nature of one’s employment, length of service is not a
ISSUE
controlling factor
WON petitioners are properly characterized as “project employees” rather than “regular
Reasoning
employees” of NSC
- Jurisprudence: The SC ruled that respondents were regular employees but not for the
SC’s NOTE: The issue relates to an important consequence: the services of project
reasons given by the LA (which both the NLRC and the CA affirmed). Citing Palomar, et
employees are co-terminous with the project and may be terminated upon the end or
al. v. NLRC, the SC held that contrary to the disquisitions of the LA, employees (like
completion of the project for which they were hired. Regular employees, in contrast, are
respondents) who work under different project employment contracts for several years do
legally entitled to remain in the service of their employer until that service is terminated
not automatically become regular employees; they can remain as project employees
by one or another of the recognized modes of termination of service under the Labor
regardless of the number of years they work. Length of time is not a controlling factor in
Code.
determining the nature of one’s employment.
- Moreover, employees who are members of a “work pool” from which a company (like
HELD
petitioner corp.) draws workers for deployment to its different projects do not become
YES
regular employees by reason of that fact alone. The Court has enunciated in the cases of
petitioners are project employees.
Raycor Aircontrol Systems, Inc. v. NLRC , and ALU-TUCP v. NLRC, that members of a
- The law governing the matter is Article 280 of the Labor Code:
“work pool” can either be project employees or regular employees
ART. 280. Regular and casual employment. - The provisions of written
- The principal test for determining whether employees are “project employees” or
agreement to the contrary notwithstanding and regardless of the oral agreement
“regular employees” is whether they are assigned to carry out a specific project or
of the parties, an employment shall be deemed to be regular where the
undertaking, the duration and scope of which are specified at the time they are engaged
employee has been engaged to perform activities which are usually necessary
for that project. Such duration, as well as the particular work/service to be performed, is
or desirable in the usual business or trade of the employer, except where the
defined in an employment agreement and is made clear to the employees at the time of
employment has been fixed for a specific project or undertaking the completion
hiring.
or termination of which has been determined at the time of the engagement of
- Petitioners did not have that kind of agreement with respondents. Neither did they
the employee or where the work or service to be performed is seasonal in
inform the respondents of the nature of their work at the time of hiring. Hence, for failure
nature and the employment is for the duration of the season.
of petitioners to substantiate their claim that respondents were project employees, we are
An employment shall be deemed to be casual if it is not covered by the
constrained to declare them as regular employees
preceding paragraph: Provided, That any employee who has rendered at least
one year of service, whether such service is continuous or broken, shall be
Labor Law 1 A2010 - 85 - Disini
considered a regular employee with respect to the activity in which he is contracts were freely and voluntarily signed by Kiamco and the PNOC representatives.
employed and his employment shall continue while such activity exists. The contracts plainly stated that Kiamco was being hired for a specific project and for a
- As evident in Article 280 of the Labor Code, the principal test for determining whether fixed term. Therefore Kiamco could not question his dismissal since it was in accordance
particular employees are properly characterized as “project employees” as distinguished with his employment contract.
from “regular employees” is whether or not the “project employees” were assigned to - Kiamco appealed the decision of the Labor Arbiter to public respondent NLRC which on
carry out a “specific project or undertaking, the duration (and scope) of which were Sept 27 1996 reversed the Labor Arbiter and declared Kiamco as a regular employee of
specified at the time the employees were engaged for that project. the respondents and to have been illegally dismissed by the latter. Ordering respondents
- In business and industry, “project” could refer to one or the other of at least two to REINSTATE the complainant to his former position without loss of seniority rights and
distinguishable types of activities. Firstly, a project could refer to a particular job or privileges with back wages from the date of his dismissal up to actual reinstatement less
undertaking that is within the regular or usual business of the employer company, but any income he may have earned during the pendency of the case.
which is distinct and separate, and identifiable as such, from the other undertakings of - Private respondents filed a MFR of the decision of the NLRC contending that it erred in
the company. Such job or undertaking begins and ends at determined or determinable holding that Kiamco was a regular employee and that the findings of the Labor Arbiter
times. Secondly, the term “project” could also refer to a particular job or undertaking that that Kiamco was a project employee should be affirmed.
is not within the regular business of the corporation. Such job or undertaking must also - NLRC modified its Sept 27 1996 Decision declaring that “the complainant-appellant is
be identifiably separate and distinct from the ordinary or regular business operations of declared a project employee at respondent’s Geothermal Plant and to continue with said
the employer. The job or undertaking also begins and ends at determined or employment until the full completion of the project but in the absence of proof to that
determinable times. effect, complainant is hereby awarded back wages for a period of 6 months or in the
- Whichever type of project employment is found in a particular case, a common basic amount of P23,100.00. The order declaring the complainant-appellant as a regular
requisite is that the designation of named employees as “project employees” and their employee of respondent PNOC, and for said company to reinstate the complainant with
assignment to a specific project, are effected and implemented in good faith, and not full back wages is hereby deleted.”
merely as a means of evading otherwise applicable requirements of labor laws. - In his petition for certiorari, Kiamco charges the NLRC with grave abuse of discretion
- The particular component projects embraced in the FAYEP, to which petitioners were amounting to lack or excess of jurisdiction in issuing the questioned Resolution and prays
assigned, were distinguishable from the regular or ordinary business of NSC, which is that it be nullified and he reinstated to his former position. He also seeks payment of
the production or making and marketing of steel products. During the time petitioners back wages, damages and attorney’s fees.
rendered services to NSC, their work was limited to one or another of the specific
component projects which made up the FAYEP I and II. It is not shown that petitioners ISSUES
were hired for or assigned to other purposes. 1. WON petitioner is a regular employee or a project employee
Re Length of Service 2. WON petitioner is entitled to reinstatement without loss of seniority rights and
- SC affirmed the Labor Arbiter and NLRC’s basic finding that the length of service of a privileges and to the payment of full back wages
p[roject employee is not the controlling test of employment tenure but whether or not ‘the 3. WON petitioner is entitled to moral and exemplary damages.
employment has been fixed for a specific project or undertaking the completion or
termination of which has been determined at the time of the engagement of the HELD
employee’. 1. Kiamco was correctly labeled by the NLRC as a project employee.
- The simple fact that the employment of petitioners as project employees had gone -Article 280 of the Labor Code
beyond one year does not detract from, or legally dissolve, their status as project Regular and casual employment. - The provisions of written agreement to the contrary
employees. notwithstanding and regardless of the oral agreement of the parties, an employment
Disposition Petition for Certiorari is dismissed. Resolutions of NLRC affirmed. shall be deemed to be regular where the employee has been engaged to perform
activities which are usually necessary or desirable in the usual business or trade of
KIAMCO V NLRC (PNOC) the employer, except where the employment has been fixed - for a specific project or
undertaking the completion or termination of which has been determined at the time of
309 SCRA 424 the engagement of the employee or where the work or service to be performed is
BELLOSILLO; June 29, 1999 seasonal in nature and the employment is for the duration of the season.
- An employee shall be deemed to be casual if it is not covered by the preceding
FACTS paragraph: Provided, that any employee who has rendered at least one year of service,
- Private respondent PHILIPPINE NATIONAL OIL COMPANY (PNOC) through its Energy whether such service is continuous or broken, shall be considered a regular employee
Research and Development Division, hired petitioner Cisell Kiamco as a project with respect to the activity in which he is employed and his employment shall continue
employee in its Geothermal Agro-Industrial Plant Project in Valencia, Negros Oriental. while such activity exists.
The Contract of Employment1 stipulated among others that Kiamco was being hired by - In Violeta v. NLRC [10 October 1997, 280 SCRA 520.] it was held -
the company as a technician for a period of 5 months from July 1 1992 to Nov 30 1992, The principal test for determining whether particular employees are properly
or up to the completion of the project, whichever would come first. characterized as "project employees," as distinguished from "regular employees," is
- After the termination of the contract, a 2nd one was entered into by the parties whether or not the "project employees" were assigned to carry out a "specific project or
containing basically the same terms and conditions. The period of employment was from undertaking," the duration (and scope) of which were specified at the time the employees
Dec 1 1992 to April 30 1993. were engaged for that project. As defined, project employees are those workers hired (1)
- Kiamco was again re-hired for 6 months (May 1 1993 to Nov 30 1993) for a specific project or undertaking, and (2) the completion or termination of such project
- On Oct 20 1993 Kiamco received a Memorandum from the administration department or undertaking has been determined at the time of engagement of the employee.
demanding an explanation from him on certain infractions he allegedly committed: 1. - Under Policy Instruction No. 20 of the Secretary of Labor, project employees are those
Misconduct 2. Absence without official leave (AWOL) 3. Non-compliance of employed in connection with a particular project. Non-project or regular employees are
administrative reporting procedure on accidents 4. Unauthorized use of company those employed without reference to any particular project.
vehicles - The three Contracts of Employment entered into by Kiamco clearly established that he
- Kiamco tried to explain his side but private respondents found his explanation was a project employee because (a) he was specifically assigned to work for a particular
unsatisfactory. On Oct 28 1993 Kiamco received a Memorandum placing him under project, which was the Geothermal Agro-Industrial Demonstration Plant Project of private
preventive suspension from Nov 1 1993 to Nov 30 1993 pending further investigation. No respondents, and (b) the termination and the completion of the project or undertaking
investigation however was ever conducted. Private respondents contended that an was determined and stipulated in the contract at the time of his employment.
investigation was not necessary since Kiamco had ceased to be an employee ipso facto 2. YES
upon the expiration of his employment contract on Nov 30 1993. - In Santos v. NLRC (154 SCRA 166) it was held -
- On Dec 1 1993 Kiamco reported back to work but was prevented by security guards The normal consequences of a finding that an employee has been illegally dismissed
from entering the company premises. On May 27 1994 private respondent reported to are, that the employee becomes entitled to reinstatement to his former position
the Department of Labor and Employment that petitioner Kiamco was terminated on Nov without loss of seniority rights and the payment of back wages.
1 1993 due to the expiration of his employment contract and the abolition of his position. - Reinstatement restores the employee who was unjustly dismissed to the position from
- On April 25 1994 Kiamco filed before the NLRC Sub-Regional Arbitration Branch No. VII which he was removed, that is, to his status quo ante dismissal; while the grant of back
a Complaint for illegal suspension and dismissal against the PNOC. He prayed that he wages allows the same employee to recover from the employer that which he had lost by
be reinstated to his former position and paid back wages. Labor Arbiter dismissed the way of wages as a result of his dismissal.
complaint for lack of merit. According to the Labor Arbiter, the three (3) employment
Labor Law 1 A2010 - 86 - Disini
- The argument of private respondents that reinstatement and payment of back wages SANDOVAL V NLRC
could not be made since Kiamco was not a regular employee is apparently misplaced. As
136 SCRA 675
quoted above, the normal consequences of an illegal dismissal are the reinstatement of
the aggrieved employee and the grant of back wages. These rights of an employee do AQUINO; May 31, 1985
not depend on the status of his employment prior to his dismissal but rather to the legality
and validity of his termination. The fact that an employee is not a regular employee does
not mean that he can be dismissed any time, even illegally, by his employer. NATURE
3. NO Appeal by certiorari
- Moral damages are recoverable only where the dismissal of the employee was
attended with bad faith or fraud or constituted an act oppressive to labor or was done in a FACTS
manner contrary to morals, good custom or public policy. Exemplary damages, on the - 5 workers were assigned to the construction of the LCT Catamaran. After three months
other hand, may be awarded only if the dismissal was effected in a wanton, oppressive of work, the project was completed and the five workers were served a termination
or malevolent manner. The evidence on record does not show any fraud, malice or bad notice. The termination was reported to the Ministry of Labor. The workers filed a
faith on the part of private respondents that would justify payment to petitioner of moral complaint for illegal dismissal.
and exemplary damages. - The Labor Arbiter ordered the reinstatement of the workers with backwages. The NLRC
affirmed.
- 55 workers were assigned to work in the construction of a tanker. When the tanker was
PHIL. JAI-ALAI & AMUSEMENT CORP V CLAVE finished, the personel manager of Sandoval Shipyards terminated the services of the
126 SCRA 299 welders, helpers, and construction workers. The termination was duly reported to the
MELENCIO-HERRERA; December 21, 1983 Ministry of Labor. 17 workers filed a complaint for illegal dismissal.
- The Director of the Ministry’s Capital Region ordered the reinstatement of the
complainants. The Deputy Minister of Labor affirmed. Hence this petition.
NATURE
Petition for Certiorari with Preliminary Injunction
ISSUE
WON private respondents were project employees whose work was coterminous with the
FACTS
project for which they were hired
- Petitioner is a corporation operating a jai-alai fronton for sport and amusement.
- It has its own maintenance group for the upkeep of its premises. For the renovation of
HELD
its main building, which work is not included in maintenance, it hired private respondents,
YES
Cadatal, Jr., a plumber, and Delgra, a mason, together with 30 other workers on
Ratio The public respondents in the instant two cases acted with grave abuse of
February 2, 1976 for a period of one month, open to extension should the need for the
discretion amounting to lack of jurisdiction in disregarding the precedents cited by the
arise in the course of the renovation.
petitioners.
- Renovation was completed by October 1976. Management then decided to construct
Reasoning
an annex to the building and private respondents worked on the fire escape.
- Project Employees, as distinguished from regular or non-project employees, are
- November 27, 1976 – Notice of termination given to the respondents effective
mentioned in Article 281 of the Labor Code, as those “where the employment has been
November 29 but they still continued to work nonetheless. They worked until December
fixed for a specific project or undertaking the completion or termination of which has been
11 and were fully paid for the work they rendered up to that date.
determined at the time of the engagement of the employee”.
- December 13, 1976 - Petitioner filed with the former Department of Labor a report of
- The petitioner cited three of its own cases wherein the NLRC, Deputy Minister of Labor,
termination of the services of private respondents and 30 others, listing them as casual
and the Director of the National Capital Region held that the layoff of its project
emergency workers. Private workers alleged illegal termination. Assistant Minister
employees was lawful.
Leogardo ordered the reinstatement of the workers with full backwages before petitioner
- In the case of In Re: Sandoval Shipyards, Inc. Application for Clearance to Terminate
could file a reply to the letter-complaint of the respondents.
Employees, it was held that:
- Leogardo said that the respondents were already regular employees according to Art.
…It is significant to note that the corporation does not construct vessels for sale or
170 (now Art. 281) of the CC and that termination was unjust.
otherwise which will demand continuous productions of ships and will need permanent
- An appeal was filed which Clave, in his capacity as Presidential Executive Assistant,
or regular workers…
dismissed it.
ISSUE
…The completion of their work or project automatically terminates their employment…
WON private respondents are regular employees entitled to security of tenure
- The other two cases cited affirmed that the workers of the petitioner were project
employees whose employment was terminated upon the completion of the project.
HELD
- Respondent Deputy Minister himself affirmed such finding. He ruled that the
NO
complainants “are project workers whose employments are coterminous with the
Ratio Casual employees are engaged for a specific project or undertaking and fall within
completion of the project, regardless of the number of projects in which they have
the exception provided for in Article 281 of the Labor Code, supra . Not being regular
worked, as provided under Policy Instructions No. 20 of the Ministry of Labor and
employees, it cannot be justifiably said that petitioner had dismissed them without just
Employment” and “as their employment is one for a definite period, they are not entitled
cause. They are not entitled to reinstatement with full backwages.
to separation pay”.
Reasoning
Disposition REVERSED.
- A281 defines regular and casual employees. In the case at hand, the casual or limited
character of private respondents' employment, therefore, is evident.
- Private respondents were hired for a specific project - to renovate the main budding, IMBUIDO V NLRC (LIBRANDO)
where major repairs such as painting the main building, repair of the roof, cleaning of 329 SCRA 357
clogged water pipes and drains, and other necessary repairs were required.
BUENA; March 31, 2000
- It was made known, and so understood at the start of the hiring, that their services
would last until the completion of the renovation. They rendered service from February 2
to December 11, 1976, almost 11 months, but less than a year. NATURE
- There could be no other reason, however, than that the termination of private Petition for review on certiorari of the decision of the NLRC
respondents was because their services were no longer needed and they had nothing
more to do since the project for which they were hired had been completed. FACTS
Disposition Order of public respondent Vicente Leogardo, Jr., dated December 24, - Petitioner was employed as a data encoder by private respondent International
1976, and the Orders of the other public respondents dated July 13, 1977, January 25, Information Services, Inc., a domestic corporation engaged in the business of data
1979, March 19, 1979, and June 5, 1980, are hereby reversed and set aside. The encoding and keypunching, from August 26, 1988 until October 18, 1991 when her
Complaint for illegal dismissal against petitioner in Case No. R04-12-11832-76 LS services were terminated due to "low volume of work".
(Regional Office No. IV, Department of Labor) is dismissed, and the Temporary - Petitioner filed a complaint for illegal dismissal with prayer for service incentive leave
Restraining Order heretofore issued is hereby made permanent. pay and 13th month differential with NLRC alleging that her employment was terminated
not due to the low volume of work but because she "signed a petition for certification
Labor Law 1 A2010 - 87 - Disini
election among the rank and file employees of respondents," thus charging private petitioner was dismissed until her reinstatement when private respondent was not
respondent with committing unfair labor practices. undertaking any project, should be deducted.
- Private respondent maintained that it had valid reasons to terminate petitioner's - With regard to petitioner's claim for service incentive leave pay, we agree with the labor
employment and disclaimed any knowledge of the existence or formation of a union arbiter that petitioner is entitled to service incentive leave pay, as provided
among its rank-and-file employees at the time petitioner's services were terminated. in Article 95 of the Labor Code, which reads:
Private respondent stressed that its business ". . . relies heavily on companies availing of Art. 95: Right to service incentive leave �
its services. Its retention by client companies with particular emphasis on data encoding (a) Every employee who has rendered at least one year of service shall be entitled to
is on a project to project basis," usually lasting for a period of "two (2) to five (5) months." a yearly service incentive leave of five days with pay.
Private respondent further argued that petitioner's employment was for a "specific project - Having already worked for more than three (3) years at the time of her unwarranted
with a specified period of engagement." According to private respondent, ". . . the dismissal, petitioner is undoubtedly entitled to service incentive leave benefits, computed
certainty of the expiration of complainant's engagement has been determined at the time from 1989 until the date of her actual reinstatement.
of its engagement (until 27 November 1991) or when the project is earlier completed or Disposition Petition granted.
when the client withdraws," as provided in the contract. "The happening of the second
event [completion of the project] has materialized, thus, her contract of employment is
deemed terminated.
DE OCAMPO V NLRC
186 SCRA 360
ISSUE DAVIDE JR; May 7, 2002
WON petitioner is a "project employee" and not a "regular employee" who has security of
tenure NATURE
The petition seeks a reversal of the decision of the respondent NLRC (ordering
HELD respondent to reinstate, without back wages, the individual complainants who were
- We agree with the findings of the NLRC that petitioner is a project employee. The regular employees except those who were officers of the union among them or paid
principal test for determining whether an employee is a project employee or a regular separation pay at their option, equivalent to one month's pay or one-half month's pay for
employee is whether the project employee was assigned to carry out a specific project or every year of service, whichever is greater. )
undertaking, the duration and scope of which were specified at the time the employee
was engaged for that project. A project employee is one whose employment has been FACTS
fixed for a specific project or undertaking, the completion or termination of which has - On September 30, 1980, the services of 65 employees of private respondent Makati
been determined at the time of the engagement of the employee or where the work or Development Corporation were terminated on the ground of the expiration of their
service to be performed is seasonal in nature and the employment is for the duration of contracts. The said employees filed a complaint for illegal dismissal against the MDC on
the season. In the instant case, petitioner was engaged to perform activities which were October 1, 1980; On October 8, 1980, as a result of the aforementioned termination, the
usually necessary or desirable in the usual business or trade of the employer, as Philippine Transport and General Workers Association, of which the complainants were
admittedly, petitioner worked as a data encoder for private respondent, a corporation members, filed a notice of strike on the grounds of union-busting, subcontracting of
engaged in the business of data encoding and keypunching, and her employment was projects which could have been assigned to the dismissed employees, and unfair labor
fixed for a specific project or undertaking the completion or termination of which had practice; that on October 14, 1980, the PTGWA declared a strike and established picket
been determined at the time of her engagement, as may be observed from the series of lines in the perimeter of the MDC premises- On November 4, 1980, the MDC filed with
employment contracts 32 between petitioner and private respondent, all of which the Bureau of Labor Relations a motion to declare the strike illegal and restrain the
contained a designation of the specific job contract and a specific period of employment. workers from continuing the strike; that on that same day and several days thereafter the
- However, even as when petitioner is a project employee, according to jurisprudence "[a] MDC filed applications for clearance to terminate the employment of 90 of the striking
project employee or a member of a work pool may acquire the status of aregular workers, whom it had meanwhile preventively suspended; that of the said workers, 74
employee when the following concur: were project employees under contract with the MDC with fixed terms of employment;
1) There is a continuous rehiring of project employees even after the cessation of a and that on August 31, 1982, Labor Arbiter Apolinar L. Sevilla rendered a decision 1
project; and denying the applications for clearance filed by the MDC and directing it to reinstate the
2) The tasks performed by the alleged "project employee" are vital, necessary and individual complainants with two months back wages each.
indispensable to the usual business or trade of the employer. - This is the decision modified by the NLRC 2 which is now faulted by the petitioners for
grave abuse of discretion. The contention is that the public respondent acted arbitrarily
- The evidence on record reveals that petitioner was employed by private respondent as and erroneously in ruling that: a) the motion for reconsideration was filed out of time; b)
a data encoder, performing activities which are usually necessary or desirable in the the strike was illegal; and c) the separation of the project employees was justified
usual business or trade of her employer, continuously for a period of more than three (3)
years, from August 26, 1988 to October 18, 1991 and contracted for a total of thirteen ISSUES
(13) successive projects. We have previously ruled that "[h]owever, the length of time 1. WON the strike held by the workers was legal
during which the employee was continuouslyre-hired is not controlling, but merely serves 2. WON the contract workers are considered regular employees
as a badge of regular employment." Based on the foregoing, we conclude that petitioner 3. WON the project workers are entitled to separation pay
has attained the status of a regular employee of private respondent.
- Being a regular employee, petitioner is entitled to security of tenure and could only be HELD
dismissed for a just or authorized cause, as provided in Article 279 of the 1. YES
Labor Code, as amended: - under the law then in force, to wit, PD No. 823 as amended by PD No. 849, the strike
Art. 279. Security of Tenure � In cases of regular employment, the employer shall not was indeed illegal. In the first place, it was based not on the ground of unresolved
terminate the services of an employee except for a just cause or when economic issues, which was the only ground allowed at that time, when the policy was
authorized by this Title. An employee who is unjustly dismissed from work shall be indeed to limit and discourage strikes. Secondly, the strike was declared only after 6 days
entitled to reinstatement without loss of seniority rights and other privileges from the notice of strike and before the lapse of the 30-day period prescribed in the said
and to his full backwages, inclusive of allowances, and to his other benefits or their law for a cooling-off of the differences between the workers and management and a
monetary equivalent computed from the time his compensation was withheld possible avoidance of the intended strike. That law clearly provided Sec. 1. It is the policy
from him up to the time of his actual reinstatement. of the state to encourage free trade unionism and free collective bargaining within the
- The alleged causes of petitioner's dismissal (low volume of work and belatedly, framework of compulsory and voluntary arbitration. Therefore all forms of strikes,
completion of project) are not valid causes for dismissal under Articles 282 and 283 of picketing and lockout are hereby strictly prohibited in vital industries such as in public
the Labor Code. Thus, petitioner is entitled to reinstatement without loss of seniority utilities, including transportation and communication, companies engaged in the
rights and other privileges, and to her full backwages, inclusive of allowances, and to her manufacturer processing as well as in the distribution of fuel gas, gasoline and fuel or
other benefits or their monetary equivalent computed from the time her compensation lubricating oil, in companies engaged in the production or processing of essential
was withheld from her up to the time of her actual reinstatement. However, complying commodities or products for export, and in companies engaged in banking of any kind,
with the principles of "suspension of work" and "no work, no pay" between the end of one as well as in hospitals and in schools and colleges. However, any legitimate labor union
project and the start of a new one, in computing petitioner's backwages, the amounts may strike and any employer may lockout in establishments not covered by General
corresponding to what could have been earned during the periods from the date Order No. 5 only on grounds of unresolved economic issues in collective bargaining, in
Labor Law 1 A2010 - 88 - Disini
which case the union or the employer shall file a notice with the Bureau of Labor - On September 18, 1980, respondent Grulla reported back to his Project Manager and
Relations at least 30 days before the intended strike or lockout. (Emphasis supplied) presented to the latter a medical certificate declaring the former already physically fit for
The Court ruled that the leaders of the illegal strike were correctly punished with work. Since then, he stated working again until he received a notice of termination of his
dismissal, but their followers (other than the contract workers) were properly ordered employment on October 9, 1980.
reinstated, considering their lesser degree of responsibility. The penalty imposed upon - Grulla filed a complaint for illegal dismissal, recovery of medical benefits, unpaid wages
the leaders was only proper because it was they who instigated the strike even if they for the unexpired ten (10) months of his contract and the sum of P1,000.00 as
knew, or should have known, that it was illegal. It was also fair to rule that the reinstated reimbursement of medical expenses against A.M. Oreta and Company, Inc. and
strikers were not entitled to backpay as they certainly should not be compensated for ENDECO with the POEA.
services not rendered during the illegal strike. In our view, this is a reasonable - The petitioner A.M. Oreta and Company, Inc. and ENDECO filed their answer and
compromise between the demands of the workers and the rights of the employer. alleged that the contract of employment entered into between petitioners and Grulla
2. The Court stress the rule in Cartagenas v. Romago Electric Co., that contract workers provides, as one of the grounds for termination of employment, violation of the rules and
are not considered regular employees, their services being needed only when there are regulations promulgated by the contractor; and that Grulla was dismissed because he
projects to be undertaken. 'The rationale of this rule is that if a project has already been has not performed his duties satisfactorily within the probationary period of three months.
completed, it would be unjust to require the employer to maintain them in the payroll - POEA held that complainant's dismissal was illegal and warrants the award of his
while they are doing absolutely nothing except waiting until another project is begun, if at wages for the unexpired portion of the contract.
all. In effect, these stand-by workers would be enjoying the status of privileged retainers, - Petitioner appealed from the adverse decision to the respondent Commission.
collecting payment for work not done, to be disbursed by the employer from profits not - Respondent Commission dismissed the appeal for lack of merit and affirmed in toto the
earned. This is not fair by any standard and can only lead to a coddling of labor at the decision of the POEA..
expense of management. However, this rule is not applicable in the case at bar. The
record shows that although the contracts of the project workers had indeed expired, the ISSUES
project itself was still on-going and so continued to require the workers' services for its 1. WON the employment of respondent Grulla was illegally terminated by the petitioner
completion. There is no showing that such services were unsatisfactory to justify their 2. WON Grulla is entitled to salaries corresponding to the unexpired portion of his
termination. It is obvious that the real reason for the termination of their services-which, employment contract.
to repeat, were still needed-was the complaint the project workers had filed and their
participation in the strike against the private respondent. These were the acts that HELD
rendered them persona non grata to the management. Their services were discontinued 1. YES
by the MDC not because of the expiration of their contracts, which had not prevented - Article 280 (formerly Article 281) of the Labor Code, as amended, provides:
their retention or rehiring before as long as the project they were working on had not yet "Article 280. Regular and Casual Employment. - The provisions of written
been completed. The real purpose of the MDC was to retaliate against the workers, to agreement to the contrary notwithstanding and regardless of the oral agreements
punish them for their defiance by replacing them with more tractable employees. of the parties, an employment shall be deemed to be regular where the employee
3. Noteworthy in this connection is Policy Instruction No. 20 of the Department of Labor, has been engaged to perform activities which are usually necessary or desirable in
providing that "project employees are not entitled to separation pay if they are terminated the usual business or trade of the employer, except where the employment has
as a result of the completion of the project or any phase thereof in which they are been fixed for a specific project or undertaking the completion or termination of
employed, regardless of the projects in which they had been employed by a particular which has been determined at the time of the engagement of the employment or
construction company." This rule would entitle project employees to separation pay if the where the work or service to be performed is seasonal in nature and the
projects they are working on have not yet been completed when their services are employment is far the duration of the season.
terminated. And this should be true even if their contracts have expired, on the theory "An employment shall be deemed to be casual if it is not covered by the preceding
that such contracts would have been renewed anyway because their services were still paragraph: Provided, that any employee who has rendered at least one year of
needed. service, whether such service is continuous or broken, shall be considered a
- Applying this rule, The Court held that the project workers who were separated even regular employee with respect to the activity in which he is employed and his
before the completion of the project at the New Alabang Village and not really for the employment shall continue while such actually exists."
reason that their contracts had expired, are entitled to separation pay. Considering the - Policy Instructions No. 12 of the then Minister of Labor (now Secretary of Labor and
workers to have been separated without valid cause, the Court shall compute their Employment) which provides:
separation pay at the rate of one month for every year of service of each dismissed "PD 850 has defined the concept of regular and casual employment. What
employee, up to the time of the completion of the project. This is the most equitable way determines regularity or casualness is not the employment contract, written or
to treat their claim in light of their cavalier dismissal by the private respondent despite otherwise, but the nature of the job. If the job is usually necessary or desirable to
their long period of satisfactory service with it. the main business of the employer, then employment is regular. . . ."
Disposition The appealed decision of the NLRC is affirmed but with the modification - A perusal of the employment contract reveals that although the period of employment of
that the contract workers are hereby declared to have been illegally separated before the respondent Grulla is 12 months, the contract period is renewable subject to future
expiration of the project they were working on and so are entitled to separation pay agreement of the parties. It is clear from the employment contract that the respondent
equivalent to one month salary for every year of service. Grulla was hired by the company as a regular employee and not just a mere probationary
employee.
A.M. ORETA & CO INC V NLRC (GRULLA) - On the matter of probationary employment, the law in point is Article 281 (formerly
Article 252) of the Labor Code which provides in part:
176 SCRA 218 "Art. 281. Probationary Employment. - . . . . The services of an employee who has
MEDIALDEA; August 10, 1989 been engaged on a probationary basis may be terminated for a just cause or when
he fails to qualify as a regular employee in accordance with reasonable standards
NATURE made known by the employer to the employee at the time of his engagement. An
Petition for certiorari employee who is allowed to work after a probationary period shall be considered
regular employee."
FACTS - The law is clear to the effect that in all cases involving employees engaged on
- Private respondent Grulla was engaged by Engineering Construction and Industrial probationary' basis, the employer shall make known to the employee at the time he is
Development Company (ENDECO) through A.M. Oreta and Co., Inc. as a carpenter in its hired, the standards by which he will qualify as a regular employee. Nowhere in the
project in Jeddah, Saudi Arabia. employment contract executed between petitioner company and respondent Grulla is
- The contract of employment, which was entered into on June 11, 1980 was for a period there a stipulation that the latter shall undergo a probationary period for three months
of 12 months. Respondent Grulla left the Philippines for Jeddah, Saudi Arabia on August before he can quality as a regular employee. There is also no evidence on record
5, 1980. showing that the Grulla had been apprised of his probationary status and the
- On August 15, 1980, Grulla met an accident which fractured his lumbar vertebrae while requirements which he should comply in order to be a regular employee. In the absence
working at the jobsite. He was rushed to the New Jeddah Clinic and was confined there of these requisites, there is justification in concluding that respondent Grulla was a
for 12 days. regular employee at the time he was dismissed by petitioner.
- On August 27, 1980, Grulla was discharged from the hospital and was told that he could As such, he is entitled to security of tenure during his period of employment and his
resume his normal duties after undergoing physical therapy for two weeks. services cannot be terminated except for just and authorized causes enumerated under
the Labor Code and under the emloyment contract.
Labor Law 1 A2010 - 89 - Disini
Granting, in gratia argumenti, that respondent is a probationary employee, he cannot, ISSUES
likewise, be removed except for cause during the period of probation. Although a 1. WON employees hired for a definite period and whose services are necessary and
probationary or temporary employee has limited tenure, he still enjoys security of tenure. desirable in the usual business or trade of the employer are regular employees
During his tenure of employment or before his contract expires, he cannot be removed 2. WON the private respondents' five-month contracts of employment are valid
except for cause as provided for by law. 3. WON the execution by the private respondents of a "Release and Quitclaim"
- The alleged ground of unsatisfactory performance relied upon by petitioner for precluded them from questioning the termination of their services
dismissing respondent Grulla is not one of the just causes for dismissal provided in the
Labor Code. Neither is it included among the grounds for termination of employment HELD
under Article VII of the contract of employment executed by petitioner company and 1. YES
respondent Grulla. - Art. 280 of the Labor Code defines regular and casual employment. Under this
- Grulla was not, in any manner, notified of the charges against him before he was provision, there are two kinds of regular employees : (1) those who are engaged to
outrightly dismissed. Neither was any hearing or investigation conducted by the company perform activities which are necessary or desirable in the usual business or trade of the
to give the respondent a chance to be heard concerning the alleged unsatisfactory employer; and (2) those casual employees who have rendered at least one year of
performance of his work. service, whether continuous or broken, with respect to the activity in which they are
2. YES employed.
- The dismissal of Grulla violated the security of tenure under the contract of employment - In the instant case, the private respondents' activities consisted in the receiving,
which specifically provides that the contract term shall be for a period of twelve (12) skinning, loining, packing, and casing-up of tuna fish which were then exported by PFC.
calendar months. Consequently, the respondent Grulla should be paid his salary for the Indisputably, they were performing activities which were necessary and desirable in
unexpired portion of his contract of employment which is ten (10) months. petitioner's business or trade.
Disposition Petition was dismissed - not hired for a specific project or undertaking. The term "specific project or
undertaking" under Article 280 of the Labor Code contemplates an activity which is not
commonly or habitually performed or such type of work which is not done on a daily basis
PURE FOODS CORPORATION V NLRC (CORDOVA, but only for a specific duration of time or until completion; the services employed are then
CRUSIS, ET AL) necessary and desirable in the employer's usual business only for the period of time it
174 SCRA 415 takes to complete the project.
DAVIDE, JR; December 12, 1997 2. NO
- None of the criteria, under which term employment cannot be said to be in
circumvention of the law on security of tenure, had been met in the present case.
NATURE
-Brent School, Inc. v. Zamora ruling on the legality of fixed-term employment has held
Petition for certiorari
that the decisive determinant in term employment should not be the activities that the
employee is called upon to perform but the day certain agreed upon by the parties for the
FACTS
commencement and termination of their employment relationship. But, where from the
- Private respondents (numbering 906) were hired by Pure Foods Corporation (PFC) to
circumstances it is apparent that the periods have been imposed to preclude acquisition
work for a fixed period of five months at its tuna cannery plant in Tambler, General
of tenurial security by the employee, they should be struck down or disregarded as
Santos City. After the expiration of their respective contracts of employment in June and
contrary to public policy and morals.
July 1991, their services were terminated. They forthwith executed a "Release and
-Criteria under which term employment cannot be said to be in circumvention of
Quitclaim" stating that they had no claim whatsoever against PFC.
the law on security of tenure:
- 29 July 1991: private respondents filed before the NLRC Sub-Regional Arbitration
a) The fixed period of employment was knowingly and voluntarily agreed upon by the
Branch a complaint for illegal dismissal against PFC and its plant manager, Marciano
parties without any force, duress, or improper pressure being brought to bear upon the
Aganon. Labor Arbiter Arturo P. Aponesto dismissed the complaint on the ground that the
employee and absent any other circumstances vitiating his consent; or
private respondents were mere contractual workers, and not regular employees; hence,
b) It satisfactorily appears that the employer and the employee dealt with each other on
they could not avail of the law on security of tenure. The termination of their services by
more or less equal terms with no moral dominance exercised by the former over the
reason of the expiration of their contracts of employment was, therefore, justified.
latter.
- On appeal, NLRC affirmed the Labor Arbiter's decision. But on MR, NLRC held that the
- It was shown that it was really the practice of the company to hire workers on a
private respondent and their co-complainants were regular employees. It declared that
uniformly fixed contract basis and replace them upon the expiration of their contracts with
the contract of employment for five months was a "clandestine scheme employed by PFC
other workers on the same employment duration. This scheme of PFC was apparently
to stifle private respondents' right to security of tenure" and should therefore be struck
designed to prevent the private respondents and the other "casual" employees from
down and disregarded for being contrary to law, public policy, and morals. Hence, their
attaining the status of a regular employee. It was a clear circumvention of the employees'
dismissal on account of the expiration of their respective contracts was illegal.
right to security of tenure and to other benefits like minimum wage, cost-of-living
- Accordingly, the NLRC ordered PFC to reinstate the private respondents to their former
allowance, sick leave, holiday pay, and 13th month pay.
position without loss of seniority rights and other privileges, with full back wages; and in
case their reinstatement would no longer be feasible, PFC should pay them separation
3. NO
pay equivalent to one-month pay or one-half-month pay for every year of service,
- The execution by the private respondents of a "Release and Quitclaim" did not preclude
whichever is higher, with back wages and 10% of the monetary award as attorney's fees.
them from questioning the termination of their services. Generally, quitclaims by laborers
- PFC's motion for reconsideration was denied. Hence, this petition.
are frowned upon as contrary to public policy and are held to be ineffective to bar
Purefoods Corp's Contention: that the private respondents are now estopped from
recovery for the full measure of the workers' rights. The reason for the rule is that the
questioning their separation from petitioner's employ in view of their express conformity
employer and the employee do not stand on the same footing.
with the five-month duration of their employment contracts; that the "Release and
Disposition Petition dismissed. NLRC decision affirmed, subject to modification on the
Quitclaim" private respondents had executed has unconditionally released PFC from any
computation of the separation pay and back wages. Since reinstatement is no longer
and all other claims which might have arisen from their past employment with PFC.
possible because PFC's tuna cannery plant had, admittedly, been close in November
OSG's Comment: the private respondents were regular employees, since they
1994, the proper award is separation pay equivalent to one month pay or one-half month
performed activities necessary and desirable in the business or trade of PFC. The period
pay for every year of service, whichever is higher, to be computed from the
of employment stipulated in the contracts of employment was null and void for being
commencement of their employment up to the closure of the tuna cannery plant. The
contrary to law and public policy, as its purpose was to circumvent the law on security of
amount of back wages must be computed from the time the private respondents were
tenure. The expiration of the contract did not, therefore, justify the termination of their
dismissed until the time petitioner's cannery plant ceased operation.
employment. Also, private respondents' quitclaim was ineffective to bar the enforcement
for the full measure of their legal rights.
Private Respondent's Argument: contracts with a specific period of employment may LABAYOG V MY SAN BISCUITS INC
be given legal effect provided, however, that they are not intended to circumvent the 494 SCRA 486
constitutional guarantee on security of tenure; the practice of PFC in hiring workers to
CORONA; July 11, 2006
work for a fixed duration of five months only to replace them with other workers of the
same employment duration was apparently to prevent the regularization of these so-
called "casuals," which is a clear circumvention of the law on security of tenure. NATURE
Petition for review on certiorari is the resolution of the Court of Appeals.
Labor Law 1 A2010 - 90 - Disini
- On 20 August 1985, private respondents Andres Paguio, Pablo Canale, Ruel Pangan,
FACTS Aurelio Paguio, Rolando Trinidad, Romeo Tapang and Carlos Maliwat (hereinafter
- 1992: petitioners entered into contracts of employment with respondent company as referred to as respondents) filed a Petition with the SSC for SSS coverage and
mixers, packers and machine operators for a fixed term. On the expiration of their contributions against petitioner Reynaldo Chua, owner of Prime Mover Construction
contracts, their services were terminated. Development, claiming that they were all regular employees of the petitioner in his
- April 15, 1993: petitioners filed complaints for illegal dismissal, underpayment of wages, construction business. Private respondents alleged that petitioner dismissed all of them
non-payment of overtime, night differential and 13th month pay, damages and attorney’s without justifiable grounds and without notice to them and to the then Ministry of Labor
fees. The labor arbiter ruled their dismissal to be illegal on the ground that they had and Employment. They further alleged that petitioner did not report them to the SSS for
become regular employees who performed duties necessary and desirable in respondent compulsory coverage in flagrant violation of the Social Security Act.
company’s business.. - On the other hand, the petitioner claimed that private respondents were project
- On appeal to the National Labor Relations Commission (NLRC), the decision of the employees, whose periods of employment were terminated upon completion of the
labor arbiter was set aside. project. Thus, he claimed, no employer-employee relation existed between the parties.
- CA set aside the NLRC decision and reinstated the decision of the labor arbiter. There being no employer-employee relationship, private respondents are not entitled to
However, on respondents’ motion for reconsideration, the CA reversed itself. The CA coverage under the Social Security Act. Moreover, petitioner invokes the defense of good
reasoned that, while petitioners performed tasks which were necessary and desirable in faith, or his honest belief that project employees are not regular employees under Article
the usual business of respondent company, their employment contracts providing for a 280 of the Labor Code. The SSC and CA ruled in favor of the respondents.
fixed term remained valid. No force, duress, intimidation or moral dominance was exerted
on them. Respondents dealt with petitioners in good faith and within the valid parameters ISSUE
of management prerogatives. WON private respondents were regular employees of the petitioner

ISSUE HELD
WON the contract with a fixed period is valid (therefore determining WON the workers YES
were dismissed illegally). Ratio Elements of the control test: (a) selection and engagement of the employee; (b)
payment of wages; (c) the power of dismissal; and (d) the power of control with regard to
HELD the means and methods by which the work is to be accomplished, with the power of
YES, contract is valid. control being the most determinative factor.
Ratio Contracts of employment for a fixed period are not unlawful. What is objectionable - Even though the employer does not admit, the existence of an employer-employee
is the practice of some scrupulous employers who try to circumvent the law protecting relationship between the parties can easily be determined by the application of the
workers from the capricious termination of employment. "control test, the elements of which are: (a) selection and engagement of the employee;
Reasoning (b) payment of wages; (c) the power of dismissal; and (d) the power of control with
- Petitioners were not regular employees. While their employment as mixers, packers regard to the means and methods by which the work is to be accomplished, with the
and machine operators was necessary and desirable in the usual business of respondent power of control being the most determinative factor.
company, they were employed temporarily only, during periods when there was - There is no dispute that private respondents were employees of petitioner. Petitioner
heightened demand for production. There could have been no illegal dismissal when himself admitted that they worked in his construction projects, although the period of their
their services were terminated on expiration of their contracts. employment was allegedly co-terminus with their phase of work. It is clear that private
ART. 280. (Labor Code) Regular and Casual Employment. – The provisions of respondents are employees of petitioner, the latter having control over the results of the
written agreement to the contrary notwithstanding and regardless of the oral work done, as well as the means and methods by which the same were accomplished.
agreement of the parties, an employment shall be deemed to be regular where the Suffice it to say that regardless of the nature of their employment, whether it is regular or
employee has been engaged to perform activities which are usually necessary or project, private respondents are subject of the compulsory coverage under the SSS Law,
desirable in the usual business of the employer, except where the employment has their employment not falling under the exceptions provided by the law. This rule is in
been fixed for a specific project or undertaking the completion or termination of which accord with the Court’s ruling in Luzon Stevedoring Corp. v. SSS to the effect that all
has been determined at the time of the engagement of the employee or where the employees, regardless of tenure, would qualify for compulsory membership in the SSS,
work or services to be performed is seasonal in nature and the employment is for the except those classes of employees contemplated in Section 8(j) of the Social Security
duration of the season. Act.
Where the duties of the employee consist of activities which are necessary or - In Violeta v. NLRC, this Court ruled that to be exempted from the presumption of
desirable in the usual business of the employer, the parties are not prohibited from regularity of employment, the agreement between a project employee and his employer
agreeing on the duration of employment. must strictly conform to the requirements and conditions under Article 280 of the Labor
- Article 280 does not proscribe or prohibit an employment contract with a fixed period [11] Code. It is not enough that an employee is hired for a specific project or phase of work.
provided it is not intended to circumvent the security of tenure. There must also be a determination of, or a clear agreement on, the completion or
Two criteria validate a contract of employment with a fixed period: termination of the project at the time the employee was engaged if the objectives of
(1) The fixed period of employment was knowingly and voluntarily agreed upon Article 280 are to be achieved. This second requirement was not met in this case.
by the parties without any force, duress or improper pressure being brought - This Court has held that an employment ceases to be co-terminus with specific projects
to bear on the employee and without any circumstances vitiating consent; when the employee is continuously rehired due to the demands of the employer’s
(2) It satisfactorily appears that the employer and employee dealt with each business and re-engaged for many more projects without interruption. The Court likewise
other on more or less equal terms with no moral dominance whatever being takes note of the fact that, as cited by the SSC, even the National Labor Relations
exercised by the former on the latter. Commission in a labor case involving the same parties, found that private respondents
Disposition Resolution of CA is affirmed. were regular employees of the petitioner.
- Another cogent factor militates against the allegations of the petitioner. In the
proceedings before the SSC and the Court of Appeals, petitioner was unable to show
CHUA V CA (SOCIAL SECURITY COMMISSION, SSS, that private respondents were appraised of the project nature of their employment, the
PAGUIO ET AL) specific projects themselves or any phase thereof undertaken by petitioner and for which
440 SCRA 121 private respondents were hired. He failed to show any document such as private
TINGA; October 6, 2004 respondents’ employment contracts and employment records that would indicate the
dates of hiring and termination in relation to the particular construction project or phases
in which they were employed. Moreover, it is peculiar that petitioner did not show proof
NATURE
that he submitted reports of termination after the completion of his construction projects,
This is a petition for review of the Decision of the Court of Appeals in CA-G.R. CV No.
considering that he alleges that private respondents were hired and rehired for various
38269 dated 06 March 1996, and its Resolution dated 30 July 1996 denying petitioner’s
projects or phases of work therein
Motion for Reconsideration, affirming the Order of the Social Security Commission (SSC)
- To be exempted from the presumption of regularity of employment, the agreement
dated 1 February 1995 which held that private respondents were regular employees of
between a project employee and his employer must strictly conform to the requirements
the petitioner and ordered petitioner to pay the Social Security System (SSS) for its
and conditions under Article 280 of the Labor Code. It is not enough that an employee is
unpaid contributions, as well as penalty for the delayed remittance thereof.
hired for a specific project or phase of work. There must also be a determination of, or a
FACTS
Labor Law 1 A2010 - 91 - Disini
clear agreement on, the completion or termination of the project at the time the employee - NLRC ruled in favor of Acedilo. It held that Acedillo was a regular employee, as seen
was engaged if the objectives of Article 280 are to be achieved. from the nature of his job and the length of time he has served. The petitioner was also
held liable for the monetary benefits being claimed by Acedillo since employees, whether
regular or not, are entitled to such.
C.E. CONSTRUCTION CORP V CIOCO
437 SCRA 648 ISSUE
PUNO; September 8, 2004 WON Acedillo is a regular employee

NATURE HELD
Two (2) petitions for review of the Decision of the CA which reversed the NLRC, as well YES
as its Resolution which denied the parties’ separate motions for reconsideration. - definition of project employee: a project employee is one whose "employment has been
fixed for a specific project or undertaking, the completion or termination of which has
been determined at the time of the engagement of the employee or where the work or
FACTS services to be performed is seasonal in nature and the employment is for the duration of
- Cioco, et al. (WORKERS) were hired by C.E. Construction Corp. (COMPANY), a the season
domestic corporation engaged in the construction business. They were hired as - petitioner did not specify the duration and scope of the undertaking at the time
carpenters and laborers in various construction projects from 1990 to 1999, the latest of Acedillo's services were contracted. Neither is there any proof that the duration of his
which was the GTI Tower in Makati. Prior to the start of every project, the WORKERS assignment was made clear to him other than the self-serving assertion of petitioner that
signed individual employment contracts. the same can be inferred from the tasks he was made to perform
- Sometime in May and June 1999, the WORKERS, along with sixty-six (66) others, were - Acedillo’s work clearly was an activity "necessary or desirable in the usual business or
terminated by the COMPANY on the ground of completion of the phases of the GTI trade" of petitioner, since refrigeration requires considerable electrical work. This
Tower project for which they had been hired. Alleging that they were regular employees, necessity is further bolstered by the fact that petitioner would hire him anew after the
the WORKERS filed complaints for illegal dismissal with the NLRC. Claims for underpaid completion of each project, a practice which persisted throughout the duration of his
wages and unpaid overtime pay, premium for holiday and rest days, service incentive tenure
leave pay, night shift differential, and 13th month pay were likewise demanded. - Petitioner’s assertion that it held 2 sets of workers. This practice renders untenable
petitioner's position that Acedillo is not a regular employee. Citing Philippine National
ISSUES Construction Corp v NLRC: "Members of a work pool from which a construction company
1. WON the WORKERS were regular employees of the COMPANY draws its project employees, if considered employees of the construction company while
2. WON the WORKERS were illegally dismissed in the work pool, are non-project employees or employees for an indefinite period. If they
are employed in a particular project, the completion of the project or any phase thereof
HELD will not mean severance of (the) employer-employee relationship."
1. NO Disposition Petition dismissed
Ratio The Labor Arbiter, the NLRC, and the CA, unanimously found that the WORKERS
were project employees of the COMPANY. This finding is binding on this Court. We again ABESCO CONSTRUCTION AND DEVELOPMENT CO V
hold that the fact that the WORKERS have been employed with the COMPANY for
RAMIREZ
several years on various projects, the longest being nine (9) years, did not automatically
make them regular employees considering that the definition of regular employment in [PAGE 83]
Article 280 of the Labor Code, makes specific exception with respect to project
employment. The re-hiring of petitioners on a project-to-project basis did not confer upon PALOMARES V NLRC (NATIONAL STEEL
them regular employment status.
CORPORATION)
2. NO
Ratio The labor arbiter categorically found that the appropriate notices to the WORKERS 277 SCRA 439
and the corresponding reports were submitted by the COMPANY to the DOLE. The ROMERO; August 15, 1997
NLRC affirmed this finding of fact on appeal. The rule is that factual findings of
administrative agencies, if supported by substantial evidence, are entitled to great FACTS
weight. More importantly, no prior notice of termination is required if the termination is - Palomares and Mutia was hired by respondent National Steel Corporation (NSC) by
brought about by completion of the contract or phase thereof for which the worker has virtue of contracts of employment for its Five Year Expansion Program or FYEP, Phase I
been engaged. and II-4, for varying lengths of time. Palomares and Mutia asked for regularization, wage
Disposition The decision of the CA is MODIFIED. The termination of the WORKERS is differential, CBA coverage and other benefits. Palomares, Mutia and four other
declared valid and legal. The award of backwages is set aside. complainants were adjudged as regular employees of NSC. The NLRC reversed the
findings of the Labor Arbiter. Respondent Commission held that petitioners were project
MARAGUINOT V NLRC employees and that their assumption of regular jobs were mainly due to peakloads or the
absence of regular employees during the latter's temporary leave.
[PAGE 82] - Petitioners argue that as regards functions and duration of work, contracted employees
should, by operation of law, be considered regular employees. Respondent NSC, on the
AGUILAR V NLRC (ACEDILLO) other hand, maintains that petitioners are mere project employees, engaged to work on
269 SCRA 596 the latter's Five-Year Expansion Projects (FYEP), Phases I and II-A, hence, dismissible
upon the expiration of every particular project.
ROMERO; March 13, 1997 - Petitioners were employed for a specific project or projects undertaken by respondent
corporation such as the Five Year Expansion Program include the setting up of a Cold
NATURE Rolling Mill Expansion Project, establishing a Billet Steel-Making Plant, installation of a
Petition for certiorari Five Stand TDM and Cold Mill Peripherals Project.
- Petitioners were hired to work on projects for FYEP I and II-A as shown in the records.
FACTS On account of the expiration of their contracts of employment and/or project completion,
- Romeo Acedillo worked for the petitioner as a helper-electrician. He was dismissed petitioners were terminated from their employment. They were, however, rehired for other
from allegedly due to lack of available projects and excess in the number of workers component projects of the FYEP because they were qualified. Thus, the Court is
needed. convinced that petitioners were engaged only to augment the workforce of NSC for its
- He filed a case for illegal dismissal before the NLRC upon learning that the petitioner as aforesaid expansion program.
hiring new workers and his request to be reinstated was denied. In reply, petitioner
maintained that its need for workers varied, depending on contracts procured in the ISSUE
course of its business of contracting refrigeration and other related works. According to WON petitioners should be considered regular employees of respondent corporation
them, Acedillo was a contractual employee.
HELD
Labor Law 1 A2010 - 92 - Disini
NO 1) The contracts of employment of Puente show that he was hired for specific projects.
- They should not be. It should be noted that there were intervals in petitioners' His employment was coterminous with the completion of the projects for which he had
respective employment contracts with NSC, thus bolstering the latter's position that, been hired. Those contracts expressly provided that his tenure of employment depended
indeed, petitioners are project employees. Since its work depends on availability of such on the duration of any phase of the project or on the completion of the construction
contracts or projects, necessarily the employment of its work force is not projects. Also, petitioners regularly submitted to DOLE reports of the termination of
permanent but co-terminous with the projects to which they are assigned and services of project workers. Such compliance with the requirement confirms that
from whose payrolls they are paid. It would be extremely burdensome for their respondent was a project employee
employer to retain them as permanent employees and pay them wages even if there are 2) Evidently, although the employment contract did not state a particular date, it did
no projects to work on. The fact that petitioners worked for NSC under different project specify that the termination of the parties’ employment relationship was to be on a “day
employment contracts for several years cannot be made a basis to consider them as certain” -- the day when the phase of work termed “Lifting & Hauling of Materials” for the
regular employees, for they remain project employees regardless of the number of “World Finance Plaza” project would be completed. Thus, respondent cannot be
projects in which they have worked. considered to have been a regular employee. He was a project employee.
Even if petitioners were repeatedly and successively re-hired on the basis of a contact of 3) That he was employed with Filsystems for 10 yrs. in various projects did not ipso facto
employment for more than one year, they cannot be considered regularized. Length of make him a regular employee, considering that the definition of regular employment in
service is not the controlling determinant of the employment tenure of a project Art.280 of the Labor Code makes a specific exception with respect to project
employee. As stated earlier, it is based on whether or not the employment has employment. The mere rehiring of respondent on a project-to-project basis did not
been fixed for a specific project or undertaking, the completion of which has confer upon him regular employment status. The practice was dictated by the practical
been determined at the time of the engagement of the employee. Furthermore, consideration that experienced construction workers are more preferred. It did not
the second paragraph of Article 280, providing that an employee who has rendered change his status as a project employee.
service for at least one (1) year, shall be considered a regular employee, pertains to 2. YES
casual employees and not to project employees such as petitioners . Ratio In termination cases, the burden of proving that an employee has been lawfully
-The principal test for determining whether an employee is a project employee dismissed lies with the employer. Employers who hire project employees are mandated
and not a regular employee is whether he was assigned to carry out a specific to state and, once its veracity is challenged, to prove the actual basis for the latter’s
project or undertaking, the duration and scope of which were specified at the dismissal.
time he was engaged for that project. Reasoning
Disposition instant petition is DISMISSED. The decision and resolution of the National - There is no allegation or proof, however, that the World Finance Plaza project -- or the
Labor Relations Commission dated November 23, 1994 and March 23, 1995, phase of work therein to which respondent had been assigned -- was already completed
respectively, are AFFIRMED. by Oct.1, 99, the date when he was dismissed. The inescapable presumption is that his
services were terminated for no valid cause prior to the expiration of the period of his
employment; hence, the termination was illegal. Reinstatement with full back wages,
FILIPINAS PRE-FABRICATED BUILDING SYSTEMS inclusive of allowances and other benefits or their monetary equivalents -- computed
INC V PUENTE from the date of his dismissal until his reinstatement -- is thus in order. If reinstatement
453 SCRA 820 no longer possible due to the completion of the project during the pendency of this case,
PANGANIBAN; March 18, 2005 he must be entitled to salary and benefits of the unexpired portion of his employment.
Disposition Petition is PARTLY GRANTED.
NATURE
Petition for Review

FACTS
- Respondent Puente’s contention :
> That he began working with Petitioner Filsystems, Inc., a corporation engaged in
construction business, on June 12, 1989; that he was initially hired by [petitioner]
company as an ‘installer’; that he was later promoted to mobile crane operator and was
stationed at the company premises in Quezon City; that his work was not dependent on
the completion or termination of any project; that since his work was not dependent on
any project, his employment with the Filsystems was continuous and without interruption
for the past 10 years; that on Oct. 1, 1999, he was dismissed from his employment
allegedly because he was a project employee. He filed complaint for illegal dismissal
against the petitioner.
- Petitioner-company’s claims
> That complainant was hired as a project employee in the company’s various projects;
that his employment contracts showed that he was a project worker with specific project
assignments; that after completion of each project assignment, his employment was
likewise terminated and the same was correspondingly reported to the DOLE.
Labor Arbiter dismissed complaint. NLRC affirmed. CA reversed LA and NLRC rulings
holding that respondent was a regular employee of petitioners.

ISSUES
1. WON respondent Roger Puente is a project employee
2. WON he is entitled to reinstatement with full back wages

HELD
1. YES
Ratio Provisions in the Labor Code and DOLE Order No. 19(1993) make it clear that a
project employee is one whose “employment has been fixed for a specific project or
undertaking the completion or termination of which has been determined at the time of
the engagement of the employee or where the work or services to be performed is
seasonal in nature and the employment is for the duration of the season. It is a settled
rule that “the length of service of a project employee is not the controlling test of
employment tenure but whether or not ‘the employment has been fixed for a specific
project or undertaking the completion or termination of which has been determined at the
time of the engagement of the employee.
Reasoning

You might also like